You are on page 1of 422

https://t.

me/joinchat/zyFOIg0j5goxY2E1

Telegram Group Invite Link: https://t.me/USMLEWorldStep1

epiglottitus

This febrile, ill-appearing patient with respiratory distress who is in the tripod position likely
has epiglottitis, a rare, potentially fatal infection most commonly caused by Haemophilus
influenzae type b. Edema of the epiglottis can cause laryngeal obstruction that leads to
inspiratory stridor.
During inspiration, faster airflow causes decreased intraluminal pressure in the airways (eg,
drop in intralaryngeal pressure). In patients with airway obstruction at the level of the larynx,
this decreased pressure leads to increased airway narrowing and increased turbulent
airflow. Therefore, patients with airway obstruction at the level of the larynx (eg,
epiglottitis, laryngomalacia, bilateral vocal cord paralysis) would be expected to
have inspiratory stridor. Tripod positioning (extending neck and chin to the sniffing position)

Telegram Group Invite Link: https://t.me/joinchat/zyFOIg0j5goxY2E1


https://t.me/joinchat/zyFOIg0j5goxY2E1

USMLE e e r m h nne s
Me est i e s: https://t.me/USMLEMe est

K p n ssr m i e s: https://t.me/USMLEK p n

USMLE i e s Step : https://t.me/USMLE Step

e ker i e s: https://t.me/USMLE e ker

r s e n i e s Step : https://t.me/USMLE r sAn e n

r s e n i e s Step K: https://t.me/USMLE r sAn e n Step K

Ph se i e s: https://t.me/USMLEPh se

Pi ri e i e s: https://t.me/USMLEPi ri e

P th m i e s: https://t.me/USMLEP th m

nA i s i h it : https://t.me/USMLE nA i s

Am ss r p: https://t.me/USMLEAm ss

Le t ri nk: https://t.me/USMLELe t ri

En p int P s: https://t.me/USMLEEn p int

USMLE MEs: https://t.me/USMLE ME

L E Me E : https://t.me/USMLE n ineMe E

USMLE r Step : https://t.me/USMLE r Step

USMLE r Step K: https://t.me/USMLE r Step K

USMLE r Step nk: https://t.me/USMLE r Step

USMLE Anki: https://t.me/USMLEAnki

KissPrep: https://t.me/USMLEKissPrep
Telegram Group Invite Link: https://t.me/USMLEWorldStep1

pulls the tongue forward to partially open the laryngeal airway.

(Choice A) Asymmetric breath sounds can occur with unilateral obstruction below the level of
the carina (eg, foreign body aspiration) or with pleural disorders (eg, effusion,
pneumothorax). This patient's tripod positioning is more suggestive of laryngeal obstruction.
(Choice B) Rhonchi are low-pitched rattling sounds produced when air flows through bronchi
that have excessive secretions. Because they are caused by the movement of air through wet
secretions, rhonchi can be heard on both inspiration and expiration.

Telegram Group Invite Link: https://t.me/joinchat/zyFOIg0j5goxY2E1


https://t.me/joinchat/zyFOIg0j5goxY2E1

Telegram Group Invite Link: https://t.me/USMLEWorldStep1

(Choice C) Expiratory stridor often occurs due to tracheal pathology (eg, tracheomalacia)
because inspiration decreases intrathoracic pressure (relative to the airways), which widens the
intrathoracic trachea. Expiration increases intrathoracic pressure, which narrows the
intrathoracic trachea, worsening the obstruction.

(Choices D and F) Wheezing (ie, high-pitched whistling noise) occurs with constriction of the
smaller airways of the lungs (eg, asthma). Because these small airways are intrathoracic, the
constriction is more pronounced during expiration due to increased intrathoracic pressure.

Telegram Group Invite Link: https://t.me/joinchat/zyFOIg0j5goxY2E1


Telegram Group Invite Link: https://t.me/USMLEWorldStep1

Pulmonary
hypertension

Telegram Group Invite Link: https://t.me/joinchat/zyFOIg0j5goxY2E1


https://t.me/joinchat/zyFOIg0j5goxY2E1

Telegram Group Invite Link: https://t.me/USMLEWorldStep1

Pulmonary hypertension causes specific morphologic findings in the branches of the


pulmonary arteries, including increased arteriolar smooth muscle thickness (medial
hypertrophy), intimal fibrosis, and significant luminal narrowing. In the setting of severe
hypertension, lesions can progress to form interlacing tufts of small vascular channels
called plexiform lesions. These changes can occur in both pulmonary hypertension due to
underlying cardiac, lung, or thrombotic disease and in pulmonary arterial hypertension (PAH).
PAH most commonly presents as dyspnea and exercise intolerance in women age 20-40. This
patient's family history is suggestive of the familial form of PAH, which is most often caused by
inactivating mutations involving the proapoptotic BMPR2 gene. The resulting increase in
endothelial and smooth muscle cell proliferation leads to vascular remodeling, elevated
pulmonary vascular resistance, and progressively elevated pulmonary arterial pressure.
Although lung transplantation is the definitive treatment for PAH, medical therapy targeting the
effects of endothelial dysfunction can help improve symptoms. Bosentan is an endothelin-
receptor antagonist that blocks the effects of endothelin (a potent vasoconstrictor that also
stimulates endothelial proliferation). Bosentan therapy decreases pulmonary arterial pressure
and lessens the progression of vascular remodeling and right ventricular hypertrophy.

(Choice B) Clopidogrel inhibits ADP-induced platelet aggregation. It is used in atherosclerotic


cardiovascular disease and to prevent acute stent thrombosis following percutaneous coronary
intervention.
(Choice C) Enalapril is an angiotensin-converting enzyme inhibitor. It is used for the treatment
of congestive heart failure, hypertension, and diabetic nephropathy.
(Choice D) Etanercept inhibits tumor-necrosis factor (TNF) activity by competitively binding to
TNF and preventing it from interacting with cell surface receptors. It is an anti-inflammatory
agent used for the treatment of rheumatoid arthritis, psoriasis, and psoriatic arthritis.

Telegram Group Invite Link: https://t.me/joinchat/zyFOIg0j5goxY2E1


Telegram Group Invite Link: https://t.me/USMLEWorldStep1

(Choice E) Indomethacin is a nonselective cyclooxygenase inhibitor that suppresses


prostaglandin synthesis. It is used as an anti-inflammatory agent and pain reliever.

Digital clubbing

The patient is a former smoker and presents with digital clubbing, a thickening and widening of
the nail bed and distal phalanges that give the fingers a bulbous drumstick
appearance. Normally, when two opposing fingers are held nail to nail, a diamond-shaped
space is formed between proximal nail folds; obliteration of this space is consistent with
clubbing.
Clubbing is observed in cardiopulmonary disease states where hypoxic-inflammatory pathways
are activated, releasing platelet and fibroblast-derived growth factors (especially vascular
endothelial growth factor [VEGF]) to promote platelet clumping/embedment in the distal capillary
beds (fingers, toes), driving local fibrovascular proliferation.
Lung adenocarcinoma (ie, tumor-secreted VEGF) is the most common cause of clubbing in
adults. Other common causes of clubbing include chronic chest infections (eg, bronchiectasis)
and cardiopulmonary diseases with hypoxemia (eg, cyanotic heart defects). Occasionally,
clubbing is associated with a painful periosteal reaction and synovitis, known as hypertrophic
pulmonary osteoarthropathy, a paraneoplastic syndrome strongly linked to lung
adenocarcinoma.
Examination for clubbing is clinically valuable because it may be the only visible physical sign of
lung cancer, including small and surgically resectable (curable) tumors that would be otherwise
asymptomatic. Therefore, the presence of clubbing should prompt further imaging of the chest

Telegram Group Invite Link: https://t.me/joinchat/zyFOIg0j5goxY2E1


https://t.me/joinchat/zyFOIg0j5goxY2E1

Telegram Group Invite Link: https://t.me/USMLEWorldStep1

for malignancy.

(Choice A) Uncomplicated chronic obstructive pulmonary disease (COPD without hypoxemia)


alone is not associated with clubbing; clubbing in a patient with COPD should trigger further
investigation for underlying complications (eg, malignancy, bronchiectasis, pulmonary

Telegram Group Invite Link: https://t.me/joinchat/zyFOIg0j5goxY2E1


Telegram Group Invite Link: https://t.me/USMLEWorldStep1

hypertension). In addition, this patient has normal breath sounds, making COPD (eg,
emphysema, chronic bronchitis) unlikely.
(Choice B) Chronic renal failure is associated with "half-and-half" nails (Lindsay nails), a
sharply demarcated transverse leukonychia. The mechanism remains unknown but is unlikely
to be related to azotemia because the transition line disappears after renal transplantation but
not with hemodialysis.

(Choice C) Iron deficiency anemia is associated with koilonychia, or scooped concavity of the
nail bed ("spoon nails"), possibly due to chronic nutrient deficiency leading to regression of the

Telegram Group Invite Link: https://t.me/joinchat/zyFOIg0j5goxY2E1


https://t.me/joinchat/zyFOIg0j5goxY2E1

Telegram Group Invite Link: https://t.me/USMLEWorldStep1

nail matrix.

(Choice E) Psoriatic nail dystrophy is due to primary involvement of the nail by psoriasis
itself. It manifests as nail pitting, thickening, brittleness, and onycholysis (separation from the

Telegram Group Invite Link: https://t.me/joinchat/zyFOIg0j5goxY2E1


Telegram Group Invite Link: https://t.me/USMLEWorldStep1

underlying nail bed).

Telegram Group Invite Link: https://t.me/joinchat/zyFOIg0j5goxY2E1


https://t.me/joinchat/zyFOIg0j5goxY2E1

Telegram Group Invite Link: https://t.me/USMLEWorldStep1

Telegram Group Invite Link: https://t.me/joinchat/zyFOIg0j5goxY2E1


Telegram Group Invite Link: https://t.me/USMLEWorldStep1

emphysema

This patient with chronic progressive dyspnea has spirometry findings (reduced forced
expiratory volume in 1 second [FEV1]/forced vital capacity [FVC] ratio) that are consistent
with obstructive lung disease. His young age and family history of lung and liver
disease suggest emphysema due to alpha-1 antitrypsin deficiency. The diagnosis is further
suggested by his minimal smoking history; chronic obstructive pulmonary disease due to
tobacco exposure alone is most commonly seen in those with a >30-pack-year smoking history.
Alpha-1 antitrypsin is the major inhibitor of neutrophil elastase, and deficiency leads to
unchecked elastase-mediated tissue damage in the lungs. Because alpha-1 antitrypsin is
deficient throughout the acinus, the entirety of the acinus is affected, resulting in panacinar
emphysema. The lower lung lobes are predominantly affected, likely due to relatively greater
perfusion compared to the upper lung lobes, allowing for an increased rate of neutrophil

Telegram Group Invite Link: https://t.me/joinchat/zyFOIg0j5goxY2E1


https://t.me/joinchat/zyFOIg0j5goxY2E1

Telegram Group Invite Link: https://t.me/USMLEWorldStep1

infiltration.

(Choice A) Centriacinar emphysema is characteristic of tobacco-related emphysema, as only


the portion of the acinus most exposed to smoke particles is affected. Tobacco-related
centriacinar emphysema predominantly affects the upper lung lobes, possibly due to a higher
ventilation-perfusion ratio in those regions.
(Choice B) Colonization of the lung with pathogenic bacteria most commonly occurs in patients
with cystic fibrosis and is not a common feature of alpha-1 antitrypsin deficiency.
(Choice C) Compensatory hyperinflation refers to the expansion of normal lung parenchyma
that occurs when adjacent lung segments or lobes collapse or are surgically removed.
(Choice E) Subpleural blebs can develop in severe emphysema but typically occur in the
apices, rather than the bases, of the lung. Their rupture is a cause of spontaneous
pneumothorax.
Idiopathic pulmonary fibrosis
This patient with progressive dyspnea, dry cough, digital clubbing, and crackles on examination
has signs of an interstitial lung disease. The histologic findings of patchy lymphoplasmacytic
infiltrates, focal fibroblastic proliferation, areas of dense fibrosis and honeycombing, and
hyperplasia of type 2 pneumocytes are highly suggestive of idiopathic pulmonary
fibrosis (IPF).
IPF is a chronic, progressive fibrotic lung disease thought to be due to recurrent episodes of
lung injury and disordered healing. Persistent inflammation likely triggers excessive activity of
growth factors normally involved in wound healing, including transforming growth factor-beta
(TGF-β), platelet-derived growth factor (PDGF), fibroblastic growth factor (FGF), and vascular
endothelial growth factor (VEGF). This leads to increased fibroblast activity, myofibroblast
formation, and collagen production, which contribute to pulmonary fibrosis. Pirfenidone is an
antifibrotic agent that inhibits TGF-β; another treatment option is nintedanib, a tyrosine kinase
inhibitor that inhibits PDGF, FGF, and VEGF. Although neither drug is curative, these therapies

Telegram Group Invite Link: https://t.me/joinchat/zyFOIg0j5goxY2E1


Telegram Group Invite Link: https://t.me/USMLEWorldStep1

have been shown to slow progressive fibrosis in patients with IPF.

Telegram Group Invite Link: https://t.me/joinchat/zyFOIg0j5goxY2E1


https://t.me/joinchat/zyFOIg0j5goxY2E1

Telegram Group Invite Link: https://t.me/USMLEWorldStep1

Telegram Group Invite Link: https://t.me/joinchat/zyFOIg0j5goxY2E1


Telegram Group Invite Link: https://t.me/USMLEWorldStep1

(Choice A) Alpha-1 antitrypsin infusion is used to treat patients with alpha-1 antitrypsin
deficiency, which causes panacinar emphysema and typically presents in young patients with

Telegram Group Invite Link: https://t.me/joinchat/zyFOIg0j5goxY2E1


https://t.me/joinchat/zyFOIg0j5goxY2E1

Telegram Group Invite Link: https://t.me/USMLEWorldStep1

dyspnea and cough. Histologic findings in emphysema include large alveoli with thin septa.

(Choice B) Endothelin-1 inhibitors (eg, ambrisentan, bosentan) are used to treat idiopathic
pulmonary hypertension. Histologic findings of pulmonary hypertension include pulmonary

Telegram Group Invite Link: https://t.me/joinchat/zyFOIg0j5goxY2E1


Telegram Group Invite Link: https://t.me/USMLEWorldStep1

vascular media and intima hypertrophy and intimal fibrosis.

(Choice C) Monoclonal antibodies directed at interleukin-5 (eg, reslizumab) inhibit eosinophil


recruitment and proliferation, and are used in patients with severe eosinophilic
asthma. Histologic findings in asthma include goblet cell hyperplasia, bronchial smooth muscle

Telegram Group Invite Link: https://t.me/joinchat/zyFOIg0j5goxY2E1


https://t.me/joinchat/zyFOIg0j5goxY2E1

Telegram Group Invite Link: https://t.me/USMLEWorldStep1

hypertrophy, edema, eosinophilia, and Charcot-Leyden crystals.

(Choice D) Phosphodiesterase-4 inhibitors (ie, roflumilast) block the degradation of cyclic AMP,
leading to reduced airway inflammation and smooth muscle relaxation in patients with chronic
obstructive pulmonary disease. Histologic findings include both airspace enlargement with thin
septa (emphysema) and goblet cell hyperplasia with excess mucus production (chronic
bronchitis).

Telegram Group Invite Link: https://t.me/joinchat/zyFOIg0j5goxY2E1


Telegram Group Invite Link: https://t.me/USMLEWorldStep1

alveoli

Ninety-five percent of the alveolar surface is covered by flat type I pneumocytes. The remaining
5% is covered by interspersed cuboidal type II pneumocytes that make up more than half of
the total number of alveolar cells. Type II pneumocytes are the source of pulmonary
surfactant and have the ability to proliferate in response to injury. They also act as progenitor
cells for type I pneumocytes, which cannot regenerate on their own (Choice E).
Telegram Group Invite Link: https://t.me/joinchat/zyFOIg0j5goxY2E1
https://t.me/joinchat/zyFOIg0j5goxY2E1

Telegram Group Invite Link: https://t.me/USMLEWorldStep1

(Choice A) Alveolar macrophages are a self-maintaining population derived from fetal


monocytes during lung development. They are the principal cells involved in clearing inhaled
foreign particles from the terminal respiratory units.
(Choice B) The pseudostratified, ciliated, columnar epithelium of the tracheobronchial tree
gradually tapers to become a simple ciliated cuboidal epithelium at the level of the terminal
bronchioles. Ciliated cells are absent from the lining of alveolar ducts and alveolar sacs.
(Choice C) Club cells are nonciliated, secretory cells found predominantly in the terminal
portions of the bronchioles. These cells can act as a regenerative source of ciliated cells in the
bronchioles.
(Choice D) Goblet cells are found in the respiratory epithelium of the bronchi and larger
bronchioles; they are absent from the alveolar mucosa. Goblet cells are considered terminally-
differentiated, mucin-producing cells.
pneumothorax

This patient with shortness of breath, hypotension, unilaterally decreased breath sounds,
and tracheal deviation shortly after subclavian catheter placement likely has a tension
pneumothorax. Because of the close proximity of the subclavian vein to the apex of the lung,
there is risk of lung puncture during the procedure.
Lung puncture allows air to pass into the pleural space. If the puncture is small, the patient may
remain asymptomatic. With a large puncture, air rushes into the pleural space, leading
to pressure equalization and loss of intrapleural negative pressure, which causes shortness
of breath due to inability to expand the affected lung. Intrapleural and lung pressures usually
remain nearly equivalent (ie, simple pneumothorax), and symptoms are limited to chest pain
and respiratory difficulty.

Telegram Group Invite Link: https://t.me/joinchat/zyFOIg0j5goxY2E1


Telegram Group Invite Link: https://t.me/USMLEWorldStep1

Sometimes lung puncture results in formation of a one-way tissue valve that opens during
inspiration to allow air into the pleural space and closes during expiration to trap that air,
causing intrapleural pressure to progressively increase with each breath (ie, tension
pathophysiology). The increasing pressure leads to contralateral mediastinal shifting (eg,
tracheal deviation) and vena cava collapse, resulting in decreased venous return to the
heart. Cardiac output is reduced, leading to hypotension and tachycardia (ie, obstructive
shock). Treatment requires emergency decompression of the pleural space with needle

Telegram Group Invite Link: https://t.me/joinchat/zyFOIg0j5goxY2E1


https://t.me/joinchat/zyFOIg0j5goxY2E1

Telegram Group Invite Link: https://t.me/USMLEWorldStep1

insertion or chest-tube placement to prevent cardiac arrest.

Telegram Group Invite Link: https://t.me/joinchat/zyFOIg0j5goxY2E1


Telegram Group Invite Link: https://t.me/USMLEWorldStep1

Telegram Group Invite Link: https://t.me/joinchat/zyFOIg0j5goxY2E1


https://t.me/joinchat/zyFOIg0j5goxY2E1

Telegram Group Invite Link: https://t.me/USMLEWorldStep1

Telegram Group Invite Link: https://t.me/joinchat/zyFOIg0j5goxY2E1


Telegram Group Invite Link: https://t.me/USMLEWorldStep1

(Choices A and B) Decreased intravascular volume is the cause of hypovolemic shock (eg,
due to massive hemorrhage), and decreased systemic vascular resistance is the primary
disturbance in distributive shock (eg, due to sepsis). The jugular veins are flat with these
conditions, and unilaterally decreased breath sounds and tracheal deviation are not expected.

Telegram Group Invite Link: https://t.me/joinchat/zyFOIg0j5goxY2E1


https://t.me/joinchat/zyFOIg0j5goxY2E1

Telegram Group Invite Link: https://t.me/USMLEWorldStep1

(Choice D) An impaired baroreceptor reflex can cause or worsen hypotension due to failure of
reflex-mediated vasoconstriction and increased heart rate. This patient's tachycardia suggests
that the baroreceptor reflex is intact.
(Choice E) Increased intrapericardial pressure occurs in cardiac tamponade. The increased
pressure impairs diastolic filling of the right ventricle, leading to obstructive shock. Although
jugular venous distension is expected, cardiac tamponade does not explain this patient's
unilaterally decreased breath sounds and tracheal deviation.

Telegram Group Invite Link: https://t.me/joinchat/zyFOIg0j5goxY2E1


Telegram Group Invite Link: https://t.me/USMLEWorldStep1

hypoxemia

This patient has diffuse pulmonary edema (inspiratory wet crackles, bilateral patchy infiltrates)
that can be cardiogenic (eg, decompensated left ventricular failure) or noncardiogenic (eg, acute
respiratory distress syndrome). With pulmonary edema, the alveoli become flooded and
unable to fill with gas (ventilation: V ≈ 0). However, they are still being perfused (Q), resulting in
many alveoli with ventilation-perfusion (V/Q) ratios approaching zero.
Because blood passing through the lungs returns to the left atrium without being oxygenated,
this situation is analogous to a true right-to-left anatomic shunt (eg, atrial septal defect with flow
from the right to the left atrium) and is termed intrapulmonary shunt effect. Increasing the
fraction of inspired oxygen (FiO2) to 100% does not improve oxygenation because the shunted
blood never "sees" the alveolar gas. Therefore, this patient's low arterial partial pressure of
oxygen (PaO2) has not corrected with 100% supplemental oxygen.
Because pure oxygen alone cannot improve hypoxemia, treatment depends on reducing the
shunt effect. This can be done by pressurizing the inspired gas in a closed circuit (eg,
mechanical ventilator). Positive end-expiratory pressure (PEEP) reduces the shunt by

Telegram Group Invite Link: https://t.me/joinchat/zyFOIg0j5goxY2E1


https://t.me/joinchat/zyFOIg0j5goxY2E1

Telegram Group Invite Link: https://t.me/USMLEWorldStep1

propping the collapsed alveoli open, displacing extracellular lung fluid and exposing more
alveolar surface area for gas exchange/oxygen uptake.

(Choice A) Anatomical dead space refers to the volume of air in the conducting airways (eg,
trachea, bronchi) that never reaches alveoli. It is increased, not decreased, in an intubated
patient due to the extra volume of the ventilator tubing. At any given tidal volume, increased
anatomic dead space causes less air to reach alveoli with each breath, contributing to
hypoventilation and worsening hypoxemia.
(Choice C) When fluid enters the alveolar airspace (ie, pulmonary edema), the surface tension
increases (due to dilution of surfactant) causing the alveolar walls to cling together and collapse
(ie, atelectasis), which decreases lung compliance. PEEP props open ("recruits") the partially
flooded alveoli, improving lung compliance.
(Choices D and E) PEEP raises intrathoracic pressure, which compresses pulmonary
capillaries and raises pulmonary vascular resistance. This increases pressure within the right
heart chambers and can increase right-to-left shunting through an existing intracardiac shunt

Telegram Group Invite Link: https://t.me/joinchat/zyFOIg0j5goxY2E1


Telegram Group Invite Link: https://t.me/USMLEWorldStep1

(eg, atrial or ventricular septal defect), worsening hypoxemia. Therefore, PEEP is generally
avoided in patients with intracardiac shunts.

Dead
space

Minute ventilation is the total volume of new air that enters the respiratory pathways per
minute. It can be calculated by multiplying the average tidal volume by the number of breaths
per minute (minute ventilation [L/min] = tidal volume [L] x breaths/min).
Alveolar ventilation refers only to the volume of new air reaching the gas exchange
areas (ie, alveoli and respiratory bronchioles) per minute. It can be calculated by subtracting
dead space volume (which does not participate in gas exchange) from the tidal volume:
Alveolar ventilation (L/min) = (tidal volume − dead space volume) × breaths/min

Telegram Group Invite Link: https://t.me/joinchat/zyFOIg0j5goxY2E1


https://t.me/joinchat/zyFOIg0j5goxY2E1

USMLE e e r m h nne s
Me est i e s: https://t.me/USMLEMe est

K p n ssr m i e s: https://t.me/USMLEK p n

USMLE i e s Step : https://t.me/USMLE Step

e ker i e s: https://t.me/USMLE e ker

r s e n i e s Step : https://t.me/USMLE r sAn e n

r s e n i e s Step K: https://t.me/USMLE r sAn e n Step K

Ph se i e s: https://t.me/USMLEPh se

Pi ri e i e s: https://t.me/USMLEPi ri e

P th m i e s: https://t.me/USMLEP th m

nA i s i h it : https://t.me/USMLE nA i s

Am ss r p: https://t.me/USMLEAm ss

Le t ri nk: https://t.me/USMLELe t ri

En p int P s: https://t.me/USMLEEn p int

USMLE MEs: https://t.me/USMLE ME

L E Me E : https://t.me/USMLE n ineMe E

USMLE r Step : https://t.me/USMLE r Step

USMLE r Step K: https://t.me/USMLE r Step K

USMLE r Step nk: https://t.me/USMLE r Step

USMLE Anki: https://t.me/USMLEAnki

KissPrep: https://t.me/USMLEKissPrep
https://t.me/joinchat/zyFOIg0j5goxY2E1

Telegram Group Invite Link: https://t.me/USMLEWorldStep1

The total volume of dead space in the lungs is known as physiologic dead space. It consists of
both the anatomic dead space of the conducting airways (ie, nose, trachea, bronchi,
bronchioles; normally ~150 mL) and alveolar dead space due to well-ventilated but poorly
perfused alveoli. Because it is difficult to directly measure physiologic dead space, it is often
estimated in mechanically ventilated patients by comparing arterial (a) and expiratory (E)
pCO2 levels:
Physiologic dead space = tidal volume × ([PaCO2 − PECO2]/PaCO2)
(Choices B and C) Functional residual capacity is the volume of air remaining in the lungs after
a normal exhalation. It is the sum of residual volume (the volume of air remaining in the lungs
after maximal exhalation) and expiratory reserve volume (the volume difference between normal
end-expiration and maximal end-expiration).

(Choice D) Respiratory rate is the number of breaths taken per minute and is a factor in
calculating both minute ventilation and alveolar ventilation.
(Choice E) Tidal volume is the volume of new air that enters the lungs with each normal,
unforced breath. It is a factor in calculating both minute ventilation and alveolar ventilation.
Prone position

Telegram Group Invite Link: https://t.me/joinchat/zyFOIg0j5goxY2E1


Telegram Group Invite Link: https://t.me/USMLEWorldStep1

Prone positioning (lying face down) is sometimes used to improve arterial oxygenation in
patients with severe acute respiratory distress syndrome. It likely does so by reducing
atelectasis of posterior alveoli to reduce intrapulmonary shunting and improve ventilation-
perfusion matching throughout the lungs.
Because the heart occupies anterior space in the chest cavity, the majority of lung mass is
located posteriorly. When a patient is in the supine position (lying face up), the weight of the
heart and anterior lung segments compresses the posterior lung segments and limits ventilation
of posterior alveoli. In addition, abdominal organ and tissue mass weighs considerably on the
posterior portion of the diaphragm, displacing it cranially and further contributing to compression
of the posterior basal lung segments. The anterior alveoli, subjected to minimal compression
and ample ventilation in the supine position, are often hyperinflated, increasing the risk of
barotrauma in those regions.

Telegram Group Invite Link: https://t.me/joinchat/zyFOIg0j5goxY2E1


https://t.me/joinchat/zyFOIg0j5goxY2E1

Telegram Group Invite Link: https://t.me/USMLEWorldStep1

Prone positioning causes the heart and anterior lung mass to rest on the anterior chest
wall, alleviating compression of the posterior lung segments. In addition, the freely
hanging abdomen removes pressure on the diaphragm, allowing the diaphragm to move
caudally and further open up collapsed posterior basal alveoli. With a higher percentage of total
alveoli opened, ventilation is more evenly distributed, improving arterial oxygenation and
reducing alveolar hyperinflation and hyperdistension (which may decrease the risk of
barotrauma) (Choice A).
(Choices B and D) Prone positioning may increase venous return (ie, right ventricular preload)
and, consequently, cardiac output by positioning the heart slightly below the large reservoir of
venous blood in the splanchnic circulation (ie, additional stores of venous blood are mobilized to
the right atrium).
(Choice C) By reducing posterior lung compression and decreasing atelectasis, prone
positioning increases functional residual capacity (ie, the volume of air in the lungs at the end of
tidal expiration).
Pneumothorax

This young male patient with sudden-onset unilateral chest pain, dyspnea, and absent breath
sounds on examination likely has primary spontaneous pneumothorax (PSP). PSP is
nontraumatic and is found in patients without preexisting pulmonary disease (eg, cystic
fibrosis). It occurs when a large change in the alveolar or intrapleural pressure results in a
break in the visceral pleura and subsequent trapping of air between the parietal and visceral
spaces. The superficial alveoli in the apices experience greater pressure changes (due to the
weight of the lungs pulling down on the apical tissue), predisposing them to the formation
of subpleural blebs. The blebs then can spontaneously rupture through the visceral pleura,
frequently while the patient is at rest.

Telegram Group Invite Link: https://t.me/joinchat/zyFOIg0j5goxY2E1


Telegram Group Invite Link: https://t.me/USMLEWorldStep1

Tall, thin males around the age of 20 are most commonly affected. Although the most important
risk factor is smoking, taller individuals also appear to be at higher risk due to more negative
intrapleural pressure in the lung apices.
(Choice B) Centriacinar emphysema occurs as the larger proximal bronchioles are destroyed,
usually as a result of exposure to tobacco smoke in patients with chronic obstructive pulmonary
disease. It predominantly affects the upper lung lobe and initially spares the superficial alveoli.
(Choice C) Compensatory hyperinflation results when normal lung parenchyma expands in
response to loss of adjacent lung volume (eg, segmental/lobar collapse or surgical
removal). PSP results in collapse of adjacent lung tissue due to extrinsic compression by
intrapleural air and does not result in parenchymal expansion.
(Choice D) Obstructive hyperinflation occurs when a lung segment expands due to a partial
obstruction (eg, bronchogenic carcinoma) of the airway supplying it. On inspiration, the airway
expands, which allows the movement of air into the segment; however, as the airway closes on
expiration, air is trapped behind the obstruction. In an otherwise healthy young patient, it is
unlikely to be the cause of PSP.
(Choice E) Panacinar emphysema occurs more commonly in the lower zones and anterior
margins of the lungs. It destroys the entire acinus uniformly and is associated with alpha-1
antitrypsin deficiency. Although pneumothorax can occur in these patients, it is usually
preceded by symptoms of emphysema.

Telegram Group Invite Link: https://t.me/joinchat/zyFOIg0j5goxY2E1


https://t.me/joinchat/zyFOIg0j5goxY2E1

Telegram Group Invite Link: https://t.me/USMLEWorldStep1

Diffusion
impairment

Telegram Group Invite Link: https://t.me/joinchat/zyFOIg0j5goxY2E1


Telegram Group Invite Link: https://t.me/USMLEWorldStep1

Telegram Group Invite Link: https://t.me/joinchat/zyFOIg0j5goxY2E1


https://t.me/joinchat/zyFOIg0j5goxY2E1

Telegram Group Invite Link: https://t.me/USMLEWorldStep1

Under normal conditions, the PO2 of inspired ambient air is approximately 160 mm Hg. This
value decreases to approximately 150 mm Hg in the trachea due to the partial pressure of water
vapor. Normal alveolar PO2 is 104 mm Hg, which lies between the tracheal (150 mm Hg) and
systemic venous blood (40 mm Hg) PO2 concentrations. Likewise, normal alveolar PCO2 is 40
mm Hg, also between its respective tracheal (0 mm Hg) and systemic venous blood
concentrations (45 mm Hg).
Transfer of O2 and CO2 between the blood and the alveoli depends on both perfusion and
diffusion. Under normal resting conditions, diffusion of O2 and CO2 is a quick process, with
blood needing to traverse only one-third of the total pulmonary capillary length to completely
equilibrate. In this setting, gas exchange is perfusion-limited (ie, diffusion is fast enough that the
total gas exchange depends only on the amount of blood supplied to the alveoli).
Diffusion-limited gas exchange takes place when diffusion across the alveolar membrane is
relatively slow enough to become the major limitation to gas exchange. This can occur with
exercise (due to extremely high perfusion volumes that the diffusion rate cannot keep up with)
or with pathologic conditions that disrupt the alveolar membrane (eg, pulmonary fibrosis,
emphysema) and slow diffusion. CO2 diffuses 20 times faster than O2, even when the alveolar
membrane is somewhat disrupted; therefore, alveolar and capillary CO2 concentrations typically
fully equilibrate while alveolar and capillary O2 concentrations do not.
In this experiment, there is a large PO2 gradient between the alveoli (104 mm Hg) and the
pulmonary venous blood (70 mm Hg) but pulmonary venous PCO2 is normal (40 mm Hg),

Telegram Group Invite Link: https://t.me/joinchat/zyFOIg0j5goxY2E1


Telegram Group Invite Link: https://t.me/USMLEWorldStep1

indicating incomplete gas exchange most likely due to diffusion impairment.

Telegram Group Invite Link: https://t.me/joinchat/zyFOIg0j5goxY2E1


https://t.me/joinchat/zyFOIg0j5goxY2E1

Telegram Group Invite Link: https://t.me/USMLEWorldStep1

(Choice A) Bronchopulmonary shunting describes the normal small drop in PO2 and small
increase in PCO2 that occurs due to the pulmonary veins receiving deoxygenated blood from the

Telegram Group Invite Link: https://t.me/joinchat/zyFOIg0j5goxY2E1


Telegram Group Invite Link: https://t.me/USMLEWorldStep1

bronchial circulation. However, this normally only results in about a 4 mm Hg drop in PO2 within
the pulmonary veins.

Telegram Group Invite Link: https://t.me/joinchat/zyFOIg0j5goxY2E1


https://t.me/joinchat/zyFOIg0j5goxY2E1

Telegram Group Invite Link: https://t.me/USMLEWorldStep1

Telegram Group Invite Link: https://t.me/joinchat/zyFOIg0j5goxY2E1


Telegram Group Invite Link: https://t.me/USMLEWorldStep1

(Choice C) In normal alveolar ventilation and perfusion, the alveolar and pulmonary capillary
PO2 rapidly reach equilibrium at 104 mm Hg.
(Choice D) Intrapulmonary shunting can occur when the alveoli fill with fluid (eg, pneumonia,
pulmonary edema) or collapse (eg, atelectasis) and alveolar ventilation is essentially zero. This
patient's alveolar PO2 of 104 mm Hg is not consistent with shunting (alveolar PO2 equilibrates

Telegram Group Invite Link: https://t.me/joinchat/zyFOIg0j5goxY2E1


https://t.me/joinchat/zyFOIg0j5goxY2E1

Telegram Group Invite Link: https://t.me/USMLEWorldStep1

with venous blood at 40 mm Hg when ventilation is zero).

Telegram Group Invite Link: https://t.me/joinchat/zyFOIg0j5goxY2E1


Telegram Group Invite Link: https://t.me/USMLEWorldStep1

Telegram Group Invite Link: https://t.me/joinchat/zyFOIg0j5goxY2E1


https://t.me/joinchat/zyFOIg0j5goxY2E1

Telegram Group Invite Link: https://t.me/USMLEWorldStep1

(Choice E) Poor alveolar ventilation (ie, hypoventilation) does not affect gas exchange between
the alveoli and the blood; therefore, no O2 gradient is created.

Telegram Group Invite Link: https://t.me/joinchat/zyFOIg0j5goxY2E1


Telegram Group Invite Link: https://t.me/USMLEWorldStep1

Telegram Group Invite Link: https://t.me/joinchat/zyFOIg0j5goxY2E1


https://t.me/joinchat/zyFOIg0j5goxY2E1

Telegram Group Invite Link: https://t.me/USMLEWorldStep1

Pulmonary embolism

Sudden-onset dyspnea in the setting of recent calf swelling is strongly suggestive


of pulmonary embolism (PE). Dyspnea is the most common symptom of PE and risk factors
include obesity, smoking, and prolonged immobilization. The embolus typically arises from a
deep venous thrombus in the lower extremities (causes calf swelling) and lodges itself within the
arterial blood supply of the lungs.

Telegram Group Invite Link: https://t.me/joinchat/zyFOIg0j5goxY2E1


Telegram Group Invite Link: https://t.me/USMLEWorldStep1

PE occludes or significantly reduces blood flow to a portion of the pulmonary parenchyma,


resulting in increased dead space ventilation and redistribution of pulmonary blood flow away
from the affected segments. This leads to increased ventilation/perfusion (V/Q) mismatch in
the remainder of the lungs with consequent hypoxemia.
Stimulation of central respiratory drive occurs in response to dyspnea and inflammatory
mediators released by ischemic pulmonary tissue. Hyperventilation does not significantly
improve arterial oxygenation because hemoglobin is nearly fully saturated with O2 in areas of
normal V/Q ratio, allowing little capacity for high V/Q regions to increase blood O2 content much
further. In contrast, CO2 removal is more directly dependent on ventilation, and high V/Q
regions have large capacity to exhale additional CO2. Therefore, the hyperventilation leads to
hypocapnia (low PaCO2) and respiratory alkalosis. The serum HCO3- is near normal in the
acute setting, but metabolic compensation with renal bicarbonate loss takes place over the next
72 hours.

Telegram Group Invite Link: https://t.me/joinchat/zyFOIg0j5goxY2E1


https://t.me/joinchat/zyFOIg0j5goxY2E1

Telegram Group Invite Link: https://t.me/USMLEWorldStep1

(Choice A) Low blood pH of 7.30 with low serum HCO3- indicates metabolic acidosis, which is
compensated by hyperventilation leading to low PaCO2.
(Choice B) A low pH of 7.32 with high PaCO2 reflects respiratory acidosis. The high
HCO3- indicates renal compensation by bicarbonate retention. Although PE may eventually lead
to respiratory collapse with an acute increase in PaCO2, a renal compensatory response would
not be expected so abruptly.
(Choice C) These arterial blood gas values are considered normal, reflecting an absence of
acid-base disturbance.

Telegram Group Invite Link: https://t.me/joinchat/zyFOIg0j5goxY2E1


Telegram Group Invite Link: https://t.me/USMLEWorldStep1

(Choice E) A high pH of 7.47 with low PaCO2 reflects respiratory alkalosis. The low
HCO3- indicates renal compensation, which is expected with chronic respiratory alkalosis but
should not be present with acute PE. Normal PaO2 can occasionally be present in acute PE but
is less likely.
(Choice F) A high pH of 7.48 with high serum HCO3- reflects metabolic alkalosis, which is
compensated by hypoventilation leading to high PaCO2.

Telegram Group Invite Link: https://t.me/joinchat/zyFOIg0j5goxY2E1


https://t.me/joinchat/zyFOIg0j5goxY2E1

Telegram Group Invite Link: https://t.me/USMLEWorldStep1

Acid base
disorder

The assessment of an arterial blood gas follows a stepwise process:


1. Evaluate the pH (normal: 7.4): A value <7.35 signifies acidosis whereas a value >7.45
indicates alkalosis. This patient has an arterial blood pH of 7.22, which is consistent
with acidosis.
2. Identify the primary process: Differentiate respiratory from metabolic processes by
determining the PaCO2 (normal: 40) and HCO3− (normal: 24).

Telegram Group Invite Link: https://t.me/joinchat/zyFOIg0j5goxY2E1


https://t.me/joinchat/zyFOIg0j5goxY2E1

USMLE e e r m h nne s
Me est i e s: https://t.me/USMLEMe est

K p n ssr m i e s: https://t.me/USMLEK p n

USMLE i e s Step : https://t.me/USMLE Step

e ker i e s: https://t.me/USMLE e ker

r s e n i e s Step : https://t.me/USMLE r sAn e n

r s e n i e s Step K: https://t.me/USMLE r sAn e n Step K

Ph se i e s: https://t.me/USMLEPh se

Pi ri e i e s: https://t.me/USMLEPi ri e

P th m i e s: https://t.me/USMLEP th m

nA i s i h it : https://t.me/USMLE nA i s

Am ss r p: https://t.me/USMLEAm ss

Le t ri nk: https://t.me/USMLELe t ri

En p int P s: https://t.me/USMLEEn p int

USMLE MEs: https://t.me/USMLE ME

L E Me E : https://t.me/USMLE n ineMe E

USMLE r Step : https://t.me/USMLE r Step

USMLE r Step K: https://t.me/USMLE r Step K

USMLE r Step nk: https://t.me/USMLE r Step

USMLE Anki: https://t.me/USMLEAnki

KissPrep: https://t.me/USMLEKissPrep
Telegram Group Invite Link: https://t.me/USMLEWorldStep1

• Respiratory acidosis occurs due to the retention of CO2 in the lungs and presents
with a low pH and a high PaCO2.
• Metabolic acidosis develops due to depletion of HCO3− and is characterized by a
low pH and a low HCO3−.
This patient has a pH of 7.22 (low) and a PaCO2 of 60 mm Hg (high), which is consistent
with respiratory acidosis.
3. Identify compensation: Because increased PaCO2 leads to an increased hydrogen ion
concentration in the body, the kidneys begin to compensate by increasing resorption of
HCO3− to help buffer the excess acid. The renal compensation begins after several
hours and takes several days to complete. After approximately 72 hours, the expected
compensatory increase in HCO3− is approximately 4 mEq/L for every 10 mm Hg elevation
in PaCO2.
This patient has a HCO3− of 24 mEq/L, which is normal and indicates respiratory acidosis
without compensation. Causes of respiratory acidosis without compensation include any
disorder that causes acute hypoventilation (eg, opioid overdose with depression of respiratory
drive).

(Choices A and E) Metabolic acidosis is indicated by low pH and low serum HCO3−. The
respiratory compensation involves hyperventilation to breathe off CO2 and help increase
pH. Respiratory compensation for metabolic disturbances begins right away and is usually
maximized within hours; therefore, full (or nearly full) compensation is expected in both the
acute and chronic setting unless the patient's breathing is compromised.
(Choice B) Metabolic alkalosis is indicated by high pH and high HCO3−. The respiratory
compensation involves hypoventilation to retain CO2 and decrease pH.
(Choice C) This patient's low pH and high PaCO2 are consistent with respiratory acidosis, but
the normal HCO3− indicates that renal compensation has not yet occurred.
(Choice D) Respiratory alkalosis is indicated by high pH and low PaCO2. The metabolic
compensation involves increased HCO3− excretion by the kidneys to help decrease pH and
requires approximately 72 hours to complete.

Telegram Group Invite Link: https://t.me/joinchat/zyFOIg0j5goxY2E1


https://t.me/joinchat/zyFOIg0j5goxY2E1

Telegram Group Invite Link: https://t.me/USMLEWorldStep1

hypercarpia

This patient most likely had a panic attack while trapped in the elevator. Panic attacks are
typically accompanied by hyperventilation with a resulting decrease in arterial partial
pressure of CO2 (PaCO2). This hypocapnia can cause decreased cerebral perfusion with
consequent neurologic sequelae, including generalized weakness, blurred vision, presyncope
(ie, dizziness, lightheadedness), and syncope.
Cerebral blood flow (CBF) remains relatively constant over a wide range of perfusion
pressures (cerebrovascular autoregulation) and is mainly influenced by arterial blood gas levels,
particularly changes in PaCO2. Hypercapnia triggers an increase in CBF (to aid in removal of
toxins), and hypocapnia triggers a decrease in CBF. In fact, because hypocapnia decreases
CBF, mechanically ventilated patients with cerebral edema are often hyperventilated to
decrease intracranial pressure and help prevent brain herniation.
CBF is less sensitive to the arterial partial pressure of oxygen (PaO2). PaO2 has little influence
on CBF until the level drops below 50 mm Hg, at which point a rapid increase in CBF is
triggered.
(Choices B and D) The total blood O2 content is predominantly made up of hemoglobin-bound
O2 (determined by the blood hemoglobin level and PaO2) plus a small contribution of dissolved
O2. Hyperventilation increases the alveolar partial pressure of O2 and should somewhat
increase (rather than decrease) both PaO2 and total blood O2 content.
(Choice C) Hyperventilation produces respiratory alkalosis, marked by an increase in arterial
pH.
(Choice E) Disturbances that increase the alveolar-arterial O2 gradient include V/Q mismatch
(eg, pulmonary embolism), diffusion limitation, and right-to-left shunting (eg, Eisenmenger
syndrome). Changes in ventilation rate (ie, hypo- or hyperventilation) have no effect on the
alveolar-arterial O2 gradient.

Telegram Group Invite Link: https://t.me/joinchat/zyFOIg0j5goxY2E1


Telegram Group Invite Link: https://t.me/USMLEWorldStep1

(Choice F) Increased arterial lactic acid content is a sign of tissue hypoxia (eg, due to sepsis or
ischemia), which is unlikely in the setting of hyperventilation from a panic attack.
SIADH

This patient's hyponatremia is most likely due to syndrome of inappropriate antidiuretic


hormone (SIADH), which is one of several paraneoplastic syndromes that may occur in
patients with small cell lung cancer (SCLC). Presenting symptoms of SIADH are often vague
(eg, lethargy), but untreated SIADH can lead to significant hyponatremia causing seizures or
coma. Physical examination should reveal euvolemia, and laboratory results should
demonstrate decreased serum osmolality consistent with true (hypotonic) hyponatremia. An
essential characteristic of SIADH is urine osmolality >100 mOsm/kg H2O, which reflects
inability of the kidneys to produce adequately dilute urine to remove excess free water from the
body (due to inappropriately high levels of ADH).
SIADH may be caused by excessive release of ADH from the posterior pituitary (eg, central
nervous system disorder, medication adverse effect) or ectopic secretion of ADH. SCLC, a
neuroendocrine tumor that typically presents as a hilar mass in patients with a
significant smoking history, is the most common cause of ectopic secretion of ADH.
(Choice A) Carcinoids are rare neuroendocrine tumors that typically occur in the digestive tract
or lungs. They may lead to carcinoid syndrome (eg, flushing, diarrhea, bronchoconstriction),
which is a paraneoplastic syndrome caused by secretion of serotonin, histamine, and
kinins. Carcinoids may rarely cause SIADH, but SCLC is more likely in this patient with a hilar
mass and significant smoking history.
(Choices B, D, and F) Lung adenocarcinoma may cause paraneoplastic hypertrophic
osteoarthropathy, and squamous cell carcinoma can lead to paraneoplastic hypercalcemia (due
to tumor secretion of parathyroid hormone-related protein). Mesothelioma is a rare malignancy

Telegram Group Invite Link: https://t.me/joinchat/zyFOIg0j5goxY2E1


https://t.me/joinchat/zyFOIg0j5goxY2E1

Telegram Group Invite Link: https://t.me/USMLEWorldStep1

derived from the mesothelial lining of the thoracic cavity and has been associated with a variety
of paraneoplastic syndromes (eg, peripheral neuropathy, migratory thrombophlebitis). SIADH
due to any of these malignancies would be unusual.
(Choice C) Paraneoplastic syndromes associated with lymphoma include hypercalcemia due
to overproduction of 1,25-dihydroxyvitamin D and cerebellar degeneration. SIADH would be
unusual.
Pulmonary
embolism

The lung is supplied by dual circulation from both the pulmonary arteries and bronchial
arteries. This collateral circulation can help protect against lung infarction as a complication
of pulmonary embolism (PE).

Telegram Group Invite Link: https://t.me/joinchat/zyFOIg0j5goxY2E1


Telegram Group Invite Link: https://t.me/USMLEWorldStep1

The pulmonary arteries are the major suppliers of blood to the lungs, providing deoxygenated
blood for gas exchange; the bronchial arteries supply nutrients, remove waste from the bronchi,
and provide collateral blood flow to the remainder of the lung parenchyma. When a clot
occludes the pulmonary system, the bronchial system continues to supply nutrients to
pulmonary tissue and can even backfill the pulmonary capillaries to partially maintain gas
exchange in the area of the thromboembolism.
Distal PEs in small arteries (≤3 mm) are more likely to cause infarction as they may occlude
areas distal to the pulmonary-bronchial anastomoses. When a pulmonary infarction does occur,
it is typically hemorrhagic (red) rather than ischemic (white) due to the relatively low density of
lung tissue and the dual blood supply.
(Choice B) Alveolar inflammation triggered by PE can eventually lead to decreased surfactant
and some degree of atelectasis. This process can take up to 2 days to develop, and the
resulting intrapulmonary shunting makes a small contribution to the hypoxemia that occurs with
PE.
(Choice C) Recombinant tissue plasminogen activator (TPA) may be used to treat PE in
patients who are hemodynamically unstable. However, endothelial-derived TPA is limited
primarily to the bronchial circulation, and spontaneous recanalization of the pulmonary artery is
a slow process.
(Choice D) PE leads to increased pulmonary arterial pressure; however, the pulmonary venous
pressure is unchanged or decreased as blood flow through the proximal pulmonary circulation is
obstructed. Patients with an underlying elevation in pulmonary venous pressure (eg,
decompensated heart failure) may be more likely to experience lung infarction with PE due to
the high pulmonary venous pressure impairing collateral flow from the bronchial circulation.
(Choice E) Intrapulmonary shunting occurs when an area of the lung is adequately perfused
but poorly ventilated. PE causes intrapulmonary shunting due to redistribution of blood away
from segments directly affected by the clot; the remaining accessible alveoli are unable to fully
oxygenate all the blood passing through the pulmonary circulation, resulting in hypoxemia. In
contrast, areas distal to the clot receive adequate ventilation but poor perfusion (ie, dead space
ventilation).

Telegram Group Invite Link: https://t.me/joinchat/zyFOIg0j5goxY2E1


https://t.me/joinchat/zyFOIg0j5goxY2E1

Telegram Group Invite Link: https://t.me/USMLEWorldStep1

Telegram Group Invite Link: https://t.me/joinchat/zyFOIg0j5goxY2E1


Telegram Group Invite Link: https://t.me/USMLEWorldStep1

Alveolar
hypoventilation

Telegram Group Invite Link: https://t.me/joinchat/zyFOIg0j5goxY2E1


https://t.me/joinchat/zyFOIg0j5goxY2E1

Telegram Group Invite Link: https://t.me/USMLEWorldStep1

Telegram Group Invite Link: https://t.me/joinchat/zyFOIg0j5goxY2E1


Telegram Group Invite Link: https://t.me/USMLEWorldStep1

The partial pressure of oxygen in the alveoli (PAO2) is normally 104 mm Hg, and due to the high
rate of O2 diffusion across the alveolar-capillary membrane, the O2 level in the alveolar capillary
blood rapidly equilibrates with the PAO2. However, the blood O2 level then drops slightly due to
the addition of deoxygenated blood from the bronchial circulation; therefore, the partial pressure
of oxygen in arterial blood (PaO2) is normally around 100 mm Hg. This discrepancy between
alveolar and arterial O2 concentration is termed the alveolar-arterial (A-a) gradient. In healthy
individuals, a normal A-a gradient is typically between 4 and 15 mm Hg, with older individuals
having higher normal values due to an age-related decline in O2-diffusing capacity.
This patient has low PaO2 and PAO2 with a normal A-a gradient (68 − 60 = 8), indicating that his
low PaO2 is directly due to low PAO2. Possible causes of hypoxemia in the setting of a normal
A-a gradient include alveolar hypoventilation and low partial pressure of inspired oxygen
(PiO2) (ie, high altitude). Common causes of alveolar hypoventilation include suppressed
central respiratory drive (eg, sedative overdose) and diseases that decrease inspiratory capacity

Telegram Group Invite Link: https://t.me/joinchat/zyFOIg0j5goxY2E1


https://t.me/joinchat/zyFOIg0j5goxY2E1

Telegram Group Invite Link: https://t.me/USMLEWorldStep1

(eg, myasthenia gravis, obesity).

Telegram Group Invite Link: https://t.me/joinchat/zyFOIg0j5goxY2E1


Telegram Group Invite Link: https://t.me/USMLEWorldStep1

(Choices B and D) Dead space ventilation is one extreme of ventilation-perfusion mismatch


that occurs when the alveoli are adequately ventilated, but there is no alveolar perfusion (eg,
pulmonary embolism). An intrapulmonary shunt is the other extreme of ventilation-perfusion
mismatch that occurs when blood perfuses alveoli that are not ventilated (eg, pneumonia,

Telegram Group Invite Link: https://t.me/joinchat/zyFOIg0j5goxY2E1


https://t.me/joinchat/zyFOIg0j5goxY2E1

Telegram Group Invite Link: https://t.me/USMLEWorldStep1

pulmonary edema). The A-a gradient is elevated in ventilation-perfusion mismatch.

Telegram Group Invite Link: https://t.me/joinchat/zyFOIg0j5goxY2E1


Telegram Group Invite Link: https://t.me/USMLEWorldStep1

Telegram Group Invite Link: https://t.me/joinchat/zyFOIg0j5goxY2E1


https://t.me/joinchat/zyFOIg0j5goxY2E1

Telegram Group Invite Link: https://t.me/USMLEWorldStep1

(Choice C) Gas diffusion is impaired in diseases that disrupt the alveolar-capillary membrane,
such as alveolar hyaline membrane disease (acutely) or emphysema (chronically). Diffusion
impairment causes an elevated A-a gradient because O2 cannot be effectively transported into

Telegram Group Invite Link: https://t.me/joinchat/zyFOIg0j5goxY2E1


Telegram Group Invite Link: https://t.me/USMLEWorldStep1

the blood.

Telegram Group Invite Link: https://t.me/joinchat/zyFOIg0j5goxY2E1


https://t.me/joinchat/zyFOIg0j5goxY2E1

Telegram Group Invite Link: https://t.me/USMLEWorldStep1

(Choice E) A left-to-right shunt occurs when oxygenated blood from the left side of the heart is
shunted into the right side of the heart (eg, atrial or ventricular septal defect). Left-to-right
shunts do not cause hypoxemia; however, if left untreated, they may progress into a right-to-left
shunt (ie, Eisenmenger syndrome) with hypoxemia, cyanosis, and an elevated A-a gradient.
Pulmonary
hypertension

This patient has pulmonary hypertension, which typically presents with progressive dyspnea
and fatigue, sometimes with associated chest pain and exertional lightheadedness or
syncope. Due to right ventricular enlargement, a holosystolic murmur of functional tricuspid
regurgitation is often present.
Pulmonary hypertension can occur due to a primary change in the pulmonary arteries or it can
be secondary to a separate disease process; secondary causes include left-sided heart failure
(ruled out by pulmonary capillary wedge pressure ≤12 mm Hg), chronic lung hypoxia (eg,
interstitial lung disease), and chronic pulmonary thromboembolic disease. Pulmonary
hypertension due to a primary change in the pulmonary arteries, termed pulmonary arterial
hypertension (PAH), results from proliferative vasculopathy of the pulmonary arteriolar
smooth muscle, which may occur in the setting of hereditary genetic mutation (BMPR2
mutation is most common), connective tissue disease, or HIV infection.

Telegram Group Invite Link: https://t.me/joinchat/zyFOIg0j5goxY2E1


Telegram Group Invite Link: https://t.me/USMLEWorldStep1

Endothelin is a potent vasoconstrictor that also promotes smooth muscle cell proliferation;
it is typically found in high concentrations in patients with PAH and it is an important target of
therapy for the disease. Endothelin receptor antagonists (eg, bosentan, ambrisentan) inhibit
pulmonary arteriolar smooth muscle proliferation and alleviate vasoconstriction to lower
pulmonary arterial pressure and improve dyspnea in patients with PAH.

(Choices A, B, and D) Aldosterone (mineralocorticoid) receptor antagonists (eg,


spironolactone), angiotensin II receptor blockers (eg, losartan), and beta blockers (antagonists
of the beta-1 adrenergic receptors that norepinephrine stimulates, eg, metoprolol) help prevent
cardiac remodeling in patients with left ventricular systolic dysfunction. These drugs can be
used to treat pulmonary hypertension that develops secondary to left-sided heart failure;
however, they are not used for PAH as a direct effect on pulmonary vascular remodeling has
not been established.
(Choice E) In carcinoid syndrome, high circulating levels of serotonin may cause deposition of
fibrous tissue on the right-sided heart valves, typically leading to tricuspid regurgitation and/or
pulmonic stenosis. Agents that block the effects of serotonin (eg, octreotide) can be helpful in
treating carcinoid syndrome; however, carcinoid syndrome would not explain the elevated
pulmonary arterial pressure in this patient. The role of serotonin in PAH has not been well-
defined.

Telegram Group Invite Link: https://t.me/joinchat/zyFOIg0j5goxY2E1


https://t.me/joinchat/zyFOIg0j5goxY2E1

Telegram Group Invite Link: https://t.me/USMLEWorldStep1

Lactic
acidosis

This patient's signs and symptoms (eg, fever, leukocytosis, hypotension, tachycardia) suggest
that he is in septic shock. Decreased bicarbonate level and increased anion gap are indicative
of anion-gap metabolic acidosis secondary to an elevated lactic acid level.
Lactic acidosis may result from overproduction and/or impaired clearance of lactic acid. End-
organ hypoperfusion in septic shock impairs tissue oxygenation and decreases oxidative
phosphorylation, leading to a buildup of NADH and shunting of pyruvate to lactate following
glycolysis. Hepatic hypoperfusion during sepsis also contributes to the accumulation of lactic

Telegram Group Invite Link: https://t.me/joinchat/zyFOIg0j5goxY2E1


Telegram Group Invite Link: https://t.me/USMLEWorldStep1

acid as the liver is the primary site of lactate clearance (via conversion back into glucose).

(Choice B) Impaired hepatic gluconeogenesis occurs with several inborn errors of metabolism
(eg, pyruvate carboxylase deficiency, glucose-6-phosphatase deficiency) and can result in the
buildup of lactic acid. These diseases usually present in infancy with hypoglycemia and lactic
acidosis.
(Choice C) Impaired renal tubular bicarbonate reabsorption is seen in type 2 (proximal) renal
tubular acidosis. Poor bicarbonate reabsorption can occur in a variety of inherited or acquired
conditions, including multiple myeloma and drug toxicity (eg, acetazolamide).
(Choice D) Increased lipolysis and ketogenesis occur in patients with diabetic ketoacidosis
(DKA), which also presents with anion-gap metabolic acidosis. Serum glucose is much higher
in DKA (on average >350 mg/dL), and ketones can be detected in blood and urine in these
patients.
(Choice E) Increased protein breakdown can occur in the setting of chronic metabolic
acidosis. However, it is not a direct cause of acidosis.

Telegram Group Invite Link: https://t.me/joinchat/zyFOIg0j5goxY2E1


https://t.me/joinchat/zyFOIg0j5goxY2E1

Telegram Group Invite Link: https://t.me/USMLEWorldStep1

Aspiration
pneumonia

This patient developed septic shock secondary to pneumonia (rhonchi on lung examination and
radiographic lung opacities). His history of dementia and stroke with residual hemiparesis
and evidence of vocal cord paralysis (soft, breathy voice) is highly suggestive of aspiration
pneumonia due to inhalation of oral secretions. The superior regions of the lower lobes and
posterior regions of the upper lobes are the most dependent locations in the lungs of supine
individuals. Infiltrates in these locations revealed on chest x-ray are further evidence of
aspiration pneumonia. Anaerobic bacteria
(Peptostreptococcus, Bacteroides, Prevotella, Fusobacterium) are the dominant organisms in

Telegram Group Invite Link: https://t.me/joinchat/zyFOIg0j5goxY2E1


Telegram Group Invite Link: https://t.me/USMLEWorldStep1

the oral cavity and may be isolated from cultures from patients with aspiration pneumonia.

(Choice A) Enteral feeding, specifically oral intake, increases the risk of aspiration in patients
with dysphagia.
(Choices B and D) Reduced intercostal muscle strength can result in atelectasis and
hypoxia. Extended immobility can produce atelectasis in the posterior lungs if the patient
remains in a supine position. These conditions can also predispose to the development of
pneumonia if pulmonary hygiene is inadequate. However, aspiration pneumonia is less likely in
such patients as long as swallow and cough reflexes are intact.
(Choice C) Impaired cellular immunity usually results in recurrent viral and fungal infections.

Telegram Group Invite Link: https://t.me/joinchat/zyFOIg0j5goxY2E1


https://t.me/joinchat/zyFOIg0j5goxY2E1

Telegram Group Invite Link: https://t.me/USMLEWorldStep1

Aspiration
pneumonia

Due to gravity, aspiration pneumonia typically develops in the most dependent portions of the
lung. Patients who aspirate while lying supine typically have involvement of the posterior
segments of the upper lobes and the superior segments of the lower lobes.
The right lung is more prone to aspiration than the left lung as the right main bronchus has a
larger diameter, is shorter, and is more vertically oriented than the left main bronchus
(mnemonic: "Inhale a bite, goes down the right").
(Choice B) Aspiration in an upright position tends to involve the basilar segments of the lower
lobes.
(Choices C and D) Recumbent patients in the prone position (ie, lying on the abdomen) are
more likely to aspirate into the middle lobe and lingula, as these are the most dependent
portions of the lung in this position.

Telegram Group Invite Link: https://t.me/joinchat/zyFOIg0j5goxY2E1


Telegram Group Invite Link: https://t.me/USMLEWorldStep1

Lung cancer

This nonsmoking woman with cough and weight loss has a peripheral lung mass with an
associated pleural effusion. This presentation is concerning for adenocarcinoma of the lung,
the most common lung cancer in the general population, women,
and nonsmokers. Although tobacco is the most significant risk factor, the etiology in
nonsmokers is thought to be multifactorial. Risk factors include environmental exposures (eg,
radon, second-hand smoke) and underlying lung disease. In addition, certain molecular
alterations are seen more commonly in nonsmokers; these "driver mutations" are thought be
responsible for the development of the malignancy and include epidermal growth factor
receptor (EGFR) mutations and ALK gene rearrangements. KRAS mutations, another driver
in the formation of adenocarcinoma, is seen more commonly in smokers.
Adenocarcinoma typically presents with cough, weight loss, and hemoptysis. Changes to the
distal extremities, including clubbing (increased curvature of the nail plate) and hypertrophic
osteoarthropathy (proliferation of the bones, skin, and nails of the fingers), may occur and are
thought to be due to tumor-derived growth factors. The tumor tends be located
peripherally (as in this patient) and may present as a nodule, mass, or obstructive

Telegram Group Invite Link: https://t.me/joinchat/zyFOIg0j5goxY2E1


https://t.me/joinchat/zyFOIg0j5goxY2E1

Telegram Group Invite Link: https://t.me/USMLEWorldStep1

pneumonia. Like all malignancies, metastatic disease to the pleura can cause pleural effusions.

(Choice B) Bronchial carcinoid is a rare malignancy that tends to be located centrally, within or
adjacent to large bronchi. It typically presents with wheezing due to bronchial obstruction and
occasionally also causes carcinoid syndrome (ie, episodic diarrhea, flushing, bronchospasm).
(Choice C) Mesothelioma most commonly arises from the mesothelium lining the pleural
cavity; tumors are pleural based (ie, pleural mass with associated pleural effusion, pleural
thickening), rather than intraparenchymal. In addition, it is extremely rare in nonsmokers with
no history of asbestos exposure.
(Choice D) Small cell carcinoma is strongly associated with smoking; only 1% of cases occur in
nonsmokers. It arises from the basal cells of the bronchial epithelium and displays some
neuroendocrine features. Small cell carcinoma usually occurs in major bronchi.
(Choice E) Squamous cell carcinoma is also strongly associated with smoking. It usually
affects larger bronchi (centrally located) and arises from areas of squamous bronchial
metaplasia.

Telegram Group Invite Link: https://t.me/joinchat/zyFOIg0j5goxY2E1


Telegram Group Invite Link: https://t.me/USMLEWorldStep1

Pulmonary
hemartoma

Pulmonary hamartomas are common, benign lung lesions that usually present as
asymptomatic, incidental findings on chest imaging in middle-aged adults. In radiographs, they
are typically solitary, small (<3 cm), round, well-circumscribed lesions (ie, coin lesions) with a
popcorn pattern of calcification; they often grow slowly at the periphery of the
lung. Microscopy shows nodules of mature connective tissue (eg, cartilage, fat) and clefts lined
by entrapped respiratory epithelium.
Hamartomas are lesions composed of tissue types that are native to the organ of involvement
but show an abnormal, disorganized arrangement. Many hamartomas, including pulmonary
hamartomas, are now thought to be benign neoplastic lesions (rather than malformations), as
evidenced by the demonstration of clonal chromosomal abnormalities.
(Choice A) Microscopic features of adenocarcinoma in situ include atypical cells lining the
alveolar septa (ie, lepidic growth) without invasion. On chest x-ray, adenocarcinoma in situ

Telegram Group Invite Link: https://t.me/joinchat/zyFOIg0j5goxY2E1


https://t.me/joinchat/zyFOIg0j5goxY2E1

Telegram Group Invite Link: https://t.me/USMLEWorldStep1

often appears as a solitary, peripheral mass or as a pneumonia-like consolidation.

(Choice B) Neuroendocrine marker (eg, synaptophysin, chromogranin) expression is


characteristic of neuroendocrine tumors (eg, small cell carcinoma, carcinoid tumors). Small cell
lung carcinoma commonly presents as a rapidly growing, central mass in patients with a history
of smoking. Carcinoid tumors often arise within the bronchi and may present with symptoms of

Telegram Group Invite Link: https://t.me/joinchat/zyFOIg0j5goxY2E1


Telegram Group Invite Link: https://t.me/USMLEWorldStep1

bronchial obstruction.

(Choice C) Invasive nests of tumor cells with keratin pearl formation are a feature of squamous
cell carcinoma, which is typically a centrally located lung lesion in patients with a history of

Telegram Group Invite Link: https://t.me/joinchat/zyFOIg0j5goxY2E1


https://t.me/joinchat/zyFOIg0j5goxY2E1

Telegram Group Invite Link: https://t.me/USMLEWorldStep1

smoking.

(Choice E) Large cell carcinoma is an aggressive neoplasm that demonstrates large,


pleomorphic, epithelial tumor cells without squamous, glandular, or neuroendocrine
differentiation (ie, diagnosis of exclusion). Large cell carcinoma frequently presents as an

Telegram Group Invite Link: https://t.me/joinchat/zyFOIg0j5goxY2E1


Telegram Group Invite Link: https://t.me/USMLEWorldStep1

enlarging, necrotic, peripheral lung mass.

Telegram Group Invite Link: https://t.me/joinchat/zyFOIg0j5goxY2E1


https://t.me/joinchat/zyFOIg0j5goxY2E1

Telegram Group Invite Link: https://t.me/USMLEWorldStep1

Work of
breath

Work of breathing (WOB) is the energy expended during respiration, and comprises the work
required to overcome both elastic and airflow resistance:
• Elastic resistance is the opposition to lung expansion caused by the intrinsic elastic
properties of the lungs and chest wall; it is increased at higher tidal volumes (elastic
recoil increases at high lung volumes). Restrictive lung diseases (eg, pulmonary
fibrosis, severe obesity) are characterized by higher than normal elastic
resistance. Interstitial fibrosis increases lung stiffness whereas severe obesity increases
chest wall stiffness.
• Airflow resistance is the opposition to airflow created by limited airway diameter and
turbulent airflow; it is increased at higher respiratory rates (due to faster airflow with
increased turbulence) and at low lung volumes (due to reduced airway
diameter). Obstructive lung diseases (eg, asthma, chronic obstructive pulmonary
disease) increase airflow resistance due to bronchoconstriction and/or airway collapse.
To reduce the WOB, the tidal volume and respiratory rate are optimized by the respiratory
control centers. In patients with restrictive lung disease, the WOB is minimized when the tidal
volume is low. Therefore, to compensate for low lung volumes, rapid, shallow breathing is
favored (as in this patient). In contrast, in patients who have obstructive lung disease, the WOB
is minimized with lower respiratory rates and higher tidal volumes (ie, relatively slow, deep
breathing) (Choice C).
(Choice A) Alpha-1 antitrypsin deficiency causes panacinar emphysema, a type of chronic
obstructive pulmonary disease. This causes increased airflow resistance, and slow, deep
breaths will be favored to minimize the WOB.

Telegram Group Invite Link: https://t.me/joinchat/zyFOIg0j5goxY2E1


Telegram Group Invite Link: https://t.me/USMLEWorldStep1

(Choice B) Anxiety disorders (eg, panic attacks) are occasionally associated with rapid,
shallow breathing; however, the lungs function normally and, therefore, the WOB versus the
respiratory rate curve is normal. Because the breathing rate is higher than the normal optimal
rate, the work done against airflow resistance (and total work) is increased.
(Choice D) A fixed upper airway obstruction (eg, caused by a large goiter) leads to increased
air flow resistance and favors slow, deep breaths to minimize the WOB.
Reticulonodular
opacities

Telegram Group Invite Link: https://t.me/joinchat/zyFOIg0j5goxY2E1


https://t.me/joinchat/zyFOIg0j5goxY2E1

Telegram Group Invite Link: https://t.me/USMLEWorldStep1

As compared to normal imaging, this patient's chest x-ray shows a bilateral and diffuse pattern
of small, irregular (reticulonodular) opacities that are most pronounced in the lower
lobes. These findings are most consistent with pulmonary fibrosis, which typically manifests
with gradual-onset progressive dyspnea, nonproductive cough, fatigue, and eventual weight
loss and failure to thrive. Auscultation may reveal end-inspiratory crackles at the lung
bases. Pulmonary function tests show a restrictive pattern with decreased lung volumes,
decreased FEV1 and FVC levels, a normal or increased FEV1/FVC ratio, and decreased
diffusion capacity of carbon monoxide. Microscopically, progressive fibrosis can lead to
cystically dilated bronchioles that later coalesce to form the honeycomb appearance found in
advanced disease.
Rheumatoid arthritis (RA) can cause a variety of pulmonary manifestations; the most common is
a form of interstitial lung disease similar to idiopathic interstitial pneumonia. In addition, multiple
agents used to treat RA (eg, methotrexate, cyclophosphamide, sulfasalazine) can cause fibrotic
lung disease.

Telegram Group Invite Link: https://t.me/joinchat/zyFOIg0j5goxY2E1


Telegram Group Invite Link: https://t.me/USMLEWorldStep1

(Choice A) Atelectasis refers to diminished air volume in part of the lung and most commonly
occurs due to obstruction of the corresponding bronchus or bronchiole. Chest x-ray shows

Telegram Group Invite Link: https://t.me/joinchat/zyFOIg0j5goxY2E1


https://t.me/joinchat/zyFOIg0j5goxY2E1

Telegram Group Invite Link: https://t.me/USMLEWorldStep1

opacification (collapse) of the corresponding lobe/lobule.

Telegram Group Invite Link: https://t.me/joinchat/zyFOIg0j5goxY2E1


https://t.me/joinchat/zyFOIg0j5goxY2E1

USMLE e e r m h nne s
Me est i e s: https://t.me/USMLEMe est

K p n ssr m i e s: https://t.me/USMLEK p n

USMLE i e s Step : https://t.me/USMLE Step

e ker i e s: https://t.me/USMLE e ker

r s e n i e s Step : https://t.me/USMLE r sAn e n

r s e n i e s Step K: https://t.me/USMLE r sAn e n Step K

Ph se i e s: https://t.me/USMLEPh se

Pi ri e i e s: https://t.me/USMLEPi ri e

P th m i e s: https://t.me/USMLEP th m

nA i s i h it : https://t.me/USMLE nA i s

Am ss r p: https://t.me/USMLEAm ss

Le t ri nk: https://t.me/USMLELe t ri

En p int P s: https://t.me/USMLEEn p int

USMLE MEs: https://t.me/USMLE ME

L E Me E : https://t.me/USMLE n ineMe E

USMLE r Step : https://t.me/USMLE r Step

USMLE r Step K: https://t.me/USMLE r Step K

USMLE r Step nk: https://t.me/USMLE r Step

USMLE Anki: https://t.me/USMLEAnki

KissPrep: https://t.me/USMLEKissPrep
Telegram Group Invite Link: https://t.me/USMLEWorldStep1

(Choice B) Chest x-ray in decompensated left ventricular failure shows cardiomegaly (heart >1
hemithorax in size), pulmonary edema, pleural effusions, and enlarged pulmonary vessels.

(Choice C) Obstructive pulmonary diseases (eg, asthma, chronic obstructive pulmonary


disease) are associated with lung hyperinflation on chest x-ray. This patient has mild
hyperinflation (likely due to a deep inspiration during imaging) however the diaphragms are not

Telegram Group Invite Link: https://t.me/joinchat/zyFOIg0j5goxY2E1


https://t.me/joinchat/zyFOIg0j5goxY2E1

Telegram Group Invite Link: https://t.me/USMLEWorldStep1

markedly flattened. In addition, reticulonodular opacities are not seen in these conditions.

Telegram Group Invite Link: https://t.me/joinchat/zyFOIg0j5goxY2E1


Telegram Group Invite Link: https://t.me/USMLEWorldStep1

(Choice E) Chest x-ray in pulmonary hypertension shows enlargement of the pulmonary


arteries and the right ventricle.

Telegram Group Invite Link: https://t.me/joinchat/zyFOIg0j5goxY2E1


https://t.me/joinchat/zyFOIg0j5goxY2E1

Telegram Group Invite Link: https://t.me/USMLEWorldStep1

Pulmonary
embolism

This contrast-enhanced CT image reveals a large filling defect within the lumen of the
pulmonary artery, which is consistent with pulmonary embolism (PE). The thromboembolus
usually originates in the deep veins of the lower extremities and travels through the inferior vena
cava and right side of the heart to reach the pulmonary circulation. Large emboli lodge in the
pulmonary artery bifurcation ("saddle emboli") and may cause severe hypotension or sudden
cardiac death, whereas smaller emboli occlude peripheral pulmonary arterial branches. In most
cases, pulmonary emboli are multiple, with the lower lobes involved more often than the upper
lobes.
Many hospitalized patients are at increased risk for venous thromboembolism (VTE) (ie, PE or
deep vein thrombosis) due to trauma, prolonged immobilization, or underlying conditions that
contribute to a hypercoagulable state (eg, cancer, infection, pregnancy). Patients undergoing
large orthopedic surgeries (eg, hip replacement) are at particularly high risk of
VTE. Although the risk of VTE in hospitalized patients cannot be completely eliminated,
administration of prophylactic anticoagulation greatly reduces risk. Low-molecular-weight
heparin (eg, enoxaparin) is preferred in most patients due to proven efficacy and relatively low
risk of adverse effects.
In patients at high risk of VTE, the slightly increased risk of bleeding is strongly outweighed by
the reduced risk of VTE (and its associated mortality) provided by prophylactic
anticoagulation. Early ambulation also helps reduce the risk of VTE.
(Choice A) Beta-adrenergic antagonists (eg, metoprolol) may lower the risk of perioperative
myocardial infarction or aortic dissection, but they do not reduce the risk of VTE.

Telegram Group Invite Link: https://t.me/joinchat/zyFOIg0j5goxY2E1


Telegram Group Invite Link: https://t.me/USMLEWorldStep1

(Choice B) Fresh frozen plasma is used to reverse the effects of warfarin or to provide
coagulation factors to patients experiencing massive hemorrhage. It does not reduce the risk of
VTE and may even exacerbate an underlying hypercoagulable state.
(Choice C) Intravenous hydration can help prevent perioperative dehydration and prerenal
acute kidney injury.
(Choice E) Perioperative antibiotics (eg, cefazolin) may help prevent surgical wound infections.
(Choice F) Tissue plasminogen activator is indicated in the treatment of massive PE
complicated by hemodynamic instability. However, it is not appropriate for VTE prophylaxis due
to the relatively high risk of serious bleeding complications (eg, intracranial hemorrhage).

Telegram Group Invite Link: https://t.me/joinchat/zyFOIg0j5goxY2E1


https://t.me/joinchat/zyFOIg0j5goxY2E1

Telegram Group Invite Link: https://t.me/USMLEWorldStep1

Airway
resistance

The upper respiratory tract (eg, nasal passages, mouth, pharynx, larynx) accounts for about half
of total airway resistance. The remainder comes from the lower respiratory tract, which
begins at the trachea and proceeds through roughly 23 generations of airway bifurcation before
reaching the alveoli (ie, the trachea divides into approximately 8 × 106 [223] smaller airways
before reaching the alveoli). Because the airways at each level of the lower respiratory tract
are arranged in parallel, airway resistance is determined by the total cross-sectional area of all
the airways at that level; the greater the total cross-sectional area, the lower the airway
resistance.
The cross-sectional area of the trachea is relatively small (~3.5 cm2), but the total area of
the first few generations of bronchi is even smaller, leading to an initial small increase in

Telegram Group Invite Link: https://t.me/joinchat/zyFOIg0j5goxY2E1


Telegram Group Invite Link: https://t.me/USMLEWorldStep1

resistance from the trachea to the bronchi. However, from the medium-sized bronchi on, the
total cross-sectional area at each level of the lower respiratory tract increases in step-wise
fashion, creating a progressive decline in airway resistance. By the time the alveoli are
reached, the total cross-sectional area is massive (~5 × 105 cm2) and airway resistance is
essentially zero.

Fetal lung maturity

Surfactant is an essential lipoprotein complex that decreases alveolar surface tension and
facilitates alveolar expansion. Quantity and quality of surfactant increases throughout gestation
and correlates with greater fetal lung maturity.
Assessment of fetal lung maturity is not often indicated because infants of gestational age <32
weeks can be presumed to have immature lungs while those who are full term typically do
not. However, a subset of patients may benefit from an evaluation of lung maturity to weigh the
risks and benefits to the mother and fetus of preterm delivery versus continuing a medically
complex pregnancy.
Several markers of surfactant production and quality within amniotic fluid can help guide
estimations of fetal lung maturity:
• Lamellar bodies are organelles within type 2 pneumocytes that store, transport, and
secrete surfactant into the air-liquid interface of the alveoli. Increased lamellar body
counts within the amniotic fluid are associated with increased surfactant production and
lung maturity.
• Phosphatidylcholine (also known as lecithin [L]) and sphingomyelin (S) are lipid
components of surfactant. They are produced in approximately equal amounts until the
middle of the third trimester, at which point the phosphatidylcholine concentration
increases sharply while the sphingomyelin level remains low, as represented by
curves 1 and 2, respectively (Choices C and E). This change in composition (L/S >2)
later in gestation reflects production of a more mature surfactant.
• Phosphatidylglycerol is a phospholipid component of surfactant that increases late in the
third trimester, and its presence suggests advanced lung maturity.

Telegram Group Invite Link: https://t.me/joinchat/zyFOIg0j5goxY2E1


https://t.me/joinchat/zyFOIg0j5goxY2E1

Telegram Group Invite Link: https://t.me/USMLEWorldStep1

(Choices A and D) Amniotic fluid albumin concentrations normally decrease from mid-
gestation to term. Albumin is not a clinically significant marker of fetal development.

Telegram Group Invite Link: https://t.me/joinchat/zyFOIg0j5goxY2E1


Telegram Group Invite Link: https://t.me/USMLEWorldStep1

COPD

This patient's clinical presentation (ie, dyspnea, productive cough, hypoxemia, expiratory
wheezing, accessory muscle use, smoking history) is consistent with a chronic obstructive
pulmonary disease (COPD) exacerbation. Supplemental oxygen may be warranted in
patients with COPD who have significant hypoxemia; however, administration of excessively
high oxygen concentrations can lead to increased CO2 retention (oxygen-induced
hypercapnia), resulting in confusion and a depressed level of consciousness (eg, lethargy).
Several mechanisms contribute to oxygen-induced hypercapnia, but the major cause is
increased ventilation-perfusion mismatch. Hypoxia causes vasoconstriction of the pulmonary
arterioles, which acts to shunt blood toward alveoli with the highest ventilation, thereby
minimizing physiologic dead space. Providing high-concentration supplemental oxygen allows
lung regions with relatively poor ventilation to have higher oxygen levels, reversing pulmonary
vasoconstriction. The redistribution of blood flow away from well-ventilated alveoli leads to
an increase in physiologic dead space (well-ventilated alveoli are less perfused) with a
corresponding reduction in CO2 excretion.
(Choice A) Oxygen administration alleviates hypoxic pulmonary vasoconstriction, causing an
overall decrease in pulmonary vascular resistance.
(Choice B) Hyperoxia can increase production of reactive oxygen species (eg, superoxide,
hydroxyl radical, hydrogen peroxide), leading to injury of the airways and lung
parenchyma. Oxygen toxicity typically manifests with substernal heaviness, pleuritic chest pain,
and cough/dyspnea within 24 hours after breathing pure oxygen.

Telegram Group Invite Link: https://t.me/joinchat/zyFOIg0j5goxY2E1


https://t.me/joinchat/zyFOIg0j5goxY2E1

Telegram Group Invite Link: https://t.me/USMLEWorldStep1

(Choice C) In patients with hypercapnia and respiratory acidosis, the kidneys compensate by
increasing bicarbonate reabsorption and normalizing the serum pH level. This response occurs
slowly over several days and would tend to ameliorate the effects of hypercapnia.
(Choice D) Peripheral chemoreceptors in the carotid and aortic bodies sense arterial PaO2 and
are normally stimulated by hypoxemia. High-flow oxygen reduces chemoreceptor stimulation,
resulting in a decreased (not increased) respiratory rate and minute ventilation.
COPD

This patient's clinical picture is consistent with chronic obstructive pulmonary


disease (COPD), which is a combination of chronic bronchitis and emphysema and is most
commonly due to heavy smoking. Patients typically have progressive dyspnea and recurrent
upper respiratory infections (characteristic of chronic bronchitis).
Air trapping and airway obstruction in COPD reduce both forced expiratory volume in 1 second
(FEV1) and forced vital capacity (FVC) on pulmonary function testing (PFT). Because the
decrease in FEV1 is more profound, the FEV1/FVC ratio is also reduced, which is the hallmark
of obstructive lung disease on PFT. Air trapping raises functional residual capacity, causing
respiration to occur at a higher baseline lung volume. The residual volume (RV) is
increased. Total lung capacity (TLC) is also increased.
(Choices A and B) RV and TLC values are rarely discordant; therefore, these PFT findings are
unlikely to represent true clinical findings.
(Choice D) Normal RV with increased TLC can be seen in highly trained athletes due to
increased vital capacity.
(Choice E) Decreased RV and TLC, along with decreased FVC and increased or normal
FEV1/FVC ratio, are seen with restrictive lung disease (eg, pulmonary fibrosis). FEV1
decreases in this setting but to a lesser extent than FVC.

Telegram Group Invite Link: https://t.me/joinchat/zyFOIg0j5goxY2E1


Telegram Group Invite Link: https://t.me/USMLEWorldStep1

Radiation
injury

Radiation-induced lung injury usually occurs following thoracic irradiation for the treatment of
malignancy (eg, breast or lung cancers, lymphoma). Ionizing radiation causes DNA
damage directly (eg, double-strand breaks) and indirectly through generation of reactive oxygen
and nitrogen species that can damage both DNA and other cellular components.
In the lung, this damage primarily affects the alveolar-capillary barrier, formed from alveolar
epithelial cells (ie, pneumocytes) and vascular endothelial cells, and initiates an inflammatory
response with a cascade of cytokines (eg, IL-1, TNFα) and growth factors (eg, PDGF, TGF-
β). Manifestation vary based on chronicity:
• Acutely, immune-mediated damage results in impaired gas exchange due to exudative
alveolitis, with sloughing of pneumocytes and vascular endothelial cells,
alveolar/interstitial edema, and hyaline membrane formation. Clinically, this usually
manifests as dyspnea and cough 3-12 weeks after initial radiation exposure.

Telegram Group Invite Link: https://t.me/joinchat/zyFOIg0j5goxY2E1


https://t.me/joinchat/zyFOIg0j5goxY2E1

Telegram Group Invite Link: https://t.me/USMLEWorldStep1

• Radiation fibrosis is a delayed manifestation of radiation-induced lung injury that can


develop 6-12 months after initial radiation exposure. TGF-β released by immune cells
and the surrounding tissue initiates fibroblast proliferation, collagen deposition, and
formation of dense fibrous bands. The result is decreased lung volume, dilated
bronchi and bronchioles (ie, traction bronchiectasis), impaired mucociliary clearance,
and predisposition to chronic infections.
(Choices B and C) Hemosiderin-laden macrophages (due to diffuse alveolar hemorrhage) and
necrotizing vasculitis are both associated with autoimmune vasculitis (eg, anti-GBM
disease). These patients typically have hemoptysis and/or other systemic effects of vasculitis
(eg, glomerulonephritis, arthralgias).
(Choice D) Although significant neutrophilic infiltrate occurs in patients with radiation-induced
lung injury, frank formation of an abscess usually occurs following introduction of anaerobic
bacteria to the lung parenchyma due to aspiration (eg, alcohol use disorder) or septic embolism.
aging

Aging is associated with a number of changes in pulmonary function. Patients age >35
experience steady decreases in chest wall compliance as a result of stiffening from rib
calcification and from increased thoracic curvature due to osteoporosis and osteoarthritis. In
contrast, lung compliance increases with age due to a loss of elastic recoil, particularly in the
alveolar ducts. The dilation of alveolar ducts is homogenous, as opposed to the emphysema
associated with chronic obstructive pulmonary disease, and is not associated with destruction of
the alveolar walls.

Telegram Group Invite Link: https://t.me/joinchat/zyFOIg0j5goxY2E1


Telegram Group Invite Link: https://t.me/USMLEWorldStep1

Diminished elastic recoil and the collapse of supporting tissues around the airways cause a
significant increase in residual volume (RV). However, total lung capacity (TLC)
remains unchanged because the decreased chest wall compliance counterbalances increases
in lung compliance (Choices A, D, and E). In addition, as RV becomes a much higher
proportion of TLC (due to air trapping), forced vital capacity also decreases (Choice B). Other
age-associated changes include decreases in forced expiratory volume in 1 second and
diffusing capacity.
Asthma
Patients with intermittent asthma may be managed with an as-needed, short-acting beta-
agonist alone; however, persistent symptoms or severe disease necessitates the addition of
long-term controller medications. Inhaled glucocorticoids are recommended for all such
patients and have been shown to improve symptoms, reduce the need for short-acting
bronchodilators, and lower the risk of serious exacerbations and associated hospitalization.
Low-dose inhaled glucocorticoids are well tolerated and rarely cause systemic adverse
effects. However, poor inhalation technique can cause much of the medication to deposit on
the oral mucous membranes, where it is ineffective and can lead to oropharyngeal
candidiasis (OPC) (evidenced by white plaques that are easily scraped off). Patients can avoid
this complication by using a spacer and rinsing the mouth after inhalation. Dysphonia
unrelated to OPC can also occur with inhaled glucocorticoids due to myopathy of laryngeal

Telegram Group Invite Link: https://t.me/joinchat/zyFOIg0j5goxY2E1


https://t.me/joinchat/zyFOIg0j5goxY2E1

Telegram Group Invite Link: https://t.me/USMLEWorldStep1

muscles and mucosal irritation.

Telegram Group Invite Link: https://t.me/joinchat/zyFOIg0j5goxY2E1


Telegram Group Invite Link: https://t.me/USMLEWorldStep1

(Choice A) Excessive alcohol intake may increase the risk of developing OPC; however, this
patient's intake of 1 or 2 beers per day is within acceptable limits and is an unlikely contributing
factor.
(Choice B) Diphtheria (due to Corynebacterium diphtheriae) is a serious infection that typically
presents with pharyngitis and gray oropharyngeal pseudomembranes; it can be prevented with
vaccination. However, there is no vaccine to prevent OPC.
(Choice D) Patients with chronically elevated blood glucose levels have increased risk of
developing OPC; however, there is no evidence that this patient has poorly controlled diabetes
mellitus.

Telegram Group Invite Link: https://t.me/joinchat/zyFOIg0j5goxY2E1


https://t.me/joinchat/zyFOIg0j5goxY2E1

Telegram Group Invite Link: https://t.me/USMLEWorldStep1

(Choice E) Prophylactic antibiotics are not indicated for OPC and, in fact, are likely to increase
the risk of developing OPC by altering the normal oropharyngeal bacterial flora. Prophylactic
antifungals are also not indicated for OPC.
(Choice F) Vitamin A deficiency can impair cell-mediated immunity and create higher
susceptibility to OPC. However, patients with vitamin A deficiency typically have drying and
thickening of the skin and ocular conjunctivae. Improper administration of inhaled
glucocorticoids is far more likely in this patient.
SVC
syndrome

Telegram Group Invite Link: https://t.me/joinchat/zyFOIg0j5goxY2E1


Telegram Group Invite Link: https://t.me/USMLEWorldStep1

This patient's symptoms are consistent with superior vena cava (SVC) syndrome, a condition
that develops when SVC obstruction impairs venous return from the head, neck, upper trunk,
and upper extremities.
The SVC is formed by the union of the right and left brachiocephalic veins behind the right first
costal cartilage and descends inferiorly for 6-8 cm before draining directly into the right
atrium. During its course, the SVC is surrounded by multiple mediastinal structures (eg,
trachea, right bronchus, aorta, right pulmonary artery) and is close to the perihilar and
paratracheal lymph nodes. Due to its thin walls, the SVC can be
easily compressed by mediastinal masses (most commonly malignancy) that either extend
into the mediastinum from the central lung (eg, bronchogenic carcinoma) or nearby lymph
nodes (eg, lymphoma) or arise from other mediastinal structures.
Patients with SVC syndrome often experience the following symptoms due to increased venous
pressure in the upper body:
• Face and neck swelling and conjunctival edema
• Distended veins due to increased collateral venous drainage
• Headache, dizziness, and/or confusion due to cerebral edema and elevated intracranial
pressure
• Cough and/or dyspnea due to laryngeal edema and/or direct compression of adjacent
structures (eg, trachea) by the mediastinal mass

Telegram Group Invite Link: https://t.me/joinchat/zyFOIg0j5goxY2E1


https://t.me/joinchat/zyFOIg0j5goxY2E1

Telegram Group Invite Link: https://t.me/USMLEWorldStep1

(Choices A and B) Airway obstruction and autoimmune disease can cause dyspnea, but facial
swelling and dilated collateral veins would not be expected.
(Choice C) Inappropriate secretion of ACTH by some small cell lung cancers can cause
Cushing syndrome, which is often associated with a swollen or moon face due to increased fatty
tissue deposits. However, Cushing syndrome is not associated with conjunctival edema or
distended neck veins.
(Choice E) Pericardial effusions can cause dyspnea and distended neck veins; however, facial
swelling and headaches would not be expected. In addition, distant heart sounds or pulsus
paradoxus (decrease in systolic blood pressure >10 mm Hg during inspiration) is often present.

Telegram Group Invite Link: https://t.me/joinchat/zyFOIg0j5goxY2E1


Telegram Group Invite Link: https://t.me/USMLEWorldStep1

(Choice F) Patients with pleural effusions often have dyspnea but do not typically have facial
swelling or dilated veins. In addition, decreased breath sounds and dullness to percussion are
often present.
(Choice G) Superior pulmonary sulcus tumors (Pancoast tumors) arise at the lung apex and
can also cause SVC syndrome. However, shoulder pain is typically present due to tumor
invasion into the brachial plexus or nearby chest wall. Horner syndrome (ptosis, miosis, and
anhidrosis) also is common due to cervical sympathetic ganglia involvement. Neurologic
deficits (eg, hand weakness) also can occur. This patient's lack of shoulder pain, Horner
syndrome (his pupils are normal), and neurologic deficits (peripheral strength is normal) makes
a superior sulcus tumor unlikely.

Phrenic nerve
This patient is experiencing pleuritic chest pain due to acute lower lobe bacterial
pneumonia. Pleuritic chest pain is characterized by sharp, localized, often severe pain that is

Telegram Group Invite Link: https://t.me/joinchat/zyFOIg0j5goxY2E1


https://t.me/joinchat/zyFOIg0j5goxY2E1

Telegram Group Invite Link: https://t.me/USMLEWorldStep1

exacerbated by coughing, breathing, or changing position. It can result from any condition that
causes inflammation of the pleura (eg, infection, pulmonary embolism, uremia). The pleura are
divided into segments as follows:
Visceral pleura: The visceral (pulmonary) pleura covers all surfaces of the lungs, including the
surfaces within the pulmonary fissures. The visceral pleura does not carry pain fibers.
Parietal pleura: The parietal pleura forms the outer boundary of the pleural space and can be
subdivided as follows:

• Costal pleura: Covers the thoracic wall, including the ribs, sternum, intercostal spaces,
costal cartilages, and sides of the thoracic vertebrae
• Mediastinal pleura: Covers the mediastinum
• Diaphragmatic pleura: Covers the surface of the diaphragm located within the thoracic
cavity
• Cervical pleura: Extends with the apices of the lung into the neck

The phrenic nerve, which is derived from the C3-C5 nerve roots, delivers motor innervation to
the diaphragm and carries pain fibers from the diaphragmatic and mediastinal
pleura. Irritation of the pleura in either area will cause a sharp pain worsened by inspiration that
will be referred to the C3-C5 distribution at the base of the neck and over the shoulder. Sensory
innervation of the remainder of the parietal pleura is accomplished by intercostal nerves and is
typically felt closer to the source of the pain (Choice B).
(Choice A) The spinal accessory nerve (cranial nerve XI) provides motor innervation to the
sternocleidomastoid and trapezius muscles.
(Choice C) The long thoracic nerve innervates the serratus anterior. Damage to this nerve
causes a winged scapula.
(Choice E) The vagus nerve (cranial nerve X) is the major source of parasympathetic
innervation to the viscera of the chest and the foregut.

Telegram Group Invite Link: https://t.me/joinchat/zyFOIg0j5goxY2E1


Telegram Group Invite Link: https://t.me/USMLEWorldStep1

Hypersensitivity reaction

This patient with chronic dyspnea and cough accompanied by characteristic lung biopsy
findings most likely has hypersensitivity pneumonitis (HP). HP is an exaggerated
immunologic response to an inhaled antigen; common offending agents include mold, bacteria,
animal protein, and chemicals. The condition has variable presentation that is largely
dependent on the dose and chronicity of antigen exposure:
• Acute HP involves abrupt-onset and often recurring episodes of fever, chills, cough,
dyspnea, and fatigue that coincide with intermittent high-dose antigen
exposure. Leukocytosis is often present. Chest x-ray may be normal or show scattered
micronodular opacification.
• Chronic HP involves persistent and gradually progressive symptoms (as in this
patient) resulting from long-term, moderate-dose antigen exposure. Patients usually
have symptoms of cough, dyspnea, fatigue, and weight loss for several months or
more. Lung auscultation usually reveals fine crackles, and chest x-ray
shows interstitial reticular opacities consistent with the development of pulmonary
fibrosis.
The diagnosis of HP can be aided by pulmonary function testing that reveals a restrictive
pattern; the restriction is worse with chronic HP and is accompanied by more severe
impairments in gas exchange (eg, hypoxemia, low DLCO). Biopsy in both acute and chronic
disease reveals lymphocytic infiltrate with poorly formed noncaseating granulomas created
by the walling off of inhaled antigens. Chronic HP is associated with progressive alveolar
septal fibrosis. Removal of antigen exposure usually resolves acute disease; however, the
fibrosis occurring in chronic disease is often irreversible.

Telegram Group Invite Link: https://t.me/joinchat/zyFOIg0j5goxY2E1


https://t.me/joinchat/zyFOIg0j5goxY2E1

Telegram Group Invite Link: https://t.me/USMLEWorldStep1

(Choice A) Asthma presents with intermittent cough and dyspnea. Bronchiolar biopsy can
reveal noncaseating granulomas, but fibrosis is not typical and eosinophilic infiltrate is expected.
(Choice B) Chronic bronchitis presents with dyspnea and cough and is expected to cause
wheezing on lung auscultation. Lymphocytic infiltrate may be present on biopsy, but
granulomas are not typical.
(Choice D) Idiopathic pulmonary fibrosis can present quite similarly to chronic HP, but affected
patients are usually older (eg, age >50) and granulomas are not expected on lung biopsy.
(Choice E) Pulmonary tuberculosis typically presents with fever, cough, dyspnea, night sweats,
and weight loss. Lung biopsy demonstrates lymphocytic infiltrate, but caseating (rather than
noncaseating) granulomas.

Telegram Group Invite Link: https://t.me/joinchat/zyFOIg0j5goxY2E1


Telegram Group Invite Link: https://t.me/USMLEWorldStep1

Pleural
plaque

This CT scan demonstrates an incidental finding of pleural plaques (focal or band-like pleural
thickening, often with calcifications). Pleural plaques are a hallmark of asbestos exposure;
inhaled fibers result in chronic inflammation and collagen deposition within the pleura. They
occur most commonly along the parietal pleura covering the ribs and diaphragm. Diffuse pleural
thickening, benign pleural effusions, and mesothelioma can also occur.
Some patients eventually develop asbestosis, which presents with slowly progressive,
diffuse pulmonary fibrosis. Interstitial lung injury due to fiber inhalation predominantly affects
the lower pulmonary zones and manifests radiographically as interstitial densities that
predominate in the lung bases.
Asbestos exposure typically occurs during occupational exposure (eg, insulation
manufacturing and application, shipbuilding, drywall workers). There is usually a 20-30 year

Telegram Group Invite Link: https://t.me/joinchat/zyFOIg0j5goxY2E1


https://t.me/joinchat/zyFOIg0j5goxY2E1

Telegram Group Invite Link: https://t.me/USMLEWorldStep1

latent period between asbestos exposure and onset of symptoms.

(Choice A) Beryllium is used in the aerospace manufacturing and nuclear weapon


industries. Pulmonary berylliosis closely resembles sarcoidosis (nodular infiltrates, enlarged

Telegram Group Invite Link: https://t.me/joinchat/zyFOIg0j5goxY2E1


Telegram Group Invite Link: https://t.me/USMLEWorldStep1

lymph nodes, noncaseating granulomas) but is not strongly associated with pleural plaques.

(Choice B) Coal worker's pneumoconiosis presents radiologically as multiple small, discrete


nodules that are most prominent in the upper lung zones. Pleural plaques are not typical.
(Choice D) Hypersensitivity pneumonitis due to inhalation of agricultural dusts or bacteria (eg,
farmer's lung) tends to result in diffuse, nodular interstitial infiltrates; pleural plaques are
unexpected.
(Choice E) Petroleum processing can result in exposure to sulfur oxides and aromatic
hydrocarbons; these can cause occupational asthma and (rarely) bronchiolitis obliterans, which
result in air trapping and bronchial wall thickening (as opposed to the pleural thickening seen in
asbestos exposure).
(Choice F) Silicosis is most common in patients with a history of sandblasting or
mining. Numerous upper lobe nodular densities and eggshell calcifications of the hilar nodes
are seen; pleural plaques are unexpected.

Telegram Group Invite Link: https://t.me/joinchat/zyFOIg0j5goxY2E1


https://t.me/joinchat/zyFOIg0j5goxY2E1

Telegram Group Invite Link: https://t.me/USMLEWorldStep1

exudation

This patient's presentation is consistent with bacterial pneumonia. The infection causes the
alveoli to fill with inflammatory, neutrophil-rich exudate (pus); alveolar filling with any kind of fluid
(eg, pus, edema, blood) creates alveolar consolidation.
Alveolar consolidation leads to a number of characteristic physical examination findings. Sound
travels faster and more efficiently through liquids than gases; therefore, fluid-filled alveoli
transmit higher intensity sound than those filled with air. As a result, breath sounds and tactile
fremitus (vibration) are more prominent over areas of alveolar consolidation. Bronchophony,
a phenomenon in which a patient's spoken words (eg, "ninety-nine," "toy boat") are heard more
loudly and clearly when auscultated over an area of consolidation compared to normal lung, is
also present. The higher density of fluid-filled alveoli also creates relative dullness to
percussion compared to the resonant sound created on percussion of air-filled alveoli.
(Choice A) Accumulation of interstitial fluid within the pleural space (pleural effusion) insulates
breath sounds and vibrations that originate in the airways of the lungs. Consequently, tactile
fremitus and breath sound intensity are decreased. The fluid in the chest cavity creates
dullness to percussion over the affected area.
(Choice B) Pulmonary embolism typically has minimal effect on physical examination findings
for the lungs.
(Choice D) As in pleural effusion, accumulation of air in the pleural space (pneumothorax)
insulates sounds and vibrations that originate in the lungs, and breath sound intensity and tactile
fremitus are decreased. In contrast to pleural effusion, the relatively low density of the air in the
pleural space creates hyperresonance to percussion.
(Choice E) A mass in a proximal bronchial airway (eg, malignancy, mucus plug) can cause
airway obstruction and consequent collapse (atelectasis) of distal alveoli. Breath sounds and
tactile fremitus are decreased, and dullness to percussion is created due to the absence of air.
(Choice F) Transudation of interstitial fluid into the alveoli occurs in heart failure and can create
alveolar consolidation with physical examination findings similar to bacterial
pneumonia. However, in heart failure the findings are typically similar bilaterally. Pneumonia,

Telegram Group Invite Link: https://t.me/joinchat/zyFOIg0j5goxY2E1


Telegram Group Invite Link: https://t.me/USMLEWorldStep1

with accumulation of inflammatory exudate, is more likely in this patient with low-grade fever
and evidence of unilateral alveolar consolidation.
Occupational asthma
This patient with dyspnea, cough, and intermittent chest tightness likely has asthma. The
correlation of symptom onset after starting a new job and relief while traveling is suggestive
of occupational asthma (OA), which accounts for up to 25% of adult-onset asthma. OA is
characterized by airway inflammation, bronchial hyperreactivity, and a variable airflow
obstruction triggered by a workplace exposure. Like other forms of asthma, patients may have
normal chest imaging and pulmonary function tests between exacerbations.
Inciting factors in OA can be categorized as immunologic or nonimmunologic:
• Immunologic: Similar to atopic asthma, exposure to workplace aero-allergens induces a
Th2-mediated reaction, which stimulates IgE formation and eosinophilic
activation. There is often a latent period before symptoms develop during which the
patient is sensitized to an allergen. Common causes include cereals, latex, and
chemicals (eg, formaldehyde, amines, dyes).
• Nonimmunologic: Exposure to aero-irritants induces denudation of the airway mucosa,
resulting in persistent airway inflammation, loss of epithelial relaxation factors, and mast
cell degranulation. This typically occurs after large accidental exposures (eg, chemical
spill), and symptom onset is sudden and severe. Common causes include chlorine and
ammonia.
(Choice B) Coccidioidomycosis is endemic to the southwestern United States. In addition to
cough and chest pain, patients commonly have fever, arthralgia, and skin manifestations, and
imaging often demonstrates hilar adenopathy. This patient developed symptoms prior to his
travel to Arizona.
(Choice C) Alpha-1 antitrypsin deficiency is an inherited disorder that causes panacinar
emphysema. Patients present with dyspnea, cough, and wheezing; however, imaging typically

Telegram Group Invite Link: https://t.me/joinchat/zyFOIg0j5goxY2E1


https://t.me/joinchat/zyFOIg0j5goxY2E1

Telegram Group Invite Link: https://t.me/USMLEWorldStep1

reveals basilar bullous changes and spirometry shows an obstructive pattern.

(Choice D) BMPR2 gene mutations cause idiopathic pulmonary hypertension; symptoms


include progressive dyspnea and fatigue. However, cough, wheezing, and the resolution of
symptoms while traveling is unexpected. In addition, imaging often reveals enlarged pulmonary
arteries.
(Choice E) Eosinophilic granulomatosis with polyangiitis (Churg-Strauss) is a vasculitis of
small- or medium-sized arteries characterized by asthma and rhinitis. However, imaging
typically demonstrates patchy infiltrates, and symptoms would not be expected to resolve while
traveling.

Telegram Group Invite Link: https://t.me/joinchat/zyFOIg0j5goxY2E1


Telegram Group Invite Link: https://t.me/USMLEWorldStep1

HISTOLOGY OF BRONCHIAL
MUCOSA

Ciliated mucosal epithelium lines the pulmonary airways from the trachea to the respiratory
bronchioles. The airway epithelium gradually changes from pseudostratified ciliated columnar
Telegram Group Invite Link: https://t.me/joinchat/zyFOIg0j5goxY2E1
https://t.me/joinchat/zyFOIg0j5goxY2E1

Telegram Group Invite Link: https://t.me/USMLEWorldStep1

to ciliated simple cuboidal by the level of the terminal bronchioles. Ciliated cells continue to be
present through the proximal portions of the respiratory bronchioles, but are not present in the
alveolar ducts or in the alveoli themselves. This arrangement is important because cilia function
to propel mucus toward the pharynx. The presence of cilia beyond the most distal mucus-
producing cells (goblet cells end before the terminal bronchioles) prevents bronchiolar mucus
accumulation and airflow obstruction.
(Choice A) The rings of hyaline cartilage that encircle the trachea are replaced by cartilaginous
plates in the bronchi. These rings and plates help keep the larger airways open and end at the
termination of the smallest bronchi (they are not found in bronchioles).
(Choice C) Goblet cells are mucus-secreting cells that are present from the trachea down to
the larger bronchioles. They are not found within the terminal bronchioles, which are lined by
club cells.
(Choices D and E) The submucosal mucous glands and serous glands are found from the
trachea to the distal-most bronchi and produce mucinous components that help trap inhaled
particles. They are not present within the bronchioles.
Diaphragmatic
hernia

This neonate has respiratory distress with asymmetric breath sounds and a scaphoid
abdomen, findings concerning for congenital diaphragmatic hernia (CDH). This condition
develops in the first trimester due to incomplete fusion of the pleuroperitoneal folds, which
permits herniation of abdominal contents into the thoracic cavity (most commonly the left
side). Compression of the developing lung results in pulmonary hypoplasia, or
underdevelopment of lung tissue.
Neonates typically develop respiratory distress (eg, tachypnea, retractions) within hours of birth
and decreased unilateral breath sounds; a scaphoid abdomen from superiorly shifted
abdominal viscera is common. Chest x-ray findings include the presence of thoracic bowel

Telegram Group Invite Link: https://t.me/joinchat/zyFOIg0j5goxY2E1


Telegram Group Invite Link: https://t.me/USMLEWorldStep1

loops with minimal or absent lung markings and an indistinct hemidiaphragm on the affected
side. In addition, the distal end of a feeding tube may be seen within the thorax instead of the
abdomen.
(Choices A and D) Pulmonary surfactant deficiency occurs in premature (not term) neonates
and causes respiratory distress syndrome (RDS). X-ray reveals ground glass opacities and air
bronchograms. RDS can be complicated by bronchopulmonary dysplasia, characterized by
dilation of airspaces with bronchiolar metaplasia. Patients with CDH may also develop
bronchopulmonary dysplasia, but this is a late complication (age >30 days) and is not present at
birth.

(Choice B) Pulmonary capillary wedge pressure reflects left ventricular end-diastolic pressure
and is increased in left-sided congenital heart disease (eg, aortic stenosis). Poor perfusion
would be expected, but a scaphoid abdomen and abnormally placed feeding tube would not be
seen.
(Choice C) Pneumothorax is characterized by loss of negative intrapleural pressure and can
cause asymmetric breath sounds and respiratory distress. Although a mediastinal shift can also
be seen on imaging, a visceral pleural edge (blue arrows) with the absence of distal lung

Telegram Group Invite Link: https://t.me/joinchat/zyFOIg0j5goxY2E1


https://t.me/joinchat/zyFOIg0j5goxY2E1

Telegram Group Invite Link: https://t.me/USMLEWorldStep1

markings would be expected.

Telegram Group Invite Link: https://t.me/joinchat/zyFOIg0j5goxY2E1


https://t.me/joinchat/zyFOIg0j5goxY2E1

USMLE e e r m h nne s
Me est i e s: https://t.me/USMLEMe est

K p n ssr m i e s: https://t.me/USMLEK p n

USMLE i e s Step : https://t.me/USMLE Step

e ker i e s: https://t.me/USMLE e ker

r s e n i e s Step : https://t.me/USMLE r sAn e n

r s e n i e s Step K: https://t.me/USMLE r sAn e n Step K

Ph se i e s: https://t.me/USMLEPh se

Pi ri e i e s: https://t.me/USMLEPi ri e

P th m i e s: https://t.me/USMLEP th m

nA i s i h it : https://t.me/USMLE nA i s

Am ss r p: https://t.me/USMLEAm ss

Le t ri nk: https://t.me/USMLELe t ri

En p int P s: https://t.me/USMLEEn p int

USMLE MEs: https://t.me/USMLE ME

L E Me E : https://t.me/USMLE n ineMe E

USMLE r Step : https://t.me/USMLE r Step

USMLE r Step K: https://t.me/USMLE r Step K

USMLE r Step nk: https://t.me/USMLE r Step

USMLE Anki: https://t.me/USMLEAnki

KissPrep: https://t.me/USMLEKissPrep
Telegram Group Invite Link: https://t.me/USMLEWorldStep1

Pulmonary
embolism

This patient, with dyspnea, chest discomfort, and a perfusion defect on ventilation-perfusion
(V/Q) scan likely has a pulmonary embolism (PE). PEs are most commonly caused by deep
vein thrombosis in the lower extremities that subsequently embolizes to the pulmonary
vasculature. Signs and symptoms include acute-onset pleuritic chest pain, shortness of breath,
tachycardia, and hypotension. CT pulmonary angiography is typically the imaging modality of
choice; however, V/Q scans are helpful in evaluating patients in whom angiography is
contraindicated (eg, contrast allergy, renal failure).
V/Q scans are 2-part studies that compare regional ventilation and perfusion. The initial phase
uses a radiolabeled aerosol that is inhaled and delivered throughout the tracheobronchial
tree. The second phase uses an intravenous tracer that is distributed throughout the pulmonary
vasculature. The images are then overlaid for comparison. Normal results show even
distribution of both radionucleotide tracers throughout the lung (ie, V/Q match). V/Q
mismatch occurs when alveoli are ventilated but not perfused (ie, a defect in the perfusion
phase). This pattern can be seen in diseases that increase physiologic dead space,
including pulmonary embolism (as in this patient) and malignancies that obstruct arterial blood
flow.
(Choices B, C, and E) Endobronchial neoplasms can cause obstruction of the airway. Left
ventricular infarction can cause pulmonary edema. Exudative pleuritis causes a pleural
effusion, often with compression and atelectasis of the adjacent lung. These diseases cause
ventilation defects, typically with matched perfusion defects as the resulting alveolar hypoxia
induces pulmonary vasoconstriction. However, this patient has isolated perfusion defects.
(Choice D) Spontaneous pneumothorax would cause compression of the ipsilateral lung; V/Q
scan would show decreased lung volume on ventilation and perfusion images compared to the
normal lung.

Telegram Group Invite Link: https://t.me/joinchat/zyFOIg0j5goxY2E1


https://t.me/joinchat/zyFOIg0j5goxY2E1

Telegram Group Invite Link: https://t.me/USMLEWorldStep1

Hereditary telangietasia

This young adult has a history of recurrent epistaxis, cutaneous telangiectasias, and signs of
pulmonary vascular disease. These findings are consistent with hereditary hemorrhagic
telangiectasia (HHT) (Rendu-Osler-Weber syndrome). HHT is caused by autosomal dominant
mutations of genes involved in angiogenesis (eg, ENG, ACVRL1), leading to
multiorgan arteriovenous malformations (AVMs). In the lungs, HHT is associated
with pulmonary AVMs, which may present with cyanosis, digital clubbing, hemoptysis,
and platypnea (dyspnea aggravated by upright posture). The mechanism of platypnea is a
gravitational increase in perfusion through the arteriovenous (AV) communications in the lower
lung, worsening V/Q mismatch.
HHT can induce pulmonary hypertension (progressive dyspnea, loud P2 [pulmonic
component of S2]) in some patients. This is usually due to extensive systemic AVMs (eg,
hepatic) provoking chronic, high-output heart failure (widened pulse pressure, tachycardia)
that elevates mean pulmonary arterial pressure. HHT vascular gene mutations can also induce
hypertrophic remodeling, leading to occlusion of the small precapillary pulmonary arteries,

Telegram Group Invite Link: https://t.me/joinchat/zyFOIg0j5goxY2E1


Telegram Group Invite Link: https://t.me/USMLEWorldStep1

contributing to development of pulmonary hypertension.

(Choice A) Obstructive airway disease (cystic fibrosis, bronchiectasis, chronic obstructive


airway disease) may cause digital clubbing and a loud P2 (pulmonary hypertension due to lung
disease). However, lung examination would be abnormal (eg, wheezing, decreased air
movement). Furthermore, this patient's recurrent epistaxis, cutaneous angiomas, and platypnea
are more suggestive of HHT.
(Choices B and D) Infective endocarditis may cause digital clubbing and cutaneous
lesions. Dermal findings include Osler nodes, Janeway lesions, and splinter
hemorrhages. Involvement of the aortic valve can cause aortic regurgitation, resulting in
widened pulse pressure and heart failure. However, recurrent epistaxis, telangiectasias, and

Telegram Group Invite Link: https://t.me/joinchat/zyFOIg0j5goxY2E1


https://t.me/joinchat/zyFOIg0j5goxY2E1

Telegram Group Invite Link: https://t.me/USMLEWorldStep1

platypnea are more consistent with HHT.

Telegram Group Invite Link: https://t.me/joinchat/zyFOIg0j5goxY2E1


Telegram Group Invite Link: https://t.me/USMLEWorldStep1

Telegram Group Invite Link: https://t.me/joinchat/zyFOIg0j5goxY2E1


https://t.me/joinchat/zyFOIg0j5goxY2E1

Telegram Group Invite Link: https://t.me/USMLEWorldStep1

(Choice C) Pulmonary fibrosis can cause progressive dyspnea, digital clubbing, and a loud P2
(pulmonary hypertension due to lung disease). However, idiopathic pulmonary fibrosis affects
individuals in late adulthood, and lung examination would reveal inspiratory crackles.

Telegram Group Invite Link: https://t.me/joinchat/zyFOIg0j5goxY2E1


Telegram Group Invite Link: https://t.me/USMLEWorldStep1

Obstructive sleep
apnea

Obstructive sleep apnea (OSA) is characterized by recurrent episodes of upper airway


collapse during sleep. Anatomical and neuromuscular mechanisms have been implicated in
OSA. Neuromuscular weakness as a pathogenic mechanism in OSA is supported by the fact
that apneas occur only during sleep, a time of muscle relaxation. The upper airway dilator
muscles weaken during the transition from wake to sleep, leading to airway narrowing and
ultimately collapse in individuals with OSA.
Stimulation of the hypoglossal nerve using an implantable nerve stimulator causes the tongue
to move forward slightly, increasing the anteroposterior diameter of the airway. Studies with
these devices have shown a reduction in the number of obstructive events during sleep.
(Choices B and C) The lingual nerve arises from the mandibular division of the trigeminal
nerve and supplies sensory innervation to the tongue. The maxillary division of the trigeminal
nerve provides sensation to the mid-face area. Neither of these nerves provides motor
innervation to the oropharyngeal muscles involved in OSA.
(Choice D) The phrenic nerve supplies the diaphragm. Although diaphragmatic paralysis can
cause sleep apnea, OSA with loud snoring and gasping respirations is due to oropharyngeal
rather than diaphragmatic dysfunction.
(Choice E) The vocal cords are innervated by the recurrent laryngeal nerve, but sleep apnea is
primarily a problem of the pharynx, not larynx.

Telegram Group Invite Link: https://t.me/joinchat/zyFOIg0j5goxY2E1


https://t.me/joinchat/zyFOIg0j5goxY2E1

Telegram Group Invite Link: https://t.me/USMLEWorldStep1

dextromethorphan

Dextromethorphan (DXM) is a popular cough suppressant (antitussive agent) that is readily


available over the counter. Its primary mechanism of action is inhibition of the medullary cough
center through sigma (σ) receptor activation. Although structurally very closely related to
other opioid morphinans (eg, codeine, morphine), DXM has trivial mu (μ) and delta (δ) opioid
receptor affinity. Therefore, classic opioid adverse effects (eg, constipation, respiratory
depression, analgesia, sedation) are not seen at therapeutic doses. This favorable
pharmacologic profile makes DXM a first-choice agent for cough suppression.
However, DXM also increases serotonin activity in the CNS by decreasing the rate of
presynaptic serotonin reuptake and directly stimulating serotonin receptors. For this reason,
DXM abuse can cause serotonin syndrome, manifesting as muscular hypertonia, spasticity
(eg, clonus, hyperreflexia), autonomic instability (eg, hyperthermia, hypertension), and
encephalopathy. This potentially fatal condition most often occurs in settings of overdose or
accidental drug interaction with other serotonergic drugs (eg, selective serotonin reuptake

Telegram Group Invite Link: https://t.me/joinchat/zyFOIg0j5goxY2E1


Telegram Group Invite Link: https://t.me/USMLEWorldStep1

inhibitors, triptans, monoamine oxidase inhibitors).

(Choice A) Although DXM has the potential for abuse, addiction is less likely than with opioid
cough suppressants (eg, codeine). Recreational drug use typically begins in peer settings, and
adolescents and young adults are at greatest risk; this 39-year-old woman has no history of
recreational drug use and is likely at low risk.
(Choice C) DXM has no intrinsic hepatotoxicity. However, acute liver injury can occur in
overdose scenarios involving combination dextromethorphan-acetaminophen preparations (eg,
multi-symptom cold remedies).

Telegram Group Invite Link: https://t.me/joinchat/zyFOIg0j5goxY2E1


https://t.me/joinchat/zyFOIg0j5goxY2E1

Telegram Group Invite Link: https://t.me/USMLEWorldStep1

(Choice D) Although it has negligible μ receptor agonism at therapeutic doses, DXM at high
doses may cause intoxication that can occasionally elicit enough μ receptor agonism to produce
hypoventilation and stupor. These symptoms can be reversed with naloxone.
(Choice E) Patients who use over-the-counter decongestants (eg, oxymetazoline or
phenylephrine sprays, oral pseudoephedrine) for prolonged periods can develop rhinitis
medicamentosa, characterized by rebound nasal congestion, rhinorrhea, and turbinate
hypertrophy. However, rebound cough is not observed on DXM withdrawal.
ARDS

This hospitalized patient with worsening hypoxemia and bilateral lung infiltrates mostly likely
has acute respiratory distress syndrome (ARDS), a potential complication of acute
pancreatitis. ARDS involves an inflammatory response in the lungs that leads to alveolar
capillary leakage and diffuse pulmonary edema; the edema prevents ventilation of affected
alveoli and facilitates alveolar collapse, leading to increased intrapulmonary shunting (perfusion
without ventilation). In addition, functional residual capacity (FRC) (ie, the volume of air in the

Telegram Group Invite Link: https://t.me/joinchat/zyFOIg0j5goxY2E1


Telegram Group Invite Link: https://t.me/USMLEWorldStep1

lungs at end-tidal expiration) is reduced.

Telegram Group Invite Link: https://t.me/joinchat/zyFOIg0j5goxY2E1


https://t.me/joinchat/zyFOIg0j5goxY2E1

Telegram Group Invite Link: https://t.me/USMLEWorldStep1

Mechanical ventilation is typically needed for the management of ARDS because it allows for
the application of positive end-expiratory pressure (PEEP). PEEP helps treat ARDS
by opening collapsed alveoli to reduce intrapulmonary shunting (Choice D) and increase
FRC back to near-normal levels. The increase in FRC has the following beneficial effects:
• There is a critical lung capacity (ie, closing capacity) at which some of the lung's small
airways collapse during expiration (due to decreased radial traction at lower lung
volumes). When FRC drops below this critical capacity, alveoli supplied by the
collapsed airways are without ventilation for part of the respiratory cycle, which
contributes to intrapulmonary shunting and increases ventilation-perfusion
mismatching. Increasing FRC minimizes the time spent below closing capacity and in
doing so decreases ventilation-perfusion mismatching.
• Because the FRC air volume remains in the lungs throughout the respiratory cycle, it
acts as a store of oxygen that the body can pull from during brief periods of increased
need. Therefore, increasing FRC increases the oxygen reserves in the lungs.

Telegram Group Invite Link: https://t.me/joinchat/zyFOIg0j5goxY2E1


Telegram Group Invite Link: https://t.me/USMLEWorldStep1

(Choice A) PEEP increases, rather than decreases, alveolar pressure, which has the negative
effect of increasing the risk of pulmonary barotrauma.
(Choice B) Intrapleural pressure is normally negative throughout the respiratory cycle due to
the opposing elasticity of the lungs (tend to collapse) and chest wall (tends to
expand). Increased end-expiratory airway pressure during PEEP reduces the collapsing force
of the lungs, causing the intrapleural pressure to increase.
(Choice E) Minute ventilation is determined by the product of tidal volume and respiratory
rate. PEEP causes breathing to occur at a higher baseline lung volume (ie, higher FRC), but
tidal volume and respiratory rate are not directly affected, and minute ventilation is unchanged.

HYPOXEMIA

This patient has an acute benzodiazepine overdose. Benzodiazepines (eg, alprazolam,


lorazepam) cause sedation and central respiratory depression
with hypoventilation and CO2 retention; therefore, affected patients have acute respiratory
acidosis (low pH and PaCO2 >40 mm Hg). Because full metabolic compensation by the kidneys
(ie, HCO3− retention) requires approximately 72 hours, there is minimal compensation in the
acute setting, and the expected serum HCO3− is near normal (24 mEq/L).
The alveolar-arterial (A-a) O2 gradient is the difference between the partial pressure of alveolar
oxygen (PAO2) and the partial pressure of arterial oxygen (PaO2); it represents the efficiency of
gas transfer between the lungs and the circulation. A small gradient (eg, 4-15 mm Hg)
is normal in part because there is physiologic shunting of unoxygenated bronchial blood, which
decreases PaO2. In addition to CO2 retention, hypoventilation also causes low PAO2 that leads
directly to hypoxemia (PaO2 <75 mm Hg). The efficiency of gas transfer is intact and a normal

Telegram Group Invite Link: https://t.me/joinchat/zyFOIg0j5goxY2E1


https://t.me/joinchat/zyFOIg0j5goxY2E1

Telegram Group Invite Link: https://t.me/USMLEWorldStep1

A-a O2 gradient is expected.

Telegram Group Invite Link: https://t.me/joinchat/zyFOIg0j5goxY2E1


Telegram Group Invite Link: https://t.me/USMLEWorldStep1

Telegram Group Invite Link: https://t.me/joinchat/zyFOIg0j5goxY2E1


https://t.me/joinchat/zyFOIg0j5goxY2E1

Telegram Group Invite Link: https://t.me/USMLEWorldStep1

Telegram Group Invite Link: https://t.me/joinchat/zyFOIg0j5goxY2E1


Telegram Group Invite Link: https://t.me/USMLEWorldStep1

(Choice A) PaCO2 <40 mm Hg indicates respiratory alkalosis. Respiratory alkalosis (ie,


hyperventilation) with hypoxemia and a normal A-a O2 gradient is expected at high altitude. The
efficiency of gas transfer is intact, but low partial pressure of inspired oxygen (PiO2) causes low
PAO2.
(Choice B) Respiratory alkalosis with hypoxemia and an elevated A-a O2 gradient is expected
with lung pathology that causes acute V/Q mismatch (eg, pulmonary embolism and pneumonia).

Telegram Group Invite Link: https://t.me/joinchat/zyFOIg0j5goxY2E1


https://t.me/joinchat/zyFOIg0j5goxY2E1

Telegram Group Invite Link: https://t.me/USMLEWorldStep1

Telegram Group Invite Link: https://t.me/joinchat/zyFOIg0j5goxY2E1


Telegram Group Invite Link: https://t.me/USMLEWorldStep1

(Choices C and E) Respiratory acidosis with hypoxemia and an elevated A-a O2 gradient is
expected with chronic obstructive pulmonary disease. These patients have chronic
CO2 retention, and O2 exchange is impaired by emphysematous disruption of gas

Telegram Group Invite Link: https://t.me/joinchat/zyFOIg0j5goxY2E1


https://t.me/joinchat/zyFOIg0j5goxY2E1

Telegram Group Invite Link: https://t.me/USMLEWorldStep1

diffusion across the alveolar-capillary membrane.

Telegram Group Invite Link: https://t.me/joinchat/zyFOIg0j5goxY2E1


Telegram Group Invite Link: https://t.me/USMLEWorldStep1

sarcoidosis

This patient has tender erythematous nodules on the anterior lower extremities, consistent
with erythema nodosum, a nonspecific, delayed hypersensitivity reaction that can be seen in a
variety of infections or inflammatory disorders. However, in combination with hilar adenopathy
and elevated ACE levels (formed by activated macrophages), this presentation is highly
suggestive of sarcoidosis. Sarcoidosis is a systemic inflammatory disorder of unknown
etiology characterized pathologically by noncaseating granulomas.
Granulomas can occur in any organ but commonly involve the lymph nodes (eg, bilateral hilar
adenopathy), lungs (interstitial lung disease), joints (eg, arthralgias), eyes (uveitis), or
skin. Most patients with sarcoidosis also develop liver involvement, typically in the form
of asymptomatic hepatomegaly with mild liver function test abnormalities (alkaline

Telegram Group Invite Link: https://t.me/joinchat/zyFOIg0j5goxY2E1


https://t.me/joinchat/zyFOIg0j5goxY2E1

Telegram Group Invite Link: https://t.me/USMLEWorldStep1

phosphatase > aminotransferases). Biopsy will show scattered noncaseating granulomas,


predominantly around the portal veins.
(Choice A) Centrilobular necrosis is the death of hepatocytes immediately surrounding the
terminal hepatic vein. Ischemic injury (as in right-sided heart failure), drugs, toxins, and
fulminant hepatitis can be responsible.
(Choice B) Fatty change (steatosis) is most commonly seen in alcohol use or obesity, although
it can also be seen with a reversible hypoxic, toxic, or metabolic injury (eg, protein malnutrition).
(Choice C) Nodular regeneration is seen with cirrhosis. Sarcoidosis does not typically
progress to hepatic cirrhosis.
(Choice D) Periportal fibrosis may be seen in chronic cholestasis (eg, primary sclerosing
cholangitis, primary biliary cholangitis) or chronic viral hepatitis; it is not associated with
sarcoidosis.
(Choice E) Portal inflammation is a very nonspecific finding that occurs with many forms of
hepatitis, toxin- and drug-induced liver injury, and metabolic and cholestatic liver diseases.

Telegram Group Invite Link: https://t.me/joinchat/zyFOIg0j5goxY2E1


https://t.me/joinchat/zyFOIg0j5goxY2E1

USMLE e e r m h nne s
Me est i e s: https://t.me/USMLEMe est

K p n ssr m i e s: https://t.me/USMLEK p n

USMLE i e s Step : https://t.me/USMLE Step

e ker i e s: https://t.me/USMLE e ker

r s e n i e s Step : https://t.me/USMLE r sAn e n

r s e n i e s Step K: https://t.me/USMLE r sAn e n Step K

Ph se i e s: https://t.me/USMLEPh se

Pi ri e i e s: https://t.me/USMLEPi ri e

P th m i e s: https://t.me/USMLEP th m

nA i s i h it : https://t.me/USMLE nA i s

Am ss r p: https://t.me/USMLEAm ss

Le t ri nk: https://t.me/USMLELe t ri

En p int P s: https://t.me/USMLEEn p int

USMLE MEs: https://t.me/USMLE ME

L E Me E : https://t.me/USMLE n ineMe E

USMLE r Step : https://t.me/USMLE r Step

USMLE r Step K: https://t.me/USMLE r Step K

USMLE r Step nk: https://t.me/USMLE r Step

USMLE Anki: https://t.me/USMLEAnki

KissPrep: https://t.me/USMLEKissPrep
Telegram Group Invite Link: https://t.me/USMLEWorldStep1

Pulmonary
fibrosis

The pressure/volume relationship in this patient's lungs depicts dramatically reduced


compliance (ie, for any given volume the pressure will be significantly increased). Reduced
pulmonary parenchymal compliance is the hallmark of pulmonary fibrosis.
(Choice A) Alpha-1 antitrypsin deficiency causes panacinar emphysema and liver
cirrhosis. In emphysema, the lung parenchyma has increased compliance.
(Choice B) Asthma decreases peak expiratory flow rates and FEV1 during pulmonary function
testing, but it does not decrease lung compliance.
(Choice C) Fibromyalgia causes diffuse chronic axial musculoskeletal pain and
tenderness. The lung parenchyma is normal.

Telegram Group Invite Link: https://t.me/joinchat/zyFOIg0j5goxY2E1


https://t.me/joinchat/zyFOIg0j5goxY2E1

Telegram Group Invite Link: https://t.me/USMLEWorldStep1

(Choice D) Idiopathic pulmonary arterial hypertension does not significantly affect lung
compliance.
Blood
loss

Oxygen is carried in the blood both as a dissolved gas and in combination with
hemoglobin. The total oxygen content of the blood is determined primarily by the amount of
hemoglobin in the blood and its percentage of oxygen saturation (SaO2). Dissolved oxygen is
not bound to hemoglobin and accounts for a very small proportion of the total oxygen content of
blood due to low solubility in plasma. Changes in arterial oxygen tension (PaO2) affect both the
SaO2 and the amount of oxygen dissolved in the plasma; however, the change in SaO2 is far
more influential on the total oxygen content of the blood.
This patient has arterial pO2 (PaO2) and SaO2 within the normal range, but decreased total blood
oxygen content (CaO2). If the PaO2 and SaO2 are both normal, the lower total blood oxygen
content is most likely secondary to a lower hemoglobin concentration. A common cause of
anemia in premenopausal women is chronic blood loss secondary to menstruation.
(Choice B) Cyanide inhibits cellular oxidative phosphorylation by inhibiting Fe3+ in cytochrome c
oxidase, lowering peripheral tissue oxygen consumption. Arterial PaO2, SaO2, and CaO2 remain
unchanged, but venous oxygen content rises and the arterial-venous oxygen gradient
falls. Cyanide may also bind to the ferrous (Fe2+) iron of hemoglobin to form cyanohemoglobin,
but only in small amounts that do not appreciably change arterial oxygen content.
(Choice C) Reduced alveolar oxygen tension (PAO2) (eg, at high altitude) or disruption of gas
exchange at the alveolar-capillary interface can reduce the PaO2 and SaO2. In addition, factors
that reduce the affinity of hemoglobin for oxygen (eg, acidosis) can lower SaO2 (Bohr-Haldane

Telegram Group Invite Link: https://t.me/joinchat/zyFOIg0j5goxY2E1


Telegram Group Invite Link: https://t.me/USMLEWorldStep1

effect).

(Choice D) Obesity causes alveolar hypoventilation due to reduced chest wall


compliance. Obese individuals also have increased lower lobe perfusion causing ventilation
perfusion mismatch. These factors, in combination with other complex mechanisms, lead to
hypoxemia (low PaO2), low SaO2, and hypercapnia.
(Choice E) In tetralogy of Fallot, mixing of deoxygenated and oxygenated blood occurs due to
ventricular septal defect, lowering PaO2 of the left ventricle. The aorta overrides the right and

Telegram Group Invite Link: https://t.me/joinchat/zyFOIg0j5goxY2E1


https://t.me/joinchat/zyFOIg0j5goxY2E1

Telegram Group Invite Link: https://t.me/USMLEWorldStep1

left ventricles, delivering blood with low arterial PaO2 and SaO2 to the periphery.

hypoxemia

Telegram Group Invite Link: https://t.me/joinchat/zyFOIg0j5goxY2E1


Telegram Group Invite Link: https://t.me/USMLEWorldStep1

This patient with chronic fatigue, dyspnea, difficulty concentrating, hypoxemia (PaO2 <75 mm
Hg), and hypercapnia (PaCO2 >45 mm Hg) in the setting of obesity (BMI >30 kg/m2) likely
has obesity hypoventilation syndrome (OHS). OHS results from physical restriction of lung
expansion by excessive thoracic tissue mass. As such, the hypoventilation mostly results
from reduced tidal volume and patients may have normal or increased respiratory rate. Most
patients with OHS also have concomitant obstructive sleep apnea (OSA); unlike isolated OSA
that involves hypoventilation only with sleep, OHS involves hypoventilation throughout the
waking hours.
In healthy individuals, a normal alveolar to arterial (A-a) gradient ranges from 4-15 mm Hg, with
older individuals having a gradient towards the higher end of normal. A normal A-a
gradient indicates that the efficiency of gas exchange between the alveoli and the blood is
intact; therefore, hypoxemia must result from low partial pressure of alveolar oxygen
(PAO2). Low PAO2 can occur due to alveolar hypoventilation (eg, OHS, neuromuscular
disease, central depression of respiratory drive) or inspiration of low partial pressure of oxygen
at high altitude.
(Choices B and C) Destruction of lung parenchyma occurs with both emphysema and
interstitial lung disease. This leads to disruption of the alveolar-capillary membrane and
impaired diffusion capacity with an increased A-a gradient. In OHS, the lung parenchyma is
undamaged and diffusion capacity is intact.
(Choice D) Although the lung parenchyma is unaffected in OHS, restricted expansion of the
chest wall secondarily restricts lung expansion, effectively decreasing lung compliance.
(Choice E) Tissue oxygen consumption does increase with increased body mass, but it is a
relatively minor factor and would not cause hypoxemia or hypercapnia in the absence of
hypoventilation.
(Choice F) Right-to-left shunting (eg, Eisenmenger syndrome) results in hypoxemia with an
elevated A-a gradient because a large percentage of cardiac output bypasses the alveolar
capillaries and does not undergo gas exchange.

Telegram Group Invite Link: https://t.me/joinchat/zyFOIg0j5goxY2E1


https://t.me/joinchat/zyFOIg0j5goxY2E1

Telegram Group Invite Link: https://t.me/USMLEWorldStep1

auscultation

This patient most likely has a right-sided pleural effusion. Excess fluid within the pleural space
acts to insulate vibrations and breath sounds that originate in the airways of the
lungs. Consequently, tactile fremitus, the transmission of vibration from vocalized sound (eg,
saying "ninety-nine"), is decreased over a pleural effusion. Breath sounds are
also decreased or absent. The high density of pleural fluid compared to normal lung (alveolus-
air composite) causes dullness to percussion over the effusion.
(Choice A) Bronchospasm is likely to have minimal effect on tactile fremitus and dullness to
percussion.
(Choice B) The hyperinflated alveoli in emphysema should demonstrate hyperresonance to
percussion and decreased tactile fremitus.
(Choices C and G) Because sound vibrations travel faster and more efficiently through liquids
than through gases, alveolar filling processes such as lobar consolidation (alveoli filled with pus)
and pulmonary edema (alveoli filled with transudate) create increased breath sound intensity
and increased tactile fremitus.
(Choice D) A large pericardial effusion might cause dullness to percussion with decreased
tactile fremitus over the precordium, but not over the lower lung.
(Choice F) Like pleural effusion, pneumothorax (air in the pleural space) acts to insulate sound
originating in the airways; therefore, tactile fremitus and breath sounds are
decreased. However, the low density of air compared to normal lung creates hyperresonance
to percussion.

Telegram Group Invite Link: https://t.me/joinchat/zyFOIg0j5goxY2E1


Telegram Group Invite Link: https://t.me/USMLEWorldStep1

Interstial lung
disease

This patient's clinical presentation (progressive dyspnea, fine crackles, clubbing, diffuse reticular
opacities) is consistent with interstitial lung disease (ILD). Most ILDs cause
progressive pulmonary fibrosis with thickening and stiffening of the pulmonary
interstitium. This causes increased lung elastic recoil, as well as airway widening due to
increased outward pulling (radial traction) by the surrounding fibrotic tissue. The resulting
decrease in airflow resistance leads to supernormal expiratory flow rates (higher than normal
when corrected for lung volume).
Additional pulmonary function test findings in restrictive lung diseases include reduced total lung
capacity, vital capacity, inspiratory capacity, functional residual capacity, and residual
volume. The FVC and FEV1 are decreased as well. However, the FEV1/FVC ratio is typically
normal or increased as FEV1 decreases less than FVC (due to airway widening relative to the
low lung volumes). In addition, fibrosis causes a reduction in the diffusion capacity of carbon

Telegram Group Invite Link: https://t.me/joinchat/zyFOIg0j5goxY2E1


https://t.me/joinchat/zyFOIg0j5goxY2E1

Telegram Group Invite Link: https://t.me/USMLEWorldStep1

monoxide.

Telegram Group Invite Link: https://t.me/joinchat/zyFOIg0j5goxY2E1


Telegram Group Invite Link: https://t.me/USMLEWorldStep1

(Choice A) Lung compliance is defined as the change in lung volume for a given change in
pressure. Increased compliance means that the lung tissue stretches and expands more in
response to increased pressure. Restrictive lung diseases cause decreased lung compliance
and lower lung volumes due to interstitial fibrosis.
(Choice B) The outward recoil of the chest wall limits the volume of air that can be expired
during maximal exhalation. An increase in outward chest wall recoil would decrease expiratory
flow rates, as the respiratory muscles must work harder during expiration to oppose the
increased outward force.
(Choice C) Physiological dead space refers to the volume of inspired air that does not
participate in gas exchange. Changes in the amount of physiological dead space can affect
alveolar ventilation but do not affect expiratory flow rates.
(Choice E) Thickening of the bronchioles, which occurs in some obstructive lung diseases (eg,
asthma), leads to narrowing of the airways. Although reduced airway radius leads to local
increase in airflow velocity, overall the increased resistance causes a decrease in expiratory

Telegram Group Invite Link: https://t.me/joinchat/zyFOIg0j5goxY2E1


https://t.me/joinchat/zyFOIg0j5goxY2E1

Telegram Group Invite Link: https://t.me/USMLEWorldStep1

flow rates. In fibrotic lung disease, the interstitium, not the bronchiolar walls, is thickened.

Histoplasma capsulatum
This patient has calcified lung, mediastinal, and splenic lesions that are stable over time and
asymptomatic, raising suspicion for a contained granulomatous infection. The most likely
cause is the dimorphic fungus Histoplasma capsulatum, which exists as a mold in the soil of
the Ohio and Mississippi River Valleys. This pathogen is inhaled into the lungs, converts to
yeast form, and is phagocytosed by alveolar macrophages. Macrophages cannot initially
eliminate the organism due to microbial virulence factors that prevent phagolysosome formation
and acidification. Therefore, H capsulatum is able to replicate within the macrophage and
spread through the draining lymphatic system and (often) into the reticuloendothelial system
(eg, spleen, liver).
After about 2 weeks, patients with intact immunity develop a cell-mediated immune response
that contains the infection within granulomas. Over time, the granulomas fibrose
and calcify and can be visualized on radiographic imaging at the initial sites of infection (eg,
lungs, hilar and mediastinal lymph nodes, spleen). In healthy patients, most infections with H
capsulatum are asymptomatic and are therefore often discovered incidentally.

Telegram Group Invite Link: https://t.me/joinchat/zyFOIg0j5goxY2E1


Telegram Group Invite Link: https://t.me/USMLEWorldStep1

(Choice B) Metastatic cancer often causes multiple lung lesions. However, these lesions
generally enlarge in size over time and calcification is rare.

(Choice C) Miliary tuberculosis is marked by multiple pulmonary lesions and clinical illness (eg,
fever, night sweats); calcified lesions would be atypical. Although primary tuberculosis is often
associated with a calcified lung and an ipsilateral hilar lymph node lesion (Ranke complex),

Telegram Group Invite Link: https://t.me/joinchat/zyFOIg0j5goxY2E1


https://t.me/joinchat/zyFOIg0j5goxY2E1

Telegram Group Invite Link: https://t.me/USMLEWorldStep1

tuberculosis is unlikely in this case given the patient's negative tuberculin skin test.

(Choice D) Parathyroid adenoma is a benign tumor that does not spread to the lungs. It is
often associated with hypercalcemia.
(Choice E) Silica inhalation can cause innumerable, small, rounded opacities in the upper
lobes of the lungs that may fibrose over time, resulting in pulmonary disease. Silica exposure is
primarily occupational (eg, miners, sandblasters).

Telegram Group Invite Link: https://t.me/joinchat/zyFOIg0j5goxY2E1


Telegram Group Invite Link: https://t.me/USMLEWorldStep1

emphysema

The gross pathology of this patient's lung shows multiple large subpleural blebs consistent with
severe emphysema. Emphysema most commonly develops due to chronic exposure to
cigarette smoke via the following mechanisms:

Telegram Group Invite Link: https://t.me/joinchat/zyFOIg0j5goxY2E1


https://t.me/joinchat/zyFOIg0j5goxY2E1

Telegram Group Invite Link: https://t.me/USMLEWorldStep1

• Inflammation and leukocyte infiltration: Alveolar macrophages release inflammatory


cytokines (eg, tumor necrosis factor) and recruit neutrophils and CD8+ T lymphocytes to
the alveolar tissue.
• Protease-antiprotease imbalance: Inflammatory cells release tissue-destructive
proteases (eg, elastase) often in the setting of decreased production of antiproteases
due to genetic predisposition or outright genetic mutation (eg, alpha-1 antitrypsin
deficiency).
• Oxidative stress: Cigarette smoke and chronic inflammation increase exposure to
reactive oxygen species that impart further tissue damage.

The ongoing tissue damage leads to loss of alveolar elasticity and consequent alveolar
distension. In severe cases, large air spaces called subpleural blebs can form, mostly in the
lung apex. These subpleural blebs can sometimes rupture leading to a spontaneous
pneumothorax.
(Choice A) Ferruginous bodies embedded within interstitial fibrous tissue is characteristic of
pulmonary asbestosis. The ferruginous bodies represent foreign particles enclosed in iron-rich

Telegram Group Invite Link: https://t.me/joinchat/zyFOIg0j5goxY2E1


Telegram Group Invite Link: https://t.me/USMLEWorldStep1

material and they typically appear as brown "barbell" shaped structures on histopathology.

Telegram Group Invite Link: https://t.me/joinchat/zyFOIg0j5goxY2E1


https://t.me/joinchat/zyFOIg0j5goxY2E1

Telegram Group Invite Link: https://t.me/USMLEWorldStep1

(Choice B) Hemosiderin-laden macrophages with congested airways are consistent


with cardiogenic pulmonary edema.

Telegram Group Invite Link: https://t.me/joinchat/zyFOIg0j5goxY2E1


Telegram Group Invite Link: https://t.me/USMLEWorldStep1

(Choice D) Multinucleated giant cells surrounding caseating granulomas


characterize tuberculosis.

Telegram Group Invite Link: https://t.me/joinchat/zyFOIg0j5goxY2E1


https://t.me/joinchat/zyFOIg0j5goxY2E1

Telegram Group Invite Link: https://t.me/USMLEWorldStep1

(Choice E) Necrotizing arteritis with adjacent palisading epithelioid histiocytes is consistent


with granulomatosis with polyangiitis.

Telegram Group Invite Link: https://t.me/joinchat/zyFOIg0j5goxY2E1


Telegram Group Invite Link: https://t.me/USMLEWorldStep1

(Choice F) Patchy interstitial fibrosis with the presence of fibroblastic foci is consistent
with idiopathic pulmonary fibrosis.

smoke

Telegram Group Invite Link: https://t.me/joinchat/zyFOIg0j5goxY2E1


https://t.me/joinchat/zyFOIg0j5goxY2E1

Telegram Group Invite Link: https://t.me/USMLEWorldStep1

Secondhand smoke (SHS) has numerous adverse effects on nearly every organ system. The
level of tobacco in the fetus of a mother who smokes is the same as that of an active
smoker. Maternal tobacco use impairs fetal oxygenation, alters fetal development and
response, and exposes the fetus to multiple toxins (eg, nicotine, carbon monoxide,
ammonia). As a result, detrimental outcomes include abnormal placentation (eg, previa,
abruption), prematurity, perinatal mortality, and significantly reduced birth weight.
One of the most dangerous effects of SHS exposure (pre- and postnatal) is the increased risk
of sudden infant death syndrome (SIDS). SIDS refers to the unexpected death of a
seemingly healthy infant during sleep. Up to half of all SIDS cases are due to SHS exposure,
likely due to impaired arousal and abnormal cardiovascular responses to stimuli.
SHS also increases the risk of recurrent otitis media, asthma, and other respiratory tract
illnesses (eg, pneumonia) in children. Parents who smoke outside the home should be
counseled on cessation as chemicals from cigarette smoke are adsorbed and retained by
clothing, skin, and hair and therefore pose a risk to children.
(Choices A and B) Although the risk of asthma increases with secondhand smoke exposure,
there appears to be no increased risk of other atopic diseases, such as eczema or food
allergies. The major risk factor for atopic disease is a family history of atopy.
(Choice C) Although prenatal smoke exposure is associated with low birth weight and poor
fetal growth, it does not increase the risk of obesity. The primary risk factors for childhood
obesity are parental obesity and excessive television viewing.
(Choice E) SHS exposure has been shown to reduce renal function (decreased glomerular
filtration rate) in adolescents but does not increase the risk of urinary tract infections. Risk
factors for recurrent urinary tract infections in children include anatomic anomalies (eg, posterior
urethral valve, vesicoureteral reflux).

Telegram Group Invite Link: https://t.me/joinchat/zyFOIg0j5goxY2E1


Telegram Group Invite Link: https://t.me/USMLEWorldStep1

Cystic fibrosis

This patient with chronic lung problems has autopsy findings consistent with cystic
fibrosis (CF), an autosomal recessive disorder caused by a genetic mutation (eg, ΔF508)
affecting the CF transmembrane conductance regulator. A defect in this chloride
channel prevents normal hydration of mucus and results in the accumulation of thick, viscous
secretions throughout the body (eg, lungs, pancreas, vas deferens).
Respiratory disease, the most common CF manifestation, is the predominant cause of morbidity
and mortality. Patients typically have chronic cough due to impaired clearance of inspissated
secretions in the bronchioles. This buildup leads to mucus plugging (ie, obstructive lung
disease), bacterial colonization (ie, recurrent pneumonia), and chronic infiltration of
inflammatory cells. Over time, elastase produced by neutrophils
causes bronchiectasis (weakened, dilated bronchial walls) and parenchymal
destruction. Advanced disease is associated with irreversible damage, progressive respiratory

Telegram Group Invite Link: https://t.me/joinchat/zyFOIg0j5goxY2E1


https://t.me/joinchat/zyFOIg0j5goxY2E1

Telegram Group Invite Link: https://t.me/USMLEWorldStep1

failure, and shortened life expectancy.

Telegram Group Invite Link: https://t.me/joinchat/zyFOIg0j5goxY2E1


Telegram Group Invite Link: https://t.me/USMLEWorldStep1

(Choice A) Accumulation of proteinaceous material within alveoli occurs with pulmonary


alveolar proteinosis, a condition in which clearance of surfactant by alveolar macrophages is
impaired. Inflammatory cells are not present in this disorder.

Telegram Group Invite Link: https://t.me/joinchat/zyFOIg0j5goxY2E1


https://t.me/joinchat/zyFOIg0j5goxY2E1

Telegram Group Invite Link: https://t.me/USMLEWorldStep1

(Choice B) Antigen-mediated aggregation of macrophages describes the formation of


granulomas, as occurs with Mycobacterium tuberculosis and Histoplasma
capsulatum infection. The inciting organism can usually be identified on microscopy and mucus
plugs are not characteristically present.
(Choice C) Deficiency of alpha-1 antitrypsin (protease inhibitor of elastase) causes liver
dysfunction, emphysema, and eventually bronchiectasis. The onset of pulmonary symptoms
typically occurs at age >20, and mucus plugs would not be expected.

(Choice E) Increased pulmonary venous pressure is characteristic of left-sided heart failure,


which results in pulmonary congestion and interstitial and alveolar edema. Bronchiectasis is not
an associated finding.
Loeffler syndrome
This patient's presentation is concerning for Loeffler syndrome, a transient eosinophilic
pneumonitis (eg, dry cough, dyspnea, wheezing) caused by migration of parasitic
roundworm larva (eg, Ascaris suum from pigs) through the lungs. His peripheral smear shows
numerous erythrocytes and a few scattered platelets accompanied by 2 larger cells
with bilobed nuclei and eosinophilic granules; the 2 larger cells are eosinophils.

Telegram Group Invite Link: https://t.me/joinchat/zyFOIg0j5goxY2E1


Telegram Group Invite Link: https://t.me/USMLEWorldStep1

Eosinophils play a prominent role in allergic disease (eg, asthma, drug reactions) and in
defense against parasitic infection. Major basic protein is the predominant substance
contained within eosinophilic granules, and it acts as a potent toxin against helminths (parasitic
worms). Release of major basic protein by eosinophils causes damage to epithelial and
endothelial cells and is a major mechanism of chronic lung damage in certain pulmonary
conditions (eg, asthma, Ascaris infection).
(Choice A) Plasma cells produce antibodies that opsonize bacteria to facilitate phagocytosis.
(Choice C) Pattern recognition receptors (eg, dectin-1) on phagocytic cells allow for detection
and destruction of fungi by the innate immune system.
(Choice D) Virus-infected cells typically lack MHC class I proteins on their surface; these cells
are identified and killed by natural killer cells.
(Choice E) Macrophages secrete transforming growth factor-beta (TGF-β), which stimulates
fibroblasts to release collagen and form scar tissue.
COPD

Telegram Group Invite Link: https://t.me/joinchat/zyFOIg0j5goxY2E1


https://t.me/joinchat/zyFOIg0j5goxY2E1

Telegram Group Invite Link: https://t.me/USMLEWorldStep1

This patient's chest x-ray reveals hyperinflated lungs and a flattened diaphragm (compared
to normal), consistent with chronic obstructive pulmonary disease (COPD). COPD involves
components of chronic bronchitis and emphysema, both of which contribute to air-trapping and
hyperinflation.
The outward expanding pressure created by the chest wall and the inward collapsing pressure
created by the lungs are in equilibrium at the functional residual capacity (FRC). In patients
with COPD, decreased elasticity of the alveoli (from emphysema) results in decreased
collapsing pressure created by the lungs to expel air. Bronchial airway thickening and
obstruction (from chronic bronchitis) further impairs lung collapse by impeding the expulsion of
Telegram Group Invite Link: https://t.me/joinchat/zyFOIg0j5goxY2E1
Telegram Group Invite Link: https://t.me/USMLEWorldStep1

air. The decrease in collapsing pressure causes the chest wall to expand outward until the
expanding pressure of the chest is balanced by the collapsing pressure of the lungs. A new
pressure equilibrium is reached at a higher lung volume (ie, higher FRC) and, as a result,
residual volume and total lung capacity are also increased.

Telegram Group Invite Link: https://t.me/joinchat/zyFOIg0j5goxY2E1


https://t.me/joinchat/zyFOIg0j5goxY2E1

Telegram Group Invite Link: https://t.me/USMLEWorldStep1

(Choice A) The diffusing capacity of the lung for carbon monoxide (DLCO) largely depends on
the thickness and total surface area of the alveolar capillary membrane. In emphysema,
interalveolar wall destruction decreases the alveolar-capillary surface area, reducing
DLCO. Patients with emphysema may have decreased DLCO even when there is little
evidence of expiratory airflow obstruction on spirometry.
(Choice B) Expiratory airflow rates are reduced in COPD due to inflammatory and fibrotic
narrowing of the bronchi (chronic bronchitis) and decreased alveolar elastic recoil
(emphysema).
(Choice C) In COPD, forced vital capacity is usually decreased due to expiratory airflow
obstruction limiting the total expiratory volume.
(Choice E) Emphysema causes lung elastic recoil to decrease secondary to destruction of
interalveolar walls.
elastase

Telegram Group Invite Link: https://t.me/joinchat/zyFOIg0j5goxY2E1


Telegram Group Invite Link: https://t.me/USMLEWorldStep1

Elastase is a neutral protease contained in macrophage lysosomes and in the azurophilic


(primary) granules of neutrophils. Normally, elastase released from alveolar macrophages and
infiltrating neutrophils is balanced by the presence of serum and tissue protease
inhibitors. Neutrophil elastase is inhibited by serum alpha-1 antitrypsin, and macrophage
elastase is inhibited by tissue inhibitors of metalloproteinases. Neutrophil and macrophage
elastases can also degrade each other's (but not their own) inhibitors, augmenting their
destructive capacity when both proteases are present. Excess protease activity is a major
contributor to the development of both centriacinar and panacinar emphysema.
(Choice B) The ciliated epithelium of the lower respiratory tract sweeps foreign particles and
mucus toward the pharynx to promote mucociliary clearance.
(Choice C) Club (formerly Clara) cells are nonciliated cells found predominantly in the terminal
portions of the bronchioles. They secrete club cell secretory protein (which protects against
airway inflammation and oxidative stress) and surfactant components (which prevent
bronchiolar collapse).
(Choice D) Goblet cells secrete mucin to help with mucociliary clearance. They are present in
the epithelial lining of the trachea and bronchi but are not present in the more distal airways (ie,
terminal bronchioles, respiratory bronchioles, alveoli).
(Choice E) Type I pneumocytes constitute over 95% of the surface area of the alveoli. These
end-differentiated squamous cells are not a major source of alveolar fluid secretory products.
(Choice F) Type II pneumocytes secrete the major components of pulmonary surfactant,
including phospholipids such as dipalmitoyl phosphatidylcholine and surfactant-associated
proteins.
IV Fluid

Telegram Group Invite Link: https://t.me/joinchat/zyFOIg0j5goxY2E1


https://t.me/joinchat/zyFOIg0j5goxY2E1

Telegram Group Invite Link: https://t.me/USMLEWorldStep1

This patient with fever, cough, and radiologic evidence of consolidation in the right lower lung
has pneumonia. Her tachycardia, hypotension, and confusion (evidence of end-organ
hypoperfusion) are most likely due to septic shock, which is characterized by increased
permeability of the vascular endothelium with leakage of intravascular fluid into the
extravascular space.
In the management of septic shock, rapid restoration of intravascular volume and adequate end-
organ perfusion is critical. This is best accomplished with the administration of boluses
of isotonic crystalloid in the form of 0.9% (normal) saline or lactated Ringer solution through
large-bore, peripheral intravenous catheters. These solutions are ideal for volume resuscitation
because they have osmolarity very close to the normal osmolarity of the blood (~285 mOsm/kg
H2O), while the sodium and chloride ions help retain the fluid in the extracellular space.
Prompt initiation of appropriate empiric antibiotic therapy is also critical in the management of
septic shock.
(Choices A and C) 0.45% (half-normal) saline is hypotonic; 5% dextrose in 0.45% saline is
hypertonic initially but becomes hypotonic following infusion because the dextrose is rapidly
metabolized. These hypotonic solutions are often used at low infusion rates for patients with a
deficiency of free water (ie, hypernatremia) or for maintenance hydration. However, they are
not effective for rapid volume resuscitation because the low osmolarity causes much of the fluid
volume to shift into the intracellular space following infusion.
(Choice B) Infusion of 3% (hypertonic) saline can lead to rapid fluid-shifting from the
intracellular to the extracellular space with potentially devasting consequences (eg, osmotic
demyelination syndrome); therefore, 3% saline is not appropriate for rapid volume
resuscitation. It is appropriate for careful use in patients with severe symptomatic
hyponatremia.
(Choice D) Albumin solution is an isotonic colloid solution that can be used for rapid volume
resuscitation. However, it is less preferred due to high cost and limited availability compared to
isotonic crystalloid.
(Choice F) Sodium bicarbonate solutions can have variable tonicity and are typically used at
low infusion rates for patients with severe metabolic acidosis. These solutions are generally not
used for rapid volume resuscitation.

Telegram Group Invite Link: https://t.me/joinchat/zyFOIg0j5goxY2E1


Telegram Group Invite Link: https://t.me/USMLEWorldStep1

Brochi
obstruction

This patient's decreased breath sounds, hemithorax opacification on the right, and deviation of
the trachea toward the opacified side are suggestive of a collapsed lung due to bronchial
obstruction. Complete collapse of a lung usually occurs following obstruction of a mainstem
bronchus (eg, central lung tumors in chronic smokers). As the air trapped in the lung gradually
gets absorbed into the blood, there is loss of lung volume due to alveolar collapse (ie,
atelectasis), which causes the trachea to deviate toward the affected side. Other mediastinal
structures (eg, heart, esophagus, great vessels) may also shift in the same direction. The loss
of radiolucent air, combined with shifting of organs into the hemithorax, appears as
a completely opacified hemithorax on chest x-ray.

Telegram Group Invite Link: https://t.me/joinchat/zyFOIg0j5goxY2E1


https://t.me/joinchat/zyFOIg0j5goxY2E1

Telegram Group Invite Link: https://t.me/USMLEWorldStep1

(Choice A) Fluid in the alveolar spaces can occur with pulmonary edema. This typically
manifests as bilateral fluffy-appearing infiltrates, not unilateral lung opacification.

Telegram Group Invite Link: https://t.me/joinchat/zyFOIg0j5goxY2E1


https://t.me/joinchat/zyFOIg0j5goxY2E1

USMLE e e r m h nne s
Me est i e s: https://t.me/USMLEMe est

K p n ssr m i e s: https://t.me/USMLEK p n

USMLE i e s Step : https://t.me/USMLE Step

e ker i e s: https://t.me/USMLE e ker

r s e n i e s Step : https://t.me/USMLE r sAn e n

r s e n i e s Step K: https://t.me/USMLE r sAn e n Step K

Ph se i e s: https://t.me/USMLEPh se

Pi ri e i e s: https://t.me/USMLEPi ri e

P th m i e s: https://t.me/USMLEP th m

nA i s i h it : https://t.me/USMLE nA i s

Am ss r p: https://t.me/USMLEAm ss

Le t ri nk: https://t.me/USMLELe t ri

En p int P s: https://t.me/USMLEEn p int

USMLE MEs: https://t.me/USMLE ME

L E Me E : https://t.me/USMLE n ineMe E

USMLE r Step : https://t.me/USMLE r Step

USMLE r Step K: https://t.me/USMLE r Step K

USMLE r Step nk: https://t.me/USMLE r Step

USMLE Anki: https://t.me/USMLEAnki

KissPrep: https://t.me/USMLEKissPrep
Telegram Group Invite Link: https://t.me/USMLEWorldStep1

(Choice B) Interstitial lung disease, such as pulmonary fibrosis, would cause reticular markings
in both lungs on chest x-ray rather than complete opacification.

(Choice C) Intrapleural air accumulation (pneumothorax) would show increased lucency on the
affected side, whereas a large pleural effusion can cause complete hemithorax
opacification. Tension pneumothorax or a large pleural effusion will cause tracheal
deviation away from the affected lung because the excess air or fluid pushes against the

Telegram Group Invite Link: https://t.me/joinchat/zyFOIg0j5goxY2E1


https://t.me/joinchat/zyFOIg0j5goxY2E1

Telegram Group Invite Link: https://t.me/USMLEWorldStep1

mediastinal structures.

Telegram Group Invite Link: https://t.me/joinchat/zyFOIg0j5goxY2E1


Telegram Group Invite Link: https://t.me/USMLEWorldStep1

Telegram Group Invite Link: https://t.me/joinchat/zyFOIg0j5goxY2E1


https://t.me/joinchat/zyFOIg0j5goxY2E1

Telegram Group Invite Link: https://t.me/USMLEWorldStep1

(Choice D) Pulmonary embolism is a form of pulmonary vascular disease that usually presents
with a normal chest x-ray. Characteristic findings such as Westermark sign (area of lucency
due to reduced perfusion) or Hampton's hump (wedge-shaped opacity adjacent to the pleura)
occur less frequently.
Chest
tube

This patient most likely has empyema requiring drainage with a chest tube. The technique
involves placing the chest tube through the skin and subcutaneous fat into the 4th or 5th
intercostal space in the anterior axillary or midaxillary line. The tube traverses through
the serratus anterior muscle, intercostal (external, internal, innermost) muscles, and parietal
pleura to reach the pleural cavity.
The serratus anterior originates as multiple branches from the side of the chest along the 1st-
8th ribs and inserts along the entire length of the medial scapular border. The muscle is divided
into 3 parts (superior, intermediate, inferior) depending on the insertion site at the scapula. The
inferior part of the muscle facilitates arm elevation by pulling the lower end of the scapula
forward (scapular rotation). All 3 muscle parts can help with respiration by lifting the ribs when
the shoulder girdle is fixed.

Telegram Group Invite Link: https://t.me/joinchat/zyFOIg0j5goxY2E1


Telegram Group Invite Link: https://t.me/USMLEWorldStep1

(Choice A) The external oblique originates from the lateral ribs (5-12) and extends anteriorly
across the abdomen to insert into the linea alba, pubis, and iliac crest of the hip bones. The
muscle is usually inferior to the chest tube insertion site.

(Choice B) The infraspinatus is a rotator cuff muscle that attaches medially to the infraspinous
fossa of the scapula and laterally to the greater tubercle of the humerus. The muscle stabilizes

Telegram Group Invite Link: https://t.me/joinchat/zyFOIg0j5goxY2E1


https://t.me/joinchat/zyFOIg0j5goxY2E1

Telegram Group Invite Link: https://t.me/USMLEWorldStep1

the shoulder joint in addition to externally rotating the humerus.

(Choice C) The latissimus dorsi is a back muscle that helps with multiple shoulder movements
(eg, extension, adduction). It originates from the spinous processes of T7-L5, the 9th-12th ribs,
and the inferior angle of the scapula. The muscle inserts into the humerus and is located
posterior to the chest tube insertion site.
(Choice D) The pectoralis major arises from the anterior clavicle, sternum, costal cartilages,
and aponeurosis of the external oblique muscle. It attaches on the lateral lip of the bicipital
groove of the humerus and serves to adduct and internally rotate the humerus. The muscle is
anterior to the chest tube insertion site.

Telegram Group Invite Link: https://t.me/joinchat/zyFOIg0j5goxY2E1


Telegram Group Invite Link: https://t.me/USMLEWorldStep1

TB

Telegram Group Invite Link: https://t.me/joinchat/zyFOIg0j5goxY2E1


https://t.me/joinchat/zyFOIg0j5goxY2E1

Telegram Group Invite Link: https://t.me/USMLEWorldStep1

This patient has a productive cough, hemoptysis, fever, and weight loss with an apical cavitary
lung lesion, findings characteristic of active tuberculosis. Mycobacterium tuberculosis is an
acid-fast bacillus that replicates within the alveoli. The bacteria are phagocytized by alveolar
macrophages, which are initially unable to destroy the pathogen due to microbial adaptations
that inhibit the formation of an effective phagolysosome. However, in the weeks after the initial
infection, antigen presenting cells (eg, macrophages, dendritic cells) display mycobacterial
antigens to naïve CD4+ T lymphocytes in regional lymph nodes, which subsequently differentiate
into T helper type 1 (Th1) cells that secrete interferon-gamma.
Interferon-gamma activates macrophages, leading to the formation of fully acidified
phagolysosomes capable of destroying intracellular mycobacteria. Activated macrophages also
aggregate and differentiate into epithelioid histiocytes and multinucleated giant cells, which
surround extracellular mycobacteria within granulomas. The release of proteases, nitric
oxide, and reactive oxygen species by these cells helps contain the infection. However,
these compounds also cause extensive collateral tissue damage and can result in the
formation of cavitary lung lesions.

(Choice B) M tuberculosis produces trehalose dimycolate (cord factor), which inhibits


phagolysosome maturation and may contribute to the formation of caseating
granulomas. However, cord factor does not directly cause the tissue necrosis associated with

Telegram Group Invite Link: https://t.me/joinchat/zyFOIg0j5goxY2E1


Telegram Group Invite Link: https://t.me/USMLEWorldStep1

cavitary lesions; the release of digestive enzymes by activated macrophages lining the
granuloma drives caseation and cavitation.
(Choice C) Lobar pneumonia is characterized by the initial accumulation of an abundant
proteinaceous exudate within the alveoli, followed by hepatization of the affected lobe. Lobar
pneumonia is typically caused by pathogens such as Streptococcus pneumoniae. However,
lobar pneumonia presents acutely, not slowly, over several weeks.

(Choice D) Intraalveolar mycobacteria do not directly cause cavitary lesions. Activated


macrophages that surround the extracellular mycobacteria are primarily responsible for the
tissue damage that leads to cavitation.
(Choice E) Obliterative lower airway inflammation can occur as a feature of cryptogenic
organizing pneumonia. In this condition, inflammation causes the proliferation of granulation
tissue, which obstructs small bronchioles and leads to alveolar consolidation. However, apical
cavitary lesions and hemoptysis are uncommon.

Telegram Group Invite Link: https://t.me/joinchat/zyFOIg0j5goxY2E1


https://t.me/joinchat/zyFOIg0j5goxY2E1

Telegram Group Invite Link: https://t.me/USMLEWorldStep1

Mast
cell

Telegram Group Invite Link: https://t.me/joinchat/zyFOIg0j5goxY2E1


Telegram Group Invite Link: https://t.me/USMLEWorldStep1

Acute bronchial constriction during an asthma exacerbation begins with mast cell
activation secondary to allergen interaction with IgE antibodies or nonimmune triggers (eg,
exercise, cold exposure, chemical irritants). This results in the release of chemical mediators
that cause immediate bronchial constriction, bronchial wall edema, and increased mucus
production (early phase reaction). These mediators recruit additional inflammatory cells (eg,
eosinophils, basophils, neutrophils, T lymphocytes) that lead to bronchial obstruction, which
occurs several hours after the triggering event (late phase reaction).
Cromolyn and nedocromil inhibit mast cell degranulation and prevent release of preformed
chemical mediators. These medications do not influence bronchial constriction directly;
therefore, they are typically used to prevent acute attacks, rather than to treat acute
exacerbations. Even though these drugs are less efficacious than inhaled glucocorticoids, they
are very effective prophylactic agents for patients with seasonal symptoms, aspirin
hypersensitivity, and exercise-induced asthma.
(Choice A) The anti-IgE antibody, omalizumab, inhibits IgE binding to mast cells, preventing
mast cell degranulation. Omalizumab is used in some patients with severe, persistent asthma
to lower IgE levels and reduce allergen-induced bronchial constriction.
(Choice B) Cleavage of membrane phospholipids by the enzyme phospholipase A2 leads to
the formation of arachidonic acid, which is in turn converted into prostaglandins and
leukotrienes. By inhibiting phospholipase A2 synthesis, glucocorticoids inhibit the release of

Telegram Group Invite Link: https://t.me/joinchat/zyFOIg0j5goxY2E1


https://t.me/joinchat/zyFOIg0j5goxY2E1

Telegram Group Invite Link: https://t.me/USMLEWorldStep1

these downstream mediators, decreasing airway hyperresponsiveness and inflammation.

(Choices C and D) Zileuton is a selective inhibitor of the lipoxygenase pathway that leads to
decreased formation of leukotrienes. Zafirlukast and montelukast are leukotriene receptor
antagonists. These agents are typically used for chronic asthma prophylaxis.
(Choice F) Antihistamines are used to prevent and blunt an acute allergic response (as seen in
allergic rhinitis) but have not been shown to improve bronchoconstriction in asthma.

Telegram Group Invite Link: https://t.me/joinchat/zyFOIg0j5goxY2E1


Telegram Group Invite Link: https://t.me/USMLEWorldStep1

ARDS

This patient has a urinary tract infection complicated by fever and hemodynamic instability
consistent with sepsis. Her rapid-onset respiratory failure is suggestive of acute respiratory
distress syndrome (ARDS). ARDS is characterized by bilateral pulmonary infiltrates and
hypoxemia in the absence of heart failure. It can occur due to direct pulmonary trauma (eg,
pulmonary contusions, inhaled irritants) or indirect nonpulmonary insults (eg, sepsis, burns,
pancreatitis) that result in pulmonary epithelial and/or endothelial injury.
The 3 phases of ARDS follow the disease's progression:
• Exudative phase: Inflammatory cytokines (eg, tumor necrosis factor, IL-1, IL-6) activate
the pulmonary endothelium and recruit neutrophils to the lung tissue. Resultant
endothelial damage leads to increased capillary permeability and leakage of protein-
rich fluid into the alveolar space. Organization of the edema and cellular debris leads
to the formation of hyaline membranes.
• Proliferative phase: One to two weeks later, endothelial cells, pneumocytes, and
fibroblasts proliferate in attempts to repair the damaged lung; collagen is deposited and
scarring may occur. Edema is reabsorbed.
• Fibrotic phase: In a minority of patients, excessive collagen deposition leads to
irreversible pulmonary fibrosis and pulmonary hypertension.

Telegram Group Invite Link: https://t.me/joinchat/zyFOIg0j5goxY2E1


https://t.me/joinchat/zyFOIg0j5goxY2E1

Telegram Group Invite Link: https://t.me/USMLEWorldStep1

Telegram Group Invite Link: https://t.me/joinchat/zyFOIg0j5goxY2E1


Telegram Group Invite Link: https://t.me/USMLEWorldStep1

(Choice B) Necrotizing inflammation and pulmonary hemorrhage are commonly associated


with granulomatosis with polyangiitis, which involves both the lungs and the kidneys. However,
this condition normally presents subacutely with cough, hemoptysis, and nephritic syndrome;
this patient's acute respiratory decline in the setting of sepsis is more consistent with ARDS.
(Choice C) Noncaseating granulomas are found in sarcoidosis, which typically presents more
chronically with cough, skin findings, or uveitis. Hemodynamic instability and acute respiratory
failure would be unexpected.
(Choice D) Mucus plugging typically occurs in patients with underlying lung disease (eg, cystic
fibrosis, chronic obstructive pulmonary disease). Mucus plugs can cause hypoxemia but would
not be expected to cause hypotension and fevers, and acute respiratory deterioration in the
setting of sepsis is more likely due to ARDS.
(Choice E) Wedge-shaped areas of hemorrhagic necrosis can be seen with pulmonary
embolism, which can cause acute respiratory failure, hypotension, and tachycardia. However,

Telegram Group Invite Link: https://t.me/joinchat/zyFOIg0j5goxY2E1


https://t.me/joinchat/zyFOIg0j5goxY2E1

Telegram Group Invite Link: https://t.me/USMLEWorldStep1

patients often have chest pain, and in general, decompensation occurs suddenly (not
progressively over a course of hours). In addition, urinary symptoms, confusion, and high
fevers are unexpected in this condition.
Ketamine
This hemodynamically unstable patient has severe bronchospasm and impending respiratory
failure and requires intubation. Prior to intubation, anesthesia must be induced. Three
medications are commonly used because of their rapid onset of action and short duration of
effect.

• Propofol: A highly lipophilic GABA agonist that may be used for long-term
sedation. Disadvantages include vasodilation, which can result in hypotension and an
increase in serum triglycerides and lipase.

• Etomidate: A GABA agonist that has the advantage of being the


most hemodynamically neutral. It does not cause changes in heart rate, blood
pressure, or cardiac output. However, it inhibits cortisol synthesis, which can lead to
(reversible) adrenocortical suppression. Because of this, it is often avoided in patients
with septic shock, and it should not be used as maintenance of sedation after induction.

• Ketamine: An N-methyl-D-aspartate (NMDA) antagonist that is similar to PCP;


it preserves the respiratory drive during induction of anesthesia. In contrast to
propofol and etomidate, it also provides an analgesic effect. Ketamine stimulates the
release of catecholamines (ie, sympathomimetic), which can
cause bronchodilation but also increase the heart rate, myocardial contractility, and
cerebral blood flow, potentially putting the patient at risk for cardiovascular events or
increased intracranial pressure.

In this patient with an acute asthma attack, ketamine was likely chosen because it stimulates
release of catecholamines that increase sympathomimetic activity and may improve
bronchospasm.
(Choice A) Propofol and etomidate are both GABA agonists, but ketamine works at the NMDA
receptor.
(Choice B) Both ketamine and etomidate are much less lipophilic than propofol.
(Choice C) Both ketamine and etomidate are metabolized primarily by the liver. Propofol is
also metabolized by the liver, but the effect is terminated by its redistribution rather than
metabolism.
(Choice D) Neuromuscular blockade is often used to facilitate intubation. None of the three
commonly used induction anesthetics cause significant neuromuscular blockade, so an
additional medication (eg, succinylcholine, rocuronium) is used if needed.
High altitude
The reduced partial pressure of inspired oxygen at high altitude leads to hypoxemia (ie,
reduced partial pressure of arterial oxygen [PaO2]). Hyperventilation is the most immediate
and important physiologic adjustment to high altitude; peripheral chemoreceptors stimulate
increased minute ventilation (via both increased tidal volume and respiratory rate) to help
increase PaO2 and minimize the symptoms of hypoxemia (eg, headache, dyspnea).

Telegram Group Invite Link: https://t.me/joinchat/zyFOIg0j5goxY2E1


Telegram Group Invite Link: https://t.me/USMLEWorldStep1

In older individuals (eg, age >65), the body is less able to compensate for the hypoxemia at
high altitude due to a number of age-related changes to the respiratory system. These changes
include the following:
• Decreased chest wall compliance: Calcification of the intercostal joints and tendon
insertion sites, as well as degeneration of the spine (eg, kyphosis), reduces chest wall
expansion and limits the potential increase in tidal volume.
Decreased alveolar elastic recoil: Degeneration of elastin causes increased alveolar
compliance with alveolar enlargement and increased air trapping. This increases residual
volume and decreases forced vital capacity, further limiting the achievable increase in tidal
volume (Choices D and E).

• Increased alveolar-arterial O2 gradient: The alveolar enlargement increases


ventilation-perfusion mismatch and decreases the efficiency of O2 exchange in 2
ways. First, it decreases the percentage of alveolar surface area in contact with alveolar
capillaries, effectively increasing dead space (Choice B). Second, the increased air
trapping decreases ventilation of highly perfused alveoli at the base of the lungs.

Telegram Group Invite Link: https://t.me/joinchat/zyFOIg0j5goxY2E1


https://t.me/joinchat/zyFOIg0j5goxY2E1

Telegram Group Invite Link: https://t.me/USMLEWorldStep1

In this elderly patient, the body's ability to increase PaO2 at high altitude has likely been
reduced, and, given her underlying coronary artery disease, she is likely experiencing angina
(eg, exertional chest pain) due to inadequate O2 supply to the myocardium.
(Choice C) Aging appears to have minimal effect on chemoreceptor responsiveness; therefore,
respiratory drive is appropriately increased at high altitude. Age-related changes that reduce
the mechanical efficiency of the respiratory system (ie, ability to respond to increased
respiratory drive) are primarily responsible for reduced compensation for hypoxemia at high
altitude.

Telegram Group Invite Link: https://t.me/joinchat/zyFOIg0j5goxY2E1


Telegram Group Invite Link: https://t.me/USMLEWorldStep1

asthma

This patient's shortness of breath and wheezing are suggestive of asthma, and his rhinorrhea
and watery eyes are suggestive of allergic rhinitis. Both of these conditions are examples
of type I hypersensitivity, which involves the triggering of an allergic response via the binding
of previously recognized antigen to IgE antibodies on mast cells. Many type I hypersensitivity
reactions are composed of both an early and late phase.
Histamine is housed in preformed granules of unactivated mast cells and plays an important
role in the early phase of type I hypersensitivity. Upon activation, mast cells rapidly release
histamine via degranulation, making histamine the first chemical mediator to take effect. Once
released, histamine triggers smooth muscle contraction leading to bronchoconstriction,
increases vascular permeability leading to edema, and increases mucus secretion from

Telegram Group Invite Link: https://t.me/joinchat/zyFOIg0j5goxY2E1


https://t.me/joinchat/zyFOIg0j5goxY2E1

Telegram Group Invite Link: https://t.me/USMLEWorldStep1

glandular tissue.

Telegram Group Invite Link: https://t.me/joinchat/zyFOIg0j5goxY2E1


Telegram Group Invite Link: https://t.me/USMLEWorldStep1

(Choice B) Leukotriene D4, along with the other cysteinyl leukotrienes (C4 and E4), are not
preformed but require synthesis via the 5-lipoxygenase pathway of arachidonic acid
metabolism. Therefore, these mediators are released by mast cells later than histamine. Once
released, the cysteinyl leukotrienes are potent mediators of vasoconstriction,
bronchoconstriction, and increased vascular permeability.

(Choice C) Major basic protein is released from eosinophils in the late stage of a type I
hypersensitivity reaction and causes localized tissue damage.
(Choice D) Platelet-activating factor is a secondary inflammatory mediator that must be
synthesized from phospholipid prior to being released from activated mast cells and
basophils. Once released, it stimulates bronchospasm and increased vascular permeability.
(Choice E) Prostaglandin D2 is synthesized via the cyclooxygenase pathway of arachidonic
acid metabolism prior to being released from mast cells. Once released, it causes
bronchospasm and vasodilation.

Telegram Group Invite Link: https://t.me/joinchat/zyFOIg0j5goxY2E1


https://t.me/joinchat/zyFOIg0j5goxY2E1

Telegram Group Invite Link: https://t.me/USMLEWorldStep1

Plseural
effusion

In the normal state, the pleural space contains a small amount of fluid (eg, 5-10 mL) that
undergoes constant turnover, with the rate of inflow equivalent to the rate of outflow. The rate
of inflow is determined by vascular hydrostatic pressure, vascular oncotic pressure, and
vascular membrane permeability (ie, the Starling equation), whereas the rate of outflow is
determined by the drainage capacity of the parietal pleural lymphatics. A pleural
effusion results from an increased rate of pleural fluid inflow, a decreased rate of pleural fluid
outflow, or a combination of the two.
This patient with progressive dyspnea on exertion, nonproductive cough, bilateral crackles on
lung auscultation, and a right-sided pleural effusion most likely has decompensated heart
failure (chronic, poorly controlled hypertension is a common cause). Pleural effusion in
decompensated heart failure is primarily driven by backward transmission of pressure from the
failing left ventricle to the pulmonary circulation, resulting in increased pulmonary capillary
hydrostatic pressure and an increased rate of fluid inflow to the pleural space. Vascular
permeability remains normal, as does vascular oncotic pressure (which is mostly determined by
serum albumin concentration). Outflow through the parietal pleural lymphatics increases in
response to the increased fluid inflow, but it is unable to keep up, resulting in development of

Telegram Group Invite Link: https://t.me/joinchat/zyFOIg0j5goxY2E1


Telegram Group Invite Link: https://t.me/USMLEWorldStep1

pleural effusion.

(Choice A) Lung malignancy can cause pleural effusion via both an increased rate of inflow
due to an inflammatory increase in vascular membrane permeability and a decreased rate of
outflow due to obstruction of parietal pleural lymphatics.
(Choice B) All 4 of these changes encourage the development of pleural effusion and multiple
disturbances would be required for such changes to take place (eg, lung malignancy [increased
vascular permeability and decreased lymphatic flow] in the setting of both hypoalbuminemia
[decreased oncotic pressure] and heart failure [increased hydrostatic pressure]).
(Choice C) Hypoalbuminemia (eg, due to nephrotic syndrome or malnutrition) causes pleural
effusion due to an increased rate of pleural fluid inflow from reduced vascular oncotic
pressure. As with decompensated heart failure, lymphatic flow increases but is unable to keep
up with the increased inflow.
(Choice E) All 4 of these changes would discourage the development of pleural effusion.

Telegram Group Invite Link: https://t.me/joinchat/zyFOIg0j5goxY2E1


https://t.me/joinchat/zyFOIg0j5goxY2E1

Telegram Group Invite Link: https://t.me/USMLEWorldStep1

sepsis

This patient has sepsis due to an infected diabetic foot ulcer with surrounding cellulitis. Sepsis
is a florid host inflammatory response to infection that can lead to multiple organ system
dysfunction.
Septic organ dysfunction is driven mainly by poor tissue oxygen use. This cellular dysoxia is
caused by 3 major mechanisms:

• Bacterial components (eg, endotoxin) and acute phase cytokines (eg, IL-1-beta) trigger
production of free radicals that damage mitochondria and interfere with the electron
transport chain. Immediate postmortem analysis of patients with sepsis reveals
surprisingly minimal tissue necrosis but, often, extensive mitochondrial damage. This
mitochondrial dysfunction leads to decreased oxidative phosphorylation with loss of
ATP production. A compensatory bioenergetic shift toward glycolysis often results
in lactic acidosis.

• Widespread microcirculatory failure with vasodilation causes blood to shunt rapidly


through organs, decreasing the opportunity for oxygen extraction.

• Increased capillary permeability causes tissue edema (third spacing), which increases
the diffusion distance for oxygen to reach mitochondria of target cells.

Because oxygen use decreases, the transorgan arteriovenous oxygen gradient is small;
therefore, oxygen saturation of central venous blood (ScvO2) (superior vena cava) –
returning from the periphery – increases. Central venous catheters are often placed
therapeutically to deliver medications; they can be used diagnostically to differentiate septic (↑

Telegram Group Invite Link: https://t.me/joinchat/zyFOIg0j5goxY2E1


Telegram Group Invite Link: https://t.me/USMLEWorldStep1

ScvO2) from cardiogenic (↓ ScvO2) shock.

(Choice A) Cardiac output (oxygen delivery) is typically increased in sepsis due to systemic
vasodilation (warm extremities, wide pulse pressure). Decreased cardiac output is the hallmark
of severe heart failure (cold extremities, narrow pulse pressure), and ScvO2 decreases with
heart failure because peripheral oxygen consumption outstrips its delivery.
(Choice C) Sepsis is associated with microvascular (rather than macrovascular) thrombosis
due to endothelial injury and focal or disseminated intravascular coagulation, further impairing
oxygenation. Widespread large-vessel thrombosis and infarction are seen in conditions such as
catastrophic antiphospholipid-antibody syndrome.
(Choice D) Sepsis can induce capillary permeability in the lung, leading to pulmonary edema
with resultant arterial hypoxemia (acute respiratory distress syndrome). However, this patient's
pulmonary function and oxygenation are intact, as evidenced by normal lung examination (no
crackles) and arterial oxygen saturation.
(Choice E) Hemoglobin oxygen unloading to peripheral tissues is facilitated by sepsis-induced
(lactic) acidosis and fever, which lead to rightward shifting of the oxyhemoglobin dissociation
curve (ie, weaker oxygen-hemoglobin binding).

Telegram Group Invite Link: https://t.me/joinchat/zyFOIg0j5goxY2E1


https://t.me/joinchat/zyFOIg0j5goxY2E1

Telegram Group Invite Link: https://t.me/USMLEWorldStep1

ARDS

This patient with pancreatitis and subsequent respiratory failure likely has acute respiratory
distress syndrome (ARDS). Pancreatitis is a major risk factor for ARDS as it results in the
release of large amounts of inflammatory cytokines and pancreatic enzymes into the circulation,
which leads to infiltration of neutrophils into the pulmonary interstitium and alveolar
spaces. Diffuse injury to the alveolar epithelium and pulmonary microvascular endothelium
results in a leaky alveolocapillary membrane and significant pulmonary edema.
ARDS is typically characterized by progressive hypoxemia refractory to oxygen therapy and
diffuse interstitial edema in the absence of cardiogenic causes. During the first 1-6 days,
interstitial and intraalveolar edema, inflammation, and fibrin deposition cause the alveoli to
become lined with waxy hyaline membranes. These membranes consist of fibrin exudate and
inspissated protein-rich edema fluid mixed with the remnants of necrotic epithelial cells.
(Choice B) If this patient's rapid-onset dyspnea were due to cardiogenic pulmonary edema, as
can occur in decompensated heart failure, or if the patient had chronic heart failure due to
alcohol use, chest x-ray would likely reveal cardiomegaly.

Telegram Group Invite Link: https://t.me/joinchat/zyFOIg0j5goxY2E1


Telegram Group Invite Link: https://t.me/USMLEWorldStep1

(Choice C) This patient is at risk of aspiration; however, aspiration pneumonia due to


anaerobic bacteria commonly presents more indolently and is more likely to show an abscess or
empyema than diffuse patchy opacities on chest x-ray.
(Choice D) Although disseminated intravascular coagulation and ARDS can result in alveolar
hemorrhage, the absence of hemoptysis makes this choice less likely.
(Choice E) Lung hyperinflation may be seen in emphysema, which is a chronic, destructive
enlargement of the air spaces distal to the terminal bronchiole.
Lung abscess
Lung abscess is a necrotic infection of the pulmonary parenchyma that usually presents with
several days of fever, cough productive of copious sputum (often foul-smelling), and chest x-
ray evidence of cavitation with air-fluid level. Most cases are due to aspiration of anaerobic
bacteria from the oropharynx, but lung abscess can also develop in the setting of untreated
pneumonia.

Neutrophils are the key player in the formation of lung abscess. They are recruited from the
systemic circulation by chemokines and are subsequently activated by microbial molecules (eg,
lipopolysaccharide, peptidoglycan, bacterial DNA) and opsonizing factors (eg, IgG, complement)
to phagocytize and destroy the pathogenic bacteria. Activated neutrophils also release
cytotoxic granules (lysosomes) containing myeloperoxidase and other digestive enzymes that
destroy extracellular bacteria and recruit additional immune components to the area. However,
these enzymes also cause significant damage to the pulmonary parenchyma and can result

Telegram Group Invite Link: https://t.me/joinchat/zyFOIg0j5goxY2E1


https://t.me/joinchat/zyFOIg0j5goxY2E1

Telegram Group Invite Link: https://t.me/USMLEWorldStep1

in liquefying necrosis of lung tissue and (potentially) lung abscess.

(Choices A and B) Dendritic and other antigen-presenting cells release interleukin-12, which
stimulates differentiation of Th1 helper cells and production of interferon-gamma by T-
cells. Interferon-gamma activates macrophages, leading to the development of mature
phagolysosomes capable of destroying phagocytosed bacteria. Macrophages are particularly
important for the elimination of certain intracellular infections such as Mycobacterium
tuberculosis. Although tuberculosis can be associated with pulmonary cavitation, the cavitations

Telegram Group Invite Link: https://t.me/joinchat/zyFOIg0j5goxY2E1


Telegram Group Invite Link: https://t.me/USMLEWorldStep1

usually form in the upper lobes and generally take months to develop.

(Choice D) Major basic protein is the predominant component of eosinophilic granules and
plays a crucial role in the elimination of parasites. Lung abscess formation is primarily mediated
by neutrophils, not eosinophils.
(Choice E) Transforming growth factor-beta is secreted by inflammatory cells and results in the
recruitment of fibroblasts and the deposition of connective tissue. This cytokine contributes to
scar formation after injury and plays a role in the fibrosis seen with chronic inflammation.

Telegram Group Invite Link: https://t.me/joinchat/zyFOIg0j5goxY2E1


https://t.me/joinchat/zyFOIg0j5goxY2E1

Telegram Group Invite Link: https://t.me/USMLEWorldStep1

PFTs

This young woman with progressively worsening dyspnea, history of cigarette smoking, and
decreased serum levels of alpha-1 antitrypsin (AAT) likely has AAT deficiency and
associated early-onset emphysema. Emphysema from any cause leads to increased total
lung capacity as the lungs hyperinflate, decreased FEV1/FVC ratio, and decreased
DLCO (diffusing capacity for carbon monoxide) due to destruction of alveoli and adjoining
capillary beds.
AAT is a protein that inhibits several proteolytic enzymes released by inflammatory cells
(particularly neutrophil elastase), thereby reducing tissue damage from the inflammatory
cascade. Even in the absence of smoking, patients with AAT deficiency typically develop early-
onset emphysema (prior to age 50) due to the body's inability to inhibit tissue
proteolysis. Cigarette smoking and the associated pulmonary inflammation that it induces can
greatly accelerate the development of emphysema in these patients. The emphysema in AAT

Telegram Group Invite Link: https://t.me/joinchat/zyFOIg0j5goxY2E1


Telegram Group Invite Link: https://t.me/USMLEWorldStep1

deficiency typically develops in a panacinar pattern.

(Choice A) Obstructive lung disease causes increased lung volumes (residual volume and total
lung capacity) and decreased FEV1/FVC ratio, but the DLCO can vary based on
etiology. Asthma is an obstructive disease that can have high DLCO due to increased
pulmonary capillary blood volume and intact alveolar-arterial membranes (lack of
emphysematous destruction).
(Choice C) The pattern of decreased FEV1/FVC ratio, decreased lung volume, and decreased
DLCO reflects combined restrictive and obstructive defects, which can be due to multiple
diseases present simultaneously or diseases such as pneumoconiosis (eg, silicosis, asbestosis)
and sarcoidosis.

Telegram Group Invite Link: https://t.me/joinchat/zyFOIg0j5goxY2E1


https://t.me/joinchat/zyFOIg0j5goxY2E1

Telegram Group Invite Link: https://t.me/USMLEWorldStep1

(Choice D) Reduced FEV1/FVC ratio and normal DLCO may occur in asthma or in chronic
bronchitis-predominant chronic obstructive pulmonary disease.
(Choices E and F) Restrictive lung disease can be categorized according to intrinsic (eg,
interstitial lung disease) and extrinsic (eg, obesity, neuromuscular disease) etiologies, both of
which demonstrate a pattern of reduced lung volumes with normal or increased FEV1/FVC ratio
(although both the FEV1 and FVC are reduced individually). The DLCO helps distinguish
between the 2 forms: Intrinsic restrictive lung disease demonstrates reduced DLCO, whereas
extrinsic disease has normal DLCO.
PFTs

Pulmonary function testing (PFT) usually involves spirometry, during which the patient
completely inhales (to reach total lung capacity) and then forcefully exhales as much air as
possible as quickly as possible. The graph in this question depicts the measured results
following this procedure. The volume of air expelled during the first second of exhalation is
the forced expiratory volume in 1 second (FEV1), and the total volume of air expelled
represents the forced vital capacity (FVC).
The black curve on the graph depicts normal PFT results with an FEV1 of 4 liters, an FVC of 5
liters, and an FEV1/FVC ratio of 80%. The patient's red curve shows a reduced FEV1 of 3
liters, a reduced FVC of 3.5 liters, and a slightly increased FEV1/FVC ratio of 86%. These
findings are consistent with a restrictive pattern; therefore, the patient's dyspnea is most likely
due to interstitial lung disease. Neuromuscular weakness and obesity hypoventilation

Telegram Group Invite Link: https://t.me/joinchat/zyFOIg0j5goxY2E1


Telegram Group Invite Link: https://t.me/USMLEWorldStep1

syndrome also demonstrate a restrictive pattern on PFT.

(Choices A, B, and D) Asthma and chronic obstructive pulmonary disease (COPD) both
demonstrate an obstructive pattern on PFT with reduced FEV1 (eg, 2 liters), normal or reduced
FVC (eg, 4.5 liters), and reduced FEV1/FVC ratio (eg, 45%). Bronchodilator testing may be
helpful in differentiating asthma and COPD as airway obstruction due to asthma typically has a
much higher degree of reversibility than that due to COPD. However, bronchodilator testing

Telegram Group Invite Link: https://t.me/joinchat/zyFOIg0j5goxY2E1


https://t.me/joinchat/zyFOIg0j5goxY2E1

Telegram Group Invite Link: https://t.me/USMLEWorldStep1

does not play a role in evaluating restrictive lung disease.

Telegram Group Invite Link: https://t.me/joinchat/zyFOIg0j5goxY2E1


Telegram Group Invite Link: https://t.me/USMLEWorldStep1

Cystic
fibrosis

Chronic cough and recurrent sinusitis in a young Caucasian patient should raise suspicion
for cystic fibrosis (CF). The diagnosis of CF typically is based on elevated sweat chloride
concentrations, characteristic clinical findings (recurrent sinopulmonary infections,
pancreatic insufficiency), and/or a positive family history. However, patients with mild mutations
of the CF transmembrane conductance regulator (CFTR) gene may have normal sweat
testing. In these cases, a useful diagnostic adjunct involves measuring the nasal transepithelial
potential difference.
In intestinal and respiratory epithelia, the CFTR channel secretes chloride ions into the lumen
and also has a tonic inhibitory effect on the opening of the epithelial sodium channel (ENaC),
which decreases sodium reabsorption into the cell. This high luminal salt content helps retain
water in the lumen, forming well-hydrated mucus. During the nasal transepithelial potential
difference test, a saline solution is applied to the nose. Because patients with CF
have increased sodium absorption via the ENaC, sodium is absorbed intracellularly but
chloride in the saline solution is retained in the lumen. The higher relative amounts of
negatively charged chloride on the epithelial surface result in a more negative transepithelial
voltage difference.
CFTR channel functioning is reversed in sweat ducts compared with that in respiratory and
intestinal glands. CFTR reduces the salt content of sweat by reabsorbing luminal chloride and
stimulating ENaC to increase sodium absorption from the lumen into the cells. CFTR mutations

Telegram Group Invite Link: https://t.me/joinchat/zyFOIg0j5goxY2E1


https://t.me/joinchat/zyFOIg0j5goxY2E1

Telegram Group Invite Link: https://t.me/USMLEWorldStep1

therefore result in the production of sweat with high chloride and sodium content.

(Choice A) CFTR mutations lower the rate of bicarbonate secretion in the pancreatic ducts,
promoting mucin precipitation and the formation of intraductal concretions that cause exocrine
pancreatic insufficiency. However, this effect is not related to the nasal transepithelial potential
difference test.
(Choice B) CFTR mutations impair the passive transport of chloride along its concentration
gradient, decreasing chloride secretion by respiratory epithelial cells.
(Choice C) The water content of mucus is decreased (not increased) in patients with CF,
resulting in dehydrated mucus that predisposes to the formation of mucus plugs and chronic
sinopulmonary infections.
(Choice E) CFTR mutations do not significantly alter transmembrane potassium transport.

Succinylcholine
Succinylcholine is a depolarizing neuromuscular blocking agent used to induce skeletal
relaxation during intubation and surgery. It acts as a competitive agonist of nicotinic
acetylcholine receptors of the motor endplate, where it induces persistent depolarization,
leading to endplate desensitization and skeletal muscle paralysis. Succinylcholine is rapidly
metabolized by plasma pseudocholinesterase; only around 10% of the administered dose
reaches the neuromuscular junction, where it typically has a duration of action of <10 minutes.
This patient, who developed prolonged muscle weakness (eg, failure of spontaneous
respiration and voluntary muscle activity) after receiving succinylcholine, most likely
has pseudocholinesterase deficiency, an autosomal recessive disorder caused by a genetic
polymorphism in the BCHE gene. Patients are unable to metabolize succinylcholine, leading
to a large amount of the drug reaching the neuromuscular junction and prolonged
paralysis. Heterozygotes may experience double the normal duration of paralysis, and
Telegram Group Invite Link: https://t.me/joinchat/zyFOIg0j5goxY2E1
Telegram Group Invite Link: https://t.me/USMLEWorldStep1

homozygotes can have persistent paralysis for several hours. Patients with
pseudocholinesterase deficiency also experience prolonged effects of mivacurium (a
nondepolarizing neuromuscular blocker) and cocaine.
(Choices A and D) Succinylcholine is metabolized by plasma pseudocholinesterase, not the
liver or kidneys. Because pseudocholinesterase is produced in the liver, patients with severe
liver disease may have prolonged drug effects due to impaired synthetic function. However,
paralysis would not be expected to last for hours.
(Choice B) Propofol is highly lipid soluble and has a very short duration of action (seconds)
because of rapid redistribution from the brain, where it activates the GABA-A receptor, to other
tissues. Propofol is a hypnotic agent; it does not cause muscle paralysis.
(Choice E) Patients with myasthenia gravis (ie, autoantibodies to the acetylcholine receptors of
the neuromuscular junction) have fewer active acetylcholine receptors and are therefore highly
resistant to succinylcholine. Paralysis can occur with high doses but is unpredictable. Use is
usually avoided in this population.
Reids
index

This patient's dyspnea, wheezing, and productive cough in the setting of heavy smoking are
consistent with chronic obstructive pulmonary disease (COPD). Patients with COPD often
have components of both chronic bronchitis and emphysema. Chronic bronchitis is
characterized by increased mucus secretion and bronchial wall thickening with consequent

Telegram Group Invite Link: https://t.me/joinchat/zyFOIg0j5goxY2E1


https://t.me/joinchat/zyFOIg0j5goxY2E1

USMLE e e r m h nne s
Me est i e s: https://t.me/USMLEMe est

K p n ssr m i e s: https://t.me/USMLEK p n

USMLE i e s Step : https://t.me/USMLE Step

e ker i e s: https://t.me/USMLE e ker

r s e n i e s Step : https://t.me/USMLE r sAn e n

r s e n i e s Step K: https://t.me/USMLE r sAn e n Step K

Ph se i e s: https://t.me/USMLEPh se

Pi ri e i e s: https://t.me/USMLEPi ri e

P th m i e s: https://t.me/USMLEP th m

nA i s i h it : https://t.me/USMLE nA i s

Am ss r p: https://t.me/USMLEAm ss

Le t ri nk: https://t.me/USMLELe t ri

En p int P s: https://t.me/USMLEEn p int

USMLE MEs: https://t.me/USMLE ME

L E Me E : https://t.me/USMLE n ineMe E

USMLE r Step : https://t.me/USMLE r Step

USMLE r Step K: https://t.me/USMLE r Step K

USMLE r Step nk: https://t.me/USMLE r Step

USMLE Anki: https://t.me/USMLEAnki

KissPrep: https://t.me/USMLEKissPrep
https://t.me/joinchat/zyFOIg0j5goxY2E1

Telegram Group Invite Link: https://t.me/USMLEWorldStep1

narrowing of the bronchial lumen. Bronchial gland hyperplasia in the submucosa is the major
contributor to bronchial wall thickening. The severity of this change can be measured by
the Reid index.
The Reid index is a pathologic tool that measures the ratio of the thickness of the
submucosal glands to the thickness of the bronchial wall between the epithelial basement
membrane and the bronchial cartilage. A normal Reid index is 0.4. Higher values correlate
with increased duration and severity of chronic bronchitis.
Myasthenia
gravis

This patient has hypercapnic and hypoxic respiratory failure (low pH, high CO2, low O2)
indicating global hypoventilation. In association with the ptosis, bulbar weakness, and low
forced vital capacity, this presentation suggests myasthenic crisis (severe weakness and
respiratory depression due to an exacerbation of myasthenia gravis [MG]).
MG is characterized by autoantibodies against nicotinic acetylcholine receptors on the
postsynaptic membrane of the neuromuscular junction, resulting in receptor degradation. This
reduces the sensitivity of the postsynaptic membrane to acetylcholine stimulation, leading to
reduced muscular response despite normal acetylcholine release. Muscle weakness worsens

Telegram Group Invite Link: https://t.me/joinchat/zyFOIg0j5goxY2E1


Telegram Group Invite Link: https://t.me/USMLEWorldStep1

with repetition as acetylcholine stores within the presynaptic nerve terminal become
progressively depleted.
Patients with MG typically have extraocular (eg, ptosis, diplopia), bulbar (eg, dysphonia,
difficulty chewing), and facial (eg, myasthenic snarl) weakness. In addition, neck and proximal
muscle weakness may occur, and in severe cases the respiratory muscles may be affected,
leading to respiratory failure (as in this patient). Acetylcholinesterase inhibitors (eg,
pyridostigmine, neostigmine) are used for symptomatic treatment, and withdrawal can trigger a
myasthenic crisis.
(Choice A) Impairment of the respiratory control centers in the brain stem causes
hypoventilation during sleep (Ondine curse); however, voluntary breathing is unaffected.
(Choice C) Pulmonary embolism can result in dyspnea and gas exchange abnormalities (eg,
hypoxia). However, significant muscular weakness (eg, ptosis, nasal speech, decreased vital
capacity) would be unexpected.
(Choices D and E) Patients with diseases involving the terminal airways and alveoli (eg,
pulmonary fibrosis, emphysema) and large airways (eg, asthma, chronic bronchitis) can present
with respiratory failure subsequent to muscle fatigue. However, these diseases are not
associated with non-respiratory muscle weakness (eg, dysphonia, bulbar weakness).
Idiopathic pulmonary fibrosis
The described histopathologic findings are consistent with idiopathic pulmonary
fibrosis (IPF). Microscopic findings are heterogeneous and include:

• Patchy areas of interstitial fibrosis with chronic interstitial inflammation intermixed with
normal lung
• Early lesions consist of fibroblastic foci that become increasingly collagenous with time
• Honeycomb pattern with fibrotic walls and cystic spaces lined by bronchiolar epithelium
• Fibrosis most prominent in the subpleural and perilobular regions

Importantly, there should also be no other findings consistent with another disease process (eg,
granulomas).
The precise etiology of IPF is unclear, but risk factors include cigarette smoking, environmental
pollutants, chronic aspiration, older age, and certain genetic factors (eg, telomerase
mutations). Patients typically have dyspnea, nonproductive cough, finger clubbing, and
inspiratory crackles. The onset is insidious, prognosis is poor, and no curative treatment exists.

Telegram Group Invite Link: https://t.me/joinchat/zyFOIg0j5goxY2E1


https://t.me/joinchat/zyFOIg0j5goxY2E1

Telegram Group Invite Link: https://t.me/USMLEWorldStep1

Telegram Group Invite Link: https://t.me/joinchat/zyFOIg0j5goxY2E1


Telegram Group Invite Link: https://t.me/USMLEWorldStep1

(Choice A) Alpha-1 antitrypsin deficiency causes alveolar wall destruction, resulting in


panacinar emphysema. Patients tend to present at a young age with dyspnea, productive
cough, and wheezing. Emphysematous changes (eg, bullae) are seen predominantly at the
lung bases, and histologic findings include large alveoli with thin septa.
(Choice B) Asbestosis usually occurs in patients with occupational exposures (eg,
shipbuilding). Microscopy may show fibrosis and honeycombing, but asbestosis can be
differentiated from IPF by the presence of asbestos bodies (brown, beaded rods coated with
iron-containing material) and ferruginous bodies (inorganic particles with a similar ferrous
material). Pleural plaques are also common.
(Choice C) Chronic bronchitis causes significant sputum production over a prolonged period
and is often related to smoking. Histologic findings include mild lymphocytic infiltrates and
mucous gland hyperplasia with mucus-filled bronchioles.
(Choice D) Chronic hypersensitivity pneumonitis is due to repeated exposure to an inciting
agent (eg, birds, fungi, chemicals). Cough and dyspnea occur shortly after exposure to the

Telegram Group Invite Link: https://t.me/joinchat/zyFOIg0j5goxY2E1


https://t.me/joinchat/zyFOIg0j5goxY2E1

Telegram Group Invite Link: https://t.me/USMLEWorldStep1

antigen and may progress to respiratory failure. Histologic findings include noncaseating
granulomas, peribronchiolar fibrosis, and patchy lymphocytic infiltrates.
(Choice E) Cryptogenic organizing pneumonia is another interstitial lung disease of unclear
etiology that typically presents with dry cough, dyspnea, and fatigue. However, histologic
findings include fibroblastic plugs in the alveolar sacs and ducts, often extending into adjacent
alveoli in a characteristic butterfly pattern.

Telegram Group Invite Link: https://t.me/joinchat/zyFOIg0j5goxY2E1


Telegram Group Invite Link: https://t.me/USMLEWorldStep1

thoracocentesis

Telegram Group Invite Link: https://t.me/joinchat/zyFOIg0j5goxY2E1


https://t.me/joinchat/zyFOIg0j5goxY2E1

Telegram Group Invite Link: https://t.me/USMLEWorldStep1

Telegram Group Invite Link: https://t.me/joinchat/zyFOIg0j5goxY2E1


Telegram Group Invite Link: https://t.me/USMLEWorldStep1

The visceral pleura covers the surface of the lung, whereas the parietal pleura lines the inner
surface of the chest wall and diaphragm. The parietal
pleura generally extends approximately 2 ribs below the inferior margin of the lungs. The
potential space at the reflection of the costal pleura onto diaphragmatic pleura is called the
costodiaphragmatic recess, where pleural fluid accumulates when the body is erect.
Thoracentesis is typically performed between the 6th and 8th ribs along the midclavicular line,
the 8th and 10th ribs along the midaxillary line (above image), and the 10th and 12th ribs along
the paravertebral line. This allows pleural fluid to be drained without risking lung injury (which
can occur at higher insertion sites). However, insertion of a needle below the 9th rib still risks
penetrating abdominal structures.
In this case, thoracentesis is being performed along the upper border of the 10th rib at the
right midaxillary line, putting the patient at risk of liver injury if the needle is inserted too deep.
(Choice A) Hepatic veins are found deep in the liver parenchyma and are not likely to be
injured during thoracentesis.
(Choices B and C) The intercostal vein, artery, and nerve lie in the subcostal groove along the
lower border of the rib. Thoracentesis should be performed just above the upper border of the
rib to prevent injury to the intercostal vessels.
(Choice E) The lower border of the right lung is located 2 intercostal spaces above the pleural
border. Therefore, the lung is less likely to be injured by insertion of a needle into the 10th
intercostal space at the midaxillary line.

Parapneumonic effusion

This patient's several weeks of fever, shortness of breath, and fatigue with imaging showing
a loculated pleural effusion with complex septations represents a classic presentation

Telegram Group Invite Link: https://t.me/joinchat/zyFOIg0j5goxY2E1


https://t.me/joinchat/zyFOIg0j5goxY2E1

Telegram Group Invite Link: https://t.me/USMLEWorldStep1

of empyema. Empyema is an advanced form of complicated parapneumonic effusion in which


bacterial invasion into the pleural space is followed by progressive inflammation with pus
accumulation and organized fibrosis. Treatment requires prompt drainage of the infected
fluid via a chest tube, but such effusions are often difficult to drain due to numerous loculations
(ie, separated fluid pockets) and high fluid viscosity.
In some cases, intrapleural administration of a fibrinolytic agent (eg, tissue plasminogen
activator [tPA], streptokinase) in combination with deoxyribonuclease (DNase) can improve
drainage of a loculated empyema and help resolve the effusion via chest tube drainage. The
tPA activates fibrin-bound plasminogen to break down organized fibrin, and the DNase
enzyme may assist by cleaving nucleic acids that increase fluid viscosity after being deposited
by lysed leukocytes (eg, neutrophil extracellular traps).
A complicated parapneumonic effusion or empyema that cannot be successfully drained
following the administration of tPA and DNase typically requires surgical drainage via video-
assisted thoracoscopic surgery.
(Choice B) Interstitial edema formation occurs early in the development of pleural effusion,
causing increased fluid to pass into the pleural space. Once organized fibrosis and empyema
develop, reducing interstitial edema formation is of no significant benefit.
(Choices C and D) Factor Xa inhibitors (eg, rivaroxaban, apixaban) are given systemically for
the treatment of venous thromboembolic disease, and medications that inhibit platelet activity
(eg, aspirin, P2Y12 inhibitors) are used systemically to lower the risk of intra-arterial
thrombosis. However, none of these medications target fibrin (the major contributor to
development of loculated empyema), and they are therefore not helpful in the management of
empyema.
(Choice E) Pleurodesis involves the instillation of a chemical irritant (eg, talc) into the pleural
space to fuse the visceral and parietal pleural layers together. It is used to prevent
reaccumulation of pleural fluid in patients with recurrent pleural effusion, but it does not assist in
drainage of an empyema.

Telegram Group Invite Link: https://t.me/joinchat/zyFOIg0j5goxY2E1


Telegram Group Invite Link: https://t.me/USMLEWorldStep1

Pleural fluid
flow

Humans normally have approximately 5-10 mL of pleural fluid in the intrapleural space of each
hemithorax, with inflow and outflow of fluid taking place at equal rates to maintain
homeostasis. Pleural fluid is believed to enter the pleural space via filtration from systemic
circulation in both the parietal and visceral pleura, with the majority of fluid filtered from
the intercostal microvessels in the parietal pleura and a lesser amount from the bronchial
microvessels in the visceral pleura.
Most pleural fluid likely exits the pleural space via holes in the parietal pleura known as
lymphatic stoma that drain into the parietal pleural lymphatics.
Pleural effusion results from an increase in the rate of pleural fluid inflow (eg, increased fluid
filtration as occurs with decompensated heart failure, hypoalbuminemia, or an inflammatory
increase in pleural vascular permeability) or a decrease in the rate of pleural fluid outflow (eg,

Telegram Group Invite Link: https://t.me/joinchat/zyFOIg0j5goxY2E1


https://t.me/joinchat/zyFOIg0j5goxY2E1

Telegram Group Invite Link: https://t.me/USMLEWorldStep1

malignant obstruction of the parietal pleura lymphatic stoma).

(Choices A, B, and D) Neither the pulmonary capillaries nor the pulmonary parenchymal
lymphatics significantly contribute to the uptake of pleural fluid from the pleural space in the
normal or pathologic state. Fluid from the pulmonary (alveolar) capillaries (ie, the pulmonary
circulation) does not significantly contribute to inflow of pleural fluid into the pleural space in the
normal state; however, it can become a primary source of fluid inflow in pathologic states (eg,
increased pulmonary capillary hydrostatic pressure with decompensated heart failure).

Telegram Group Invite Link: https://t.me/joinchat/zyFOIg0j5goxY2E1


Telegram Group Invite Link: https://t.me/USMLEWorldStep1

Pleural effusion

This patient with dyspnea, orthopnea, jugular venous distension, and lower extremity swelling
likely has an acute heart failure exacerbation. Heart failure commonly leads to pleural
effusion due to poor forward blood flow from the left ventricle and a subsequent increase
in pulmonary venous and pulmonary capillary hydrostatic pressure. Pleural effusions that
result from such pressure changes are typically transudative, whereas those that result from an
inflammatory increase in vascular membrane permeability are typically exudative.
Because transudative and exudative pleural effusions have differing underlying causes, they
also tend to differ in chemical makeup. The Light criteria are used to differentiate the 2 types
via analysis of the total protein and lactate dehydrogenase (LDH) levels. Exudative effusions
have a high pleural fluid/serum ratio of these proteins due to increased capillary permeability,
while transudative effusions are associated with a low pleural fluid/serum ratio.
Glucose levels and leukocyte counts can suggest the degree of inflammation in a pleural
effusion. Because leukocytes metabolize glucose, highly inflammatory effusions typically have
low glucose levels. Transudative effusions are not inflammatory and almost always have low
nucleated cell counts and normal (or high) glucose levels.
(Choices B and D) These effusions meet Light criteria for an exudate. The low glucose levels
(<60 mg/dL) suggest consumption of glucose by an abundance of metabolically active cells, as
can occur with highly inflammatory exudative effusions resulting from bacterial infection,
malignancy, or rheumatologic disease.
(Choices C and E) These effusions meet Light criteria for an exudate. The normal glucose
levels and low cell counts suggest a relatively low-inflammation exudative effusion such as an
uncomplicated parapneumonic effusion or effusion due to pulmonary embolism.

Telegram Group Invite Link: https://t.me/joinchat/zyFOIg0j5goxY2E1


https://t.me/joinchat/zyFOIg0j5goxY2E1

Telegram Group Invite Link: https://t.me/USMLEWorldStep1

Fat embolism
The clinical triad of acute-onset neurologic abnormalities, hypoxemia, and petechiae in a patient
with traumatic bone fracture is strongly suggestive of fat embolism syndrome (FES). This
condition most commonly occurs 24-72 hours following long-bone and/or pelvic
fracture. Pathophysiologically, the traumatic event dislodges fat globules from the bone
marrow and allows them to enter disrupted marrow venules, where they can then traverse the
systemic veins and deposit in pulmonary microvessels. The pulmonary capillary
occlusion impairs gas exchange and induces hypoxemia; release of free fatty acids from the
fat globules also causes local toxic injury to the endothelium, potentially leading to acute
respiratory distress syndrome.
Some fat globules escape the lungs via precapillary arteriovenous shunts that open due to
increased pulmonary artery pressure. This phenomenon appears to be responsible for the fat
emboli–associated microvascular occlusion that can occur within the CNS, manifesting with
confusion and neurologic impairment, and in the dermal capillaries, resulting in erythrocyte
extravasation and a petechial rash.
(Choices A, C, and D) A neutrophil-rich alveolar exudate is not an early histologic
manifestation of FES but rather of acute bacterial or aspiration pneumonia. Mononuclear
interstitial pulmonary infiltrates are found early in the course of a variety of interstitial lung
diseases. A red thrombus lodged in the pulmonary artery is consistent with a thromboembolism
from a deep venous source. None of these conditions would typically be associated with the
neurologic impairment or petechial rash seen with FES.
(Choice E) Focal necrosis of alveolar walls with associated intra-alveolar hemorrhage is often
seen with pulmonary hemorrhage syndromes (eg, anti-glomerular basement membrane
antibody disease, granulomatosis with polyangiitis).

COPD

Telegram Group Invite Link: https://t.me/joinchat/zyFOIg0j5goxY2E1


Telegram Group Invite Link: https://t.me/USMLEWorldStep1

This patient's presentation with progressive shortness of breath, significant smoking history,
and hyperinflation on chest x-ray (flattened diaphragm, narrowed mediastinum) is consistent
with chronic obstructive pulmonary disease (COPD).
COPD causes air trapping and hyperinflation of the lungs, so these patients breathe at higher
baseline lung volumes (higher functional residual capacity). The volume of air in the lungs
that is not respired, the residual volume (RV), is substantially increased. The total lung
capacity (TLC) increases as well, but to a lesser extent than RV. Therefore, the fraction of air
in the lungs that is not involved in respiration, the RV/TLC ratio, is also increased. A high
RV/TLC ratio correlates with poor outcomes in patients with COPD.
(Choice A) Diffusing capacity of the lung for carbon monoxide (DLCO) is decreased in COPD
due to emphysematous destruction of the alveolar-arterial membrane.
(Choices B, C, and D) Both forced vital capacity (FVC) and forced expiratory volume in 1
second (FEV1) are decreased in COPD due to airway obstruction. Because FEV1 is decreased
more than FVC, the FEV1/FVC ratio is also decreased (<0.7 in COPD).
Small cell carcinoma

This patient with a heavy smoking history, weight loss, cough, and a mediastinal mass with
evidence of metastases (ie, supraclavicular node elargement) has small cell lung
cancer (SCLC), also known as oat cell carcinoma. SCLC makes up 10%-20% of all primary
lung malignancies and is strongly associated with smoking. It is usually centrally located and
arises from the primitive cells of the basal layer of the bronchial epithelium. On light
microscopy, small, round or oval cells with scant cytoplasm, hyperchromatic (blue) nuclei,
and granular chromatin are visualized. The cells, which may resemble lymphocytes but are
typically larger, form clusters or sheets. Abundant mitoses are usually seen.
Small cell carcinomas can display varying degrees of neuroendocrine
differentiation. Immunohistochemical stains are frequently positive for neuroendocrine
markers, such as neuron-specific enolase, chromogranin, neural cell adhesion molecule
(CD56), and synaptophysin. On electron microscopy, some of the cells have secretory

Telegram Group Invite Link: https://t.me/joinchat/zyFOIg0j5goxY2E1


https://t.me/joinchat/zyFOIg0j5goxY2E1

Telegram Group Invite Link: https://t.me/USMLEWorldStep1

granules in the cytoplasm.

(Choice A) Adenocarcinoma is the most common form of cancer in both nonsmokers and the
total population. It is histologically characterized by glandular differentiation (eg, gland
formation, mucin production), and the tumor cells often show abundant cytoplasm and
eccentrically placed nuclei. The cells often stain positive for mucin. It typically arises in the

Telegram Group Invite Link: https://t.me/joinchat/zyFOIg0j5goxY2E1


Telegram Group Invite Link: https://t.me/USMLEWorldStep1

periphery of the lung.

(Choice B) Hodgkin lymphoma typically presents with cervical lymphadenopathy and can
cause a mediastinal mass. However, microscopy demonstrates Reed-Sternberg cells (large cell
with multilobed nucleus or multiple nuclei, prominent nucleoli, abundant cytoplasm, and an
"owl's eye" appearance) in a background of inflammatory cells. In addition, other B symptoms

Telegram Group Invite Link: https://t.me/joinchat/zyFOIg0j5goxY2E1


https://t.me/joinchat/zyFOIg0j5goxY2E1

Telegram Group Invite Link: https://t.me/USMLEWorldStep1

(eg, fever, night sweats) are common.

(Choice C) Papillary thyroid carcinoma presents with a thyroid nodule and cervical
lymphadenopathy but can also cause an anterior mediastinal mass. Histology reveals
branching papillae, which are composed of a fibrovascular stalk covered by neoplastic cuboidal
cells with clear ("ground-glass") nuclei. Psammoma bodies (laminated calcium deposits) can

Telegram Group Invite Link: https://t.me/joinchat/zyFOIg0j5goxY2E1


Telegram Group Invite Link: https://t.me/USMLEWorldStep1

sometimes be seen.

(Choice E) Squamous cell carcinoma typically arises centrally and is composed of polygonal
cells with eosinophilic cytoplasm and distinct borders. Well-differentiated squamous cell

Telegram Group Invite Link: https://t.me/joinchat/zyFOIg0j5goxY2E1


https://t.me/joinchat/zyFOIg0j5goxY2E1

Telegram Group Invite Link: https://t.me/USMLEWorldStep1

carcinomas show keratin pearls and intercellular bridges on light microscopy.

Small cell carcinoma

Telegram Group Invite Link: https://t.me/joinchat/zyFOIg0j5goxY2E1


Telegram Group Invite Link: https://t.me/USMLEWorldStep1

This patient with a heavy smoking history, weight loss, cough, and a mediastinal mass with
evidence of metastases (ie, supraclavicular node elargement) has small cell lung
cancer (SCLC), also known as oat cell carcinoma. SCLC makes up 10%-20% of all primary
lung malignancies and is strongly associated with smoking. It is usually centrally located and
arises from the primitive cells of the basal layer of the bronchial epithelium. On light
microscopy, small, round or oval cells with scant cytoplasm, hyperchromatic (blue) nuclei,
and granular chromatin are visualized. The cells, which may resemble lymphocytes but are
typically larger, form clusters or sheets. Abundant mitoses are usually seen.
Small cell carcinomas can display varying degrees of neuroendocrine
differentiation. Immunohistochemical stains are frequently positive for neuroendocrine
markers, such as neuron-specific enolase, chromogranin, neural cell adhesion molecule
(CD56), and synaptophysin. On electron microscopy, some of the cells have secretory

Telegram Group Invite Link: https://t.me/joinchat/zyFOIg0j5goxY2E1


https://t.me/joinchat/zyFOIg0j5goxY2E1

Telegram Group Invite Link: https://t.me/USMLEWorldStep1

granules in the cytoplasm.

(Choice A) Adenocarcinoma is the most common form of cancer in both nonsmokers and the
total population. It is histologically characterized by glandular differentiation (eg, gland
formation, mucin production), and the tumor cells often show abundant cytoplasm and
eccentrically placed nuclei. The cells often stain positive for mucin. It typically arises in the

Telegram Group Invite Link: https://t.me/joinchat/zyFOIg0j5goxY2E1


Telegram Group Invite Link: https://t.me/USMLEWorldStep1

periphery of the lung.

(Choice B) Hodgkin lymphoma typically presents with cervical lymphadenopathy and can
cause a mediastinal mass. However, microscopy demonstrates Reed-Sternberg cells (large cell
with multilobed nucleus or multiple nuclei, prominent nucleoli, abundant cytoplasm, and an
"owl's eye" appearance) in a background of inflammatory cells. In addition, other B symptoms

Telegram Group Invite Link: https://t.me/joinchat/zyFOIg0j5goxY2E1


https://t.me/joinchat/zyFOIg0j5goxY2E1

Telegram Group Invite Link: https://t.me/USMLEWorldStep1

(eg, fever, night sweats) are common.

(Choice C) Papillary thyroid carcinoma presents with a thyroid nodule and cervical
lymphadenopathy but can also cause an anterior mediastinal mass. Histology reveals
branching papillae, which are composed of a fibrovascular stalk covered by neoplastic cuboidal
cells with clear ("ground-glass") nuclei. Psammoma bodies (laminated calcium deposits) can
sometimes be seen.

Telegram Group Invite Link: https://t.me/joinchat/zyFOIg0j5goxY2E1


Telegram Group Invite Link: https://t.me/USMLEWorldStep1

(Choice E) Squamous cell carcinoma typically arises centrally and is composed of polygonal
cells with eosinophilic cytoplasm and distinct borders. Well-differentiated squamous cell
carcinomas show keratin pearls and intercellular bridges on light microscopy.
Pulmonary
edema

This patient has chest pain and ECG evidence of acute myocardial infarction (MI) involving
the lateral wall of the left ventricle. His subsequent development of dyspnea and orthopnea is
most likely due to MI-induced acute left ventricular failure, which can result in rapid onset of
pulmonary venous hypertension and acute pulmonary edema.
Cardiogenic pulmonary edema represents increased filtration of fluid and electrolytes into the
lung interstitium and alveoli. The fluid that accumulates is a transudate (an ultrafiltrate of
plasma caused by changes in hydrostatic or oncotic pressure) rather than an exudate (an
extravasation of not only fluid and electrolytes but also plasma proteins and circulating
leukocytes due to inflammatory disruption of the vascular membrane). On light microscopy,
engorged alveolar capillaries are evident and the intraalveolar transudate appears as acellular

Telegram Group Invite Link: https://t.me/joinchat/zyFOIg0j5goxY2E1


https://t.me/joinchat/zyFOIg0j5goxY2E1

Telegram Group Invite Link: https://t.me/USMLEWorldStep1

pink material.

(Choice A) Fat globules and bone marrow cells in the pulmonary arterioles occur with fat
embolism syndrome, which is not a complication of MI but is typically associated with long bone
(eg, femur) or pelvic fracture.
(Choice B) Focal necrosis of alveolar walls with intraalveolar hemorrhage is typical of
pulmonary hemorrhage syndromes such as Goodpasture syndrome and other vasculitides (eg,
hypersensitivity angiitis, granulomatosis with polyangiitis).
(Choice C) The presence of numerous neutrophils in the alveolar fluid is consistent with
exudative alveolar filling, as occurs in bacterial pneumonia.

Telegram Group Invite Link: https://t.me/joinchat/zyFOIg0j5goxY2E1


Telegram Group Invite Link: https://t.me/USMLEWorldStep1

(Choice D) Hemosiderin-laden macrophages ("heart failure cells") form as macrophages digest


red blood cells that leak from alveolar capillaries damaged by high intravascular pressure. They
are a sign of chronic lung congestion and would not be present acutely.
(Choice F) Mononuclear interstitial pulmonary infiltrates are found in various interstitial lung
diseases (eg, idiopathic pulmonary fibrosis).
Pleural
effusion

This patient's several-month history of nonproductive cough, shortness of breath, and weight
loss combined with a large left-sided pleural effusion on chest x-ray raises suspicion for
malignancy. Thoracentesis yielding a high erythrocyte concentration (a common finding in
malignant effusions) and atypical mucin cells is consistent with adenocarcinoma; lung
adenocarcinoma and breast adenocarcinoma are 2 of the most common causes of malignant
pleural effusion.
Malignant effusions are exudative by Light criteria and can occur via several mechanisms:
• Localized lung inflammation can cause increased vascular permeability, resulting
in increased inflow of fluid into the pleural space.

Telegram Group Invite Link: https://t.me/joinchat/zyFOIg0j5goxY2E1


https://t.me/joinchat/zyFOIg0j5goxY2E1

Telegram Group Invite Link: https://t.me/USMLEWorldStep1

• Once malignant cells have metastasized to the pleural space, they can occlude the
pleural lymphatic stoma located on the parietal surface and prevent pleural fluid
reabsorption. This is likely the primary mechanism of effusion in this patient with
evidence of pleural metastasis (ie, atypical mucin cells in pleural fluid).
• Disruption of the thoracic lymphatic duct is an occasional cause of malignant effusion
that leads to a chylothorax (milky white pleural fluid with high triglyceride content). This
mechanism is most commonly seen with lymphoma; it can sometimes occur due to
mass effect of lung cancer on the thoracic duct, but this is relatively uncommon (Choice
B).

(Choice A) Decreased plasma oncotic pressure (ie, hypoalbuminemia) is a common cause of


transudative pleural effusion. Although hypoalbuminemia due to malnutrition may complicate
malignancy and could contribute to pleural effusion, it is unlikely to be the primary mechanism in
this patient with evidence of pleural metastatic disease.
(Choices C and D) Increased hydrostatic pressure in the pulmonary capillaries and intercostal
veins can cause transudative pleural effusion in patients with decompensated heart failure or
other causes of intravascular volume overload (eg, renal failure).

Telegram Group Invite Link: https://t.me/joinchat/zyFOIg0j5goxY2E1


Telegram Group Invite Link: https://t.me/USMLEWorldStep1

Pulmonary
embolism

This patient has developed a deep venous thrombosis in his left leg following a lengthy car trip
and now has clinical features highly consistent with acute pulmonary
embolism (PE). Obstruction of the pulmonary circulation by an embolus causes increased
dead space ventilation (ie, alveoli are ventilated but not perfused); the term "dead space"
refers to the volume of inspired air that does not participate in gas exchange. As dead space
ventilation increases, blood that continues to flow through the pulmonary circulation cannot be
Telegram Group Invite Link: https://t.me/joinchat/zyFOIg0j5goxY2E1
https://t.me/joinchat/zyFOIg0j5goxY2E1

Telegram Group Invite Link: https://t.me/USMLEWorldStep1

fully oxygenated by the decreased number of accessible alveoli (ventilation/perfusion (V/Q)


mismatch), leading to hypoxemia.
(Choice A) Functional residual capacity refers to the volume of air remaining in the lungs after
a normal expiration. It is increased in chronic obstructive pulmonary disease (COPD) due to air
trapping; however, it is unchanged in PE.
(Choice B) Diffusion capacity is reduced by conditions that disrupt the alveolar-capillary
interface (eg, pulmonary fibrosis), but it is not altered in PE.
(Choice D) Pulmonary compliance (ability to expand) is reduced in restrictive lung disease (eg,
pulmonary fibrosis) and is increased in COPD (due to emphysematous destruction of the distal
airways). However, it is not significantly altered in the setting of PE.
(Choice E) Total airway resistance is increased by bronchoconstriction, which occurs in
patients with asthma. PE does not affect airway resistance.
Physiological dead
space

Physiologic (total) dead space includes the anatomic dead space (the permanent volume of
dead space within the conducting airways) plus the alveolar dead space (the volume of air in the
respiratory zone that does not participate in gas exchange). Increases in physiologic dead
Telegram Group Invite Link: https://t.me/joinchat/zyFOIg0j5goxY2E1
Telegram Group Invite Link: https://t.me/USMLEWorldStep1

space occur in many lung diseases, including pulmonary embolism, emphysema, and acute
respiratory distress syndrome.
Dead space volume remains relatively constant on a minute-by-minute basis. However, lower
tidal volumes increase the proportion of each breath composed of dead space. If minute
ventilation (tidal volume x respiratory rate) is unchanged, the consequence of the low-tidal-
volumes is an increase in dead space ventilation (inefficient breathing).
Patients undergoing mechanical ventilation weaning typically have weakened respiratory
muscles and therefore tend to breathe at low tidal volumes to minimize the work of
breathing. This hypoventilation triggers an increase in respiratory drive that increases
respiratory rate to maintain minute ventilation. When patients are being weaned off the
ventilator, the ratio of their respiratory rate/tidal volume, known as the rapid shallow breathing
index (RSBI), is measured; a low RSBI indicates relatively high tidal volumes, relatively efficient
breathing, and a lower likelihood of recurrent respiratory failure once ventilatory support is
discontinued.
(Choice A) Increases in both respiratory rate and tidal volume result in increased minute
ventilation. However, the graph shows that minute ventilation remains the same after ventilator
adjustment.
(Choice C) A decrease in respiratory rate and an increase in tidal volume would keep minute
ventilation constant; however, the total volume of dead-space air breathed each minute would
decrease rather than increase.
(Choice D) Decreases in both respiratory rate and tidal volume would result in decreased,
rather than constant, minute ventilation.
Chronic bronchitis
This patient with respiratory failure, hypoxemia, and thickened bronchial walls with inflammatory
infiltrates and mucous gland enlargement likely had chronic bronchitis. Chronic bronchitis is
characterized by chronic, productive cough with airflow limitation and is part of the spectrum of
chronic obstructive pulmonary disease. It is most commonly caused by tobacco
smoking. Chronic irritation by other inhaled environmental substances, such as air pollutants
and grain, cotton, or silica dusts, can also contribute to its development. Biopsy typically
shows thickened bronchial walls with predominantly lymphocytic infiltrates, mucous gland
enlargement with increased numbers of goblet cells (increasing mucus production), and
patchy squamous metaplasia of the bronchial mucosa.
(Choice A) Patients with allergic asthma can develop pathologic remodeling of the bronchial
wall, which includes thickening of the bronchial epithelium, basement membrane, and bronchial
smooth muscle as well as edema, inflammatory infiltrates, and submucosal mucous gland
enlargement. However, the infiltrates consist predominantly of eosinophils and mast cells. In
addition, although asthma is a risk factor for chronic bronchitis, smoking is a much more
common cause.
(Choice C) Genetic factors are not known to strongly predispose to chronic bronchitis. Genetic
mutation causing alpha-1 antitrypsin deficiency can lead to panacinar emphysema; however,
chronic bronchitis is not a significant component of the disease.
(Choice D) Repeated bronchial/bronchiolar bacterial and viral infections can contribute to the
development of chronic bronchitis, although less significantly than can smoking. Tobacco
smoke predisposes to infection by impairing ciliary clearance and directly damaging the
respiratory epithelium.

Telegram Group Invite Link: https://t.me/joinchat/zyFOIg0j5goxY2E1


https://t.me/joinchat/zyFOIg0j5goxY2E1

Telegram Group Invite Link: https://t.me/USMLEWorldStep1

(Choices E and F) Nickel is a carcinogen, and occupational exposure is associated with nasal
and lung cancers. However, neoplastic transformation itself does not contribute to the
development of chronic bronchitis. Although this patient's history of nickel mining is also
suggestive of silica dust exposure, smoking is the most important risk factor for chronic
bronchitis.
COPD

The graph compares the respiration curve for a normal, healthy individual (black line) and this
patient (blue line), who has chronic obstructive pulmonary disease (COPD); the tracings
show resting breaths along with a maximal air intake and expulsion effort. In a resting breath, a
single tidal volume (TV) is exchanged. With maximal inhalation, the curve includes both the TV

Telegram Group Invite Link: https://t.me/joinchat/zyFOIg0j5goxY2E1


Telegram Group Invite Link: https://t.me/USMLEWorldStep1

and the inspiratory reserve volume (IRV), which together equal the inspiratory capacity. With
maximal exhalation, the curve reflects the expulsion of the IRV and the TV, as well as the
amount of additional air that can be expired, which is the expiratory reserve volume (ERV). The
air remaining in the lungs following full exhalation is the residual volume (RV).
COPD causes air trapping and hyperinflation of the lungs, so these patients breathe at higher
baseline lung volumes (higher FRC). The absolute volume of air in the lungs that is not
respired, the RV, is substantially increased. The total lung capacity (TLC) also increases but
to a lesser extent than RV. Therefore, the fraction of air in the lungs that is not involved in
respiration, the RV/TLC ratio, is also increased.

(Choice A) ERV is decreased in obstructive lung disease, as less air can be forcefully exhaled
due to obstruction. The substantial increase in RV is greater than the decrease in ERV,

Telegram Group Invite Link: https://t.me/joinchat/zyFOIg0j5goxY2E1


https://t.me/joinchat/zyFOIg0j5goxY2E1

Telegram Group Invite Link: https://t.me/USMLEWorldStep1

resulting in the overall increase in FRC.

(Choices B, C, and D) Both forced vital capacity (FVC) and forced expiratory volume in 1
second (FEV1) are decreased in obstructive lung disease due to airway obstruction. Because
FEV1 is decreased more than FVC, the FEV1/FVC ratio is also decreased (<0.7 in COPD).

V/Q match

Telegram Group Invite Link: https://t.me/joinchat/zyFOIg0j5goxY2E1


Telegram Group Invite Link: https://t.me/USMLEWorldStep1

In the upright position, regional differences in ventilation and perfusion occur vertically in the
lungs due to gravity. Ventilation is lowest in the apex and highest in the base because
gravity acts to stretch the lungs downward so that the alveoli at the apex are expanded more
than those at the base (ie, slinky effect). Therefore, during inspiration a small amount of air
goes to the apex where alveoli are distended and less compliant, while more air goes to the
base where alveoli have ample potential space to fill and are more compliant. Perfusion is
also lowest in the apex and highest in the base, as the increased hydrostatic pressure in the
lower lung regions facilitates increased blood flow.
Because blood is denser than air or lung tissue, the gravitational effect on perfusion is more
pronounced than on ventilation, leading to greater variability in perfusion than
ventilation from apex to base. Therefore, perfusion is very low in the apex and very high in the
base, whereas ventilation is only somewhat low in the apex and somewhat high in the

Telegram Group Invite Link: https://t.me/joinchat/zyFOIg0j5goxY2E1


https://t.me/joinchat/zyFOIg0j5goxY2E1

Telegram Group Invite Link: https://t.me/USMLEWorldStep1

base. This causes the gradient of the ventilation/perfusion (V/Q) ratio to be opposite of the
gradient of perfusion or ventilation taken individually. The V/Q ratio is lowest at the
base and highest at the apex.

(Choices A and B) These graphs depict the V/Q ratio decreasing from base to apex, which is
the opposite of what occurs.
(Choice C) This graph depicts a constant V/Q ratio of 1.0 throughout the lung, which does not
occur. The V/Q ratio varies from approximately 0.6 at the base to 3.0 at the apex. A value of
0.8 is often considered the average V/Q ratio for normal lungs.
(Choice E) This graph shows constant perfusion throughout the lung, which does not occur.

Telegram Group Invite Link: https://t.me/joinchat/zyFOIg0j5goxY2E1


Telegram Group Invite Link: https://t.me/USMLEWorldStep1

Hypersensitive pneumonitis

This patient with several months of nonproductive cough and exertional dyspnea as well as
a lymphocyte-predominant bronchoalveolar lavage (BAL) most likely has hypersensitivity
pneumonitis (HP). HP results from an exaggerated immunologic response that some
individuals develop to an inhaled antigen (eg, mold, animal protein). Those most commonly
affected include farmers (ie, farmer's lung due to exposure to moldy hay) and bird keepers
(ie, bird fancier's lung due to exposure to avian proteins).
The presentation of HP can be acute or chronic. Acute disease usually involves recurrent
episodes of abrupt-onset cough, dyspnea, fever, chills, and fatigue that correlate with
intermittent antigen exposure. Chronic disease likely results from chronic, long-term antigen
exposure, and presents with gradually progressive cough, dyspnea, fatigue, and weight loss
over a period of several months. Lung crackles are present with both acute and chronic
disease. Chest x-ray in acute disease is frequently normal, whereas diffuse reticular
interstitial opacities are present with chronic disease as a network of interstitial inflammation
and fibrosis develops.
Normally, the leukocytes in alveolar fluid consist of approximately 85% alveolar macrophages,
10% lymphocytes, and a small percentage of neutrophils and eosinophils. With both acute and
chronic HP, BAL usually shows high relative lymphocyte count (eg, >20%, often >50%),
which helps support the diagnosis. Other causes of high relative lymphocyte count in BAL
include sarcoidosis, lymphoma, and chronic fungal or mycobacterial infection.
(Choice A) Asbestosis can present with progressive dyspnea and cough, but BAL typically
shows increased neutrophils and characteristic asbestos bodies.
(Choice B) Asthma is expected to cause wheezing rather than crackles on lung
auscultation. BAL may show increased eosinophils as well as bronchial epithelial cells.

Telegram Group Invite Link: https://t.me/joinchat/zyFOIg0j5goxY2E1


https://t.me/joinchat/zyFOIg0j5goxY2E1

Telegram Group Invite Link: https://t.me/USMLEWorldStep1

(Choice C) Patients with gastroesophageal reflux disease can have chronic microaspiration,
leading to respiratory symptoms (eg, cough, dyspnea). However, BAL typically shows
increased neutrophils and characteristic lipid-laden macrophages that result from the aspiration
of lipid-containing food or drink.
(Choice E) BAL in cardiogenic pulmonary edema often demonstrates hemosiderin-laden
macrophages, resulting from elevated pulmonary capillary hydrostatic pressure leading to
extravasation of red blood cells into the alveoli. Leukocyte percentages are not significantly
affected.
Mesothelioma
This patient with progressive dyspnea and cough has nodular pleural thickening and a pleural
effusion. This presentation, in conjunction with characteristic histopathology, suggests
mesothelioma. Mesothelioma is a rare, malignant neoplasm arising from mesothelial cells,
which line body cavities (eg, pleural, peritoneal, pericardial). Histopathology shows tumor cells
with numerous long, slender microvilli and abundant tonofilaments. Immunohistochemical
markers (eg, pancytokeratin) are useful in diagnosis.
Mesothelioma typically presents with slowly progressive dyspnea, cough, and chest
pain. Pleural effusions are common and are often hemorrhagic. Radiography may demonstrate
nodular or smooth, unilateral pleural thickening (yellow arrow) and plaque formation; the lung
parenchyma is typically uninvolved. Asbestos exposure is the primary risk factor; individuals
involved in asbestos mining and industrial applications (eg, insulation, shipbuilding) are at risk

for mesothelioma.

Telegram Group Invite Link: https://t.me/joinchat/zyFOIg0j5goxY2E1


Telegram Group Invite Link: https://t.me/USMLEWorldStep1

(Choice A) Adenocarcinoma cells (which can display glandular or papillary characteristics)


have short, plump microvilli that distinguish them from mesothelioma cells. Adenocarcinoma
tends to involve peripheral lung parenchyma.

(Choice B) Carcinoid tumors are neuroendocrine malignancies that most commonly affect the
gastrointestinal tract and lungs. They sometimes cause carcinoid syndrome (flushing, diarrhea,
wheezing), which is seen more commonly with gastrointestinal tumors. They tend to involve the
large bronchi and can appear histologically as sheets of uniform cells with a "salt and pepper"

Telegram Group Invite Link: https://t.me/joinchat/zyFOIg0j5goxY2E1


https://t.me/joinchat/zyFOIg0j5goxY2E1

Telegram Group Invite Link: https://t.me/USMLEWorldStep1

pattern (chromatin with fine and coarse clumps).

(Choice D) Small cell carcinoma is associated with smoking but usually arises from the major
bronchi. On chest imaging, it is seen as a hilar mass. Pathology shows flat, oval-shaped
cells with scant cytoplasm and hyperchromatic nuclei. Neuroendocrine markers such as

Telegram Group Invite Link: https://t.me/joinchat/zyFOIg0j5goxY2E1


https://t.me/joinchat/zyFOIg0j5goxY2E1

USMLE e e r m h nne s
Me est i e s: https://t.me/USMLEMe est

K p n ssr m i e s: https://t.me/USMLEK p n

USMLE i e s Step : https://t.me/USMLE Step

e ker i e s: https://t.me/USMLE e ker

r s e n i e s Step : https://t.me/USMLE r sAn e n

r s e n i e s Step K: https://t.me/USMLE r sAn e n Step K

Ph se i e s: https://t.me/USMLEPh se

Pi ri e i e s: https://t.me/USMLEPi ri e

P th m i e s: https://t.me/USMLEP th m

nA i s i h it : https://t.me/USMLE nA i s

Am ss r p: https://t.me/USMLEAm ss

Le t ri nk: https://t.me/USMLELe t ri

En p int P s: https://t.me/USMLEEn p int

USMLE MEs: https://t.me/USMLE ME

L E Me E : https://t.me/USMLE n ineMe E

USMLE r Step : https://t.me/USMLE r Step

USMLE r Step K: https://t.me/USMLE r Step K

USMLE r Step nk: https://t.me/USMLE r Step

USMLE Anki: https://t.me/USMLEAnki

KissPrep: https://t.me/USMLEKissPrep
Telegram Group Invite Link: https://t.me/USMLEWorldStep1

chromogranin and synaptophysin are usually positive.

(Choice E) Squamous cell carcinoma is associated with smoking but arises from the major
bronchi; a hilar mass would be expected on imaging. Although desmosomes (intercellular
bridges) are seen in squamous cell carcinoma, they are nonspecific and can be found in any
tissue that experiences mechanical stress (eg, mesothelium, epithelium, heart). In addition,

Telegram Group Invite Link: https://t.me/joinchat/zyFOIg0j5goxY2E1


https://t.me/joinchat/zyFOIg0j5goxY2E1

Telegram Group Invite Link: https://t.me/USMLEWorldStep1

keratinization (keratin pearls) is expected on histologic examination.

dexamethasone
In the terminal saccular stage of lung development, the type II pneumocytes produce pulmonary
surfactant, a lipoprotein complex rich in phospholipids, most
notably dipalmitoylphosphatidylcholine (DPPC, a type of lecithin). This
surfactant decreases alveolar surface tension by creating a lipid-rich monolayer that
separates alveolar gas from the underlying aqueous fluid. The phenomenon prevents
atelectasis and end-expiratory collapse and increases pulmonary compliance.
In a fetus, efflux of lung fluid into amniotic fluid enables testing of markers of lung maturity. Until
33 weeks gestation, the lecithin and sphingomyelin levels are about equal. After 33 weeks, the
lecithin levels rise dramatically compared to sphingomyelin. A lecithin-to-sphingomyelin ratio
>1.9 is indicative of mature fetal lungs.
Premature infants born at <32 weeks gestation are at significant risk of surfactant
deficiency. Corticosteroids (eg, betamethasone, dexamethasone) have the greatest effect on
increasing surfactant production by accelerating maturation of type II pneumocytes. They are
administered to patients at risk for preterm labor (eg, preterm premature rupture of membranes)
to decrease the risk of respiratory distress syndrome and mortality.

Telegram Group Invite Link: https://t.me/joinchat/zyFOIg0j5goxY2E1


Telegram Group Invite Link: https://t.me/USMLEWorldStep1

(Choice B) Uncontrolled maternal hyperglycemia causes fetal hyperinsulinemia and inhibits the
maturational effects of cortisol and the production of surfactant proteins. Infants of these
mothers can develop respiratory distress syndrome. Although corticosteroids cause temporary
hyperglycemia, the beneficial effects of maternal steroid administration on surfactant production
and eventual reduction of mortality far outweigh this side effect.
(Choice C) In pregnancies at risk for preterm delivery, antenatal administration of magnesium
sulfate has been shown to decrease the risk for cerebral palsy. While cerebral palsy can result
in permanent neurologic disability, impaired lung function is a more common cause of death in
premature infants.
(Choices D and E) Nifedipine and terbutaline are medications used for inhibition of preterm
labor (tocolysis). Nifedipine is a calcium channel blocker and results in myometrial relaxation by
inhibition of myosin light-chain kinase-mediated phosphorylation. Terbutaline is a β-
sympathomimetic that increases intracellular cAMP formation in myometrial cells. This inhibits
myosin light-chain kinase and relaxes the smooth muscle. Neither drug directly affects fetal
lung maturity.

Telegram Group Invite Link: https://t.me/joinchat/zyFOIg0j5goxY2E1


https://t.me/joinchat/zyFOIg0j5goxY2E1

Telegram Group Invite Link: https://t.me/USMLEWorldStep1

alveoli

Surfactant, which is produced in type II pneumocytes, works to decrease the surface tension
in alveoli, facilitating lung expansion during respiration. When there is insufficient surfactant, as
in neonatal respiratory distress syndrome, the result is collapse of alveoli (atelectasis) due to
increased surface tension. Surfactant is stored and transported to the cell surface by lamellar
bodies (organelles containing parallel stacks of membrane lamellae). It is normally released by
Telegram Group Invite Link: https://t.me/joinchat/zyFOIg0j5goxY2E1
Telegram Group Invite Link: https://t.me/USMLEWorldStep1

exocytosis into the alveolar spaces, where the lamellar contents unravel and spread along the
alveolar lining. As it degrades, surfactant is recycled back into the same type II pneumocytes by
endocytosis for reprocessing.
(Choice A) Emphysema results from excessive activity of intra-alveolar proteases released
locally by infiltrating neutrophils and activated alveolar macrophages.
(Choice B) Bronchoconstriction can be caused by alveolar hypocapnia, bronchial inflammation,
parasympathetic (cholinergic) efferent nerve activity, and inhaled irritants. A deficiency of
pulmonary surfactant would not cause bronchoconstriction.
(Choice C) Chronic bronchitis manifests as excessive mucus production in the airways and
increased inflammatory cells visible in the submucosa.
(Choice E) Vasoconstriction of pulmonary arterioles can result from hypoxia secondary to
atelectasis. However, actual remodeling of pulmonary vasculature and pulmonary hypertension
solely from atelectasis-induced hypoxia are rare.
(Choice F) A deficiency of pulmonary surfactant would not be carcinogenic.
COPD

In chronic obstructive pulmonary disease (COPD), air flow obstruction and hyperinflation (air
trapping) cause increased residual volume (RV) and a consequent increase in total lung
capacity (TLC). Because the increase in RV is greater than the increase in TLC, the RV/TLC
ratio is increased. Airway obstruction also causes decreased forced vital capacity (FVC) and

Telegram Group Invite Link: https://t.me/joinchat/zyFOIg0j5goxY2E1


https://t.me/joinchat/zyFOIg0j5goxY2E1

Telegram Group Invite Link: https://t.me/USMLEWorldStep1

a more profound decrease in forced expiratory volume in one second (FEV1), resulting in a
decreased FEV1/FVC ratio.
In the above graphs, the top section represents the inspiratory reserve volume, the middle
section represents the tidal volume plus the expiratory reserve volume, and the bottom section
represents the RV. In COPD, an increase in RV and TLC is expected, as is a decrease in FVC.

(Choice A) In restrictive lung disease, the TLC, RV, FVC, and FEV1 are all reduced, but the
FEV1/FVC ratio is normal or increased.
(Choice C) This graph shows an increased TLC and FVC with a normal RV. Such an increase
in FVC may be seen in elite athletes.
(Choice D) This graph shows a decrease in TLC and an increase in RV. As changes in these
parameters are not typically discordant, this would be a rare clinical finding.

Telegram Group Invite Link: https://t.me/joinchat/zyFOIg0j5goxY2E1


Telegram Group Invite Link: https://t.me/USMLEWorldStep1

Pulmonary
hypertension

This patient's skin tightening on the fingers and Raynaud phenomenon (cold-induced digital
vasospasm) are suggestive of CREST syndrome (calcinosis, Raynaud phenomenon,
esophageal dysmotility, sclerodactyly, telangiectasia), which is strongly associated
with systemic sclerosis. Vascular manifestations are common in systemic sclerosis, and
some patients will develop pulmonary arterial hypertension (PAH). In this patient, PAH is
suggested by progressive dyspnea and a loud pulmonic component (P2) of S2;
hepatomegaly and peripheral edema suggest consequent right-sided heart failure.
PAH results from progressive remodeling of the small and medium-sized pulmonary
arteries/arterioles. In systemic sclerosis the remodeling is likely triggered by increased
proliferation of T cells with secretion of a variety of cytokines (eg, TGF-beta), which stimulate
fibroblasts to increase the production of collagen and extracellular matrix proteins. There is also
endothelial dysfunction due to an excess of vasoconstrictive, proproliferative mediators (eg,
endothelin, thromboxane A2) relative to vasodilative, antiproliferative mediators (eg, nitric oxide,
prostacyclin). Consequent vasoconstriction and smooth muscle proliferation with intimal
thickening of the vascular walls lead to increased pulmonary vascular resistance and elevated
pulmonary arterial pressure. Over time, the right ventricle is unable to pump against the
increased afterload, and right-sided heart failure develops.
(Choices A and E) Pulmonary interstitial fibrosis occurs in interstitial lung disease, which is a
common complication of systemic sclerosis and can lead to pulmonary hypertension due to
hypoxia-induced vasoconstriction. However, this patient's normal pulmonary function

Telegram Group Invite Link: https://t.me/joinchat/zyFOIg0j5goxY2E1


https://t.me/joinchat/zyFOIg0j5goxY2E1

Telegram Group Invite Link: https://t.me/USMLEWorldStep1

testing rules out significant lung disease.

(Choice C) Myocardial amyloid deposition occurs in amyloidosis and can lead to heart failure
due to restrictive cardiomyopathy. Although patients can have progressive dyspnea, Raynaud
phenomenon and digital skin tightening are not typical.
(Choice D) Pericardial fibrosis can occur in systemic sclerosis and typically manifests as right-
sided heart failure due to impaired diastolic filling of the right ventricle. However, pulmonary
arterial pressure is not typically elevated and a loud P2 is not expected.
(Choice F) Pulmonic stenosis, when severe, can lead to right-sided heart failure; however, a
soft P2 would be expected.

Telegram Group Invite Link: https://t.me/joinchat/zyFOIg0j5goxY2E1


Telegram Group Invite Link: https://t.me/USMLEWorldStep1

pneumonia

This patient has fever, productive cough, leukocytosis with left shift, and an area
of consolidation on chest x-ray that is consistent with lobar pneumonia. Pulmonary
consolidation occurs when the air in alveoli is replaced with a substance (eg, edema, pus,
blood, cellular debris) that results in a region of opacification on radiographs. The borders of a
consolidation tend to be indistinct as the alveoli become more sporadically involved at the
peripheral portions of the disease process. However, if the consolidation abuts a fissure (eg,
oblique, horizontal), it can be sharply defined because the consolidation cannot directly spread

Telegram Group Invite Link: https://t.me/joinchat/zyFOIg0j5goxY2E1


https://t.me/joinchat/zyFOIg0j5goxY2E1

Telegram Group Invite Link: https://t.me/USMLEWorldStep1

across the fissure to another lobe.

This patient's consolidation is sharply defined inferiorly by the horizontal fissure, which is
present only in the right lung and separates the right upper lobe from the right middle
lobe. Although multiple radiographic views are usually required for localization of a lung lesion,
this patient's findings on lateral radiograph are consistent with right upper lobe
pneumonia. Other features that can help localize a lobar pneumonia include:

• Right lung
o Middle lobe consolidation can be sharply defined superiorly by the horizontal
fissure and/or sharply defined inferiorly by the oblique fissure (Choice C).
o Lower lobe consolidation can be sharply defined superiorly by the oblique fissure.

• Left lung
o Left upper lobe and left lower lobe consolidations can be sharply defined by the
oblique fissure but would not be sharply defined by a horizontal fissure because
there is no middle lobe in the left lung (Choice A).

Telegram Group Invite Link: https://t.me/joinchat/zyFOIg0j5goxY2E1


Telegram Group Invite Link: https://t.me/USMLEWorldStep1

Telegram Group Invite Link: https://t.me/joinchat/zyFOIg0j5goxY2E1


https://t.me/joinchat/zyFOIg0j5goxY2E1

Telegram Group Invite Link: https://t.me/USMLEWorldStep1

Telegram Group Invite Link: https://t.me/joinchat/zyFOIg0j5goxY2E1


Telegram Group Invite Link: https://t.me/USMLEWorldStep1

(Choices B and D) In an upright chest x-ray, large accumulations of fluid in the pleural space
(eg, pleural effusion) can blunt the costophrenic angles and spread across lung fissures to
involve the area around more than one lung lobe.
Silicosis
This middle-aged patient with dyspnea on exertion, nodular densities on x-ray, calcified hilar
lymph nodes, and birefringent particles on biopsy has silicosis. Silicosis is a form of
pneumoconiosis (ie, a type of interstitial lung disease caused by inhalation of mineral
dust). Inhaled crystalline silica, typically due to industrial exposure (eg, mining, sand blasting),
is toxic to alveolar macrophages and promotes the formation of intrapulmonary free radicals,
leading to progressive lung injury and interstitial collagen deposition. Histologically, silicosis is
characterized by birefringent silicate particles within dense, whorled collagenous
nodules surrounded by dust-laden macrophages.
Silicosis is often initially asymptomatic but can present with dyspnea on exertion and productive
cough, typically 10-20 years after initial exposure. Radiography varies based on disease
progression; simple silicosis typically demonstrates numerous small, rounded
nodules predominant in the upper lobes that may rarely coalesce to form mass-like upper lobe
fibrosis (progressive massive fibrosis). Calcification of the rim of hilar nodes (eggshell
calcification) may also be seen.

Telegram Group Invite Link: https://t.me/joinchat/zyFOIg0j5goxY2E1


https://t.me/joinchat/zyFOIg0j5goxY2E1

Telegram Group Invite Link: https://t.me/USMLEWorldStep1

(Choice A) Asbestosis can present with dyspnea on exertion, but chest x-ray is more likely to
reveal an interstitial pattern of involvement most prominent in the lower zones. Pleural
plaques may also be noted. Histology shows ferruginous bodies featuring fusiform rods with a
translucent asbestos center and a golden-brown iron coating.

(Choice B) Berylliosis may present with dyspnea and ill-defined nodular or irregular opacities
on chest x-ray. Histology reveals noncaseating epithelioid granulomas without obvious

Telegram Group Invite Link: https://t.me/joinchat/zyFOIg0j5goxY2E1


Telegram Group Invite Link: https://t.me/USMLEWorldStep1

associated particles.

(Choice C) Coal worker's pneumoconiosis can present with exertional dyspnea and nodular
interstitial opacities on chest x-ray. Histology of nodal and perilymphatic lung tissue shows
accumulations of black, carbon-laden macrophages (coal macules).
(Choice D) Inhalation of organic dusts can cause hypersensitivity pneumonitis. Patients have
exertional dyspnea and diffuse nodular interstitial infiltrates on chest x-ray. Histology of nodal
tissue may reveal noncaseating granulomas.

Telegram Group Invite Link: https://t.me/joinchat/zyFOIg0j5goxY2E1


https://t.me/joinchat/zyFOIg0j5goxY2E1

Telegram Group Invite Link: https://t.me/USMLEWorldStep1

Cystic
fibrosis

Cystic fibrosis (CF) is a multisystem disorder in which a defective chloride channel (CF
transmembrane conductance regulator) prevents the normal hydration of mucus. Accumulation
of thick, viscous secretions in the lungs leads to mucus plugging and progressive
bronchiectasis, which manifest as obstructive lung disease (eg, hyperinflated lungs, barrel

Telegram Group Invite Link: https://t.me/joinchat/zyFOIg0j5goxY2E1


Telegram Group Invite Link: https://t.me/USMLEWorldStep1

chest) and chronic hypoxia (eg, digital clubbing). Gastrointestinal involvement can also lead to
malabsorption with decreased body weight.
The abnormal airway secretions in patients with CF impair mucociliary clearance and
allow colonization of bacteria such as Haemophilus influenzae, Staphylococcus aureus, and,
over time, gram-negative rods such as Pseudomonas aeruginosa and Burkholderia
cepacia complex.
The microenvironment is particularly suitable to mucoid P aeruginosa because localized hypoxia
within the airway mucus causes the bacterium to lose motility and produce alginate, a
polysaccharide involved in biofilm formation. The biofilm acts as a protective matrix for the
development of Pseudomonas macrocolonies, which are difficult to eradicate and cause

Telegram Group Invite Link: https://t.me/joinchat/zyFOIg0j5goxY2E1


https://t.me/joinchat/zyFOIg0j5goxY2E1

Telegram Group Invite Link: https://t.me/USMLEWorldStep1

recurrent and persistent infection.

Telegram Group Invite Link: https://t.me/joinchat/zyFOIg0j5goxY2E1


Telegram Group Invite Link: https://t.me/USMLEWorldStep1

(Choice A) Natural killer cell deficiency is associated with severe, disseminated viral infections
(eg, herpes simplex virus, cytomegalovirus). This patient's infiltrate in the right lower lobe and
Gram stain findings are consistent with bacterial pneumonia, and she has no history of severe
or recurrent viral infection.
(Choice C) Neutrophil free radical formation is impaired in chronic granulomatous disease,
which is characterized by recurrent multiorgan (eg, lungs, liver, lymph nodes, skin) infections by
catalase-positive organisms (eg, Aspergillus, S aureus). In contrast, this patient has isolated,
chronic pulmonary findings. Moreover, CF results in excessive neutrophil free radical formation
due to persistent pulmonary infection, which contributes to the progressive lung damage seen in
these patients.
(Choice D) Inability to form the membrane attack complex occurs with terminal complement
deficiency. These patients have recurrent Neisseria meningitis, not lung infections.
(Choice E) Bacterial colonization in the lungs of patients with CF induces a massive infiltration
of neutrophils, which results in the release of elastase, a protease that promotes
bronchiectasis. Therefore, chronic lung disease in CF is characterized by excessive, not
reduced, endobronchial protease activity.
SARCOIDOSIS

Telegram Group Invite Link: https://t.me/joinchat/zyFOIg0j5goxY2E1


https://t.me/joinchat/zyFOIg0j5goxY2E1

Telegram Group Invite Link: https://t.me/USMLEWorldStep1

This patient's hilar adenopathy, hypercalcemia, and elevated ACE levels are suggestive
of sarcoidosis, a condition characterized histologically by noncaseating
granulomas composed of activated macrophages and T lymphocytes. Activated macrophages
can ectopically produce ACE and 1-alpha-hydroxylase (increases production of 1,25-
dihydroxycholecalciferol, the active form of vitamin D). This results in high ACE levels and
hypercalcemia commonly seen in patients with sarcoidosis. Any organ can be affected;
however, the lungs (interstitial lung disease), lymph nodes (hilar adenopathy), skin (nodular
rash), and eyes (anterior uveitis) are most commonly involved.
Sarcoidosis may resemble other interstitial lung diseases (eg, hypersensitivity pneumonitis,
cryptogenic organizing pneumonia). In such cases, quantification of the CD4+/CD8+ ratio in
bronchoalveolar lavage (BAL) fluid may help in determining the diagnosis. Sarcoidosis is
a CD4+ T-cell–mediated disease in which large amounts of CD4+ lymphocytes release
interferon-gamma and TNF-alpha to drive macrophage activation and granuloma

Telegram Group Invite Link: https://t.me/joinchat/zyFOIg0j5goxY2E1


Telegram Group Invite Link: https://t.me/USMLEWorldStep1

formation. Therefore, the BAL fluid in patients with pulmonary sarcoidosis demonstrates a
lymphocytic predominance with a high CD4+/CD8+ ratio (>2:1).
(Choice B) CD8+ cells predominate in the BAL fluid of patients with hypersensitivity
pneumonitis. Although this patient has an exposure risk for hypersensitivity pneumonitis (bird
handling), her hilar adenopathy as well as elevated serum calcium and ACE levels make
sarcoidosis more likely.
(Choice C) CD19, CD20, and CD22 are markers for the B-cell lineage. Precursor B-cell
lymphoblastic leukemia (CD22+) may sometimes involve the pleura or present as diffuse, patchy
pulmonary infiltrates. In such cases, the BAL fluid might contain CD22+ lymphocytes; however,
elevated ACE and calcium levels suggest sarcoidosis.
(Choice D) BAL fluid eosinophilia is found in eosinophilic pneumonias and tropical pulmonary
eosinophilia. Although patients with hypersensitivity pneumonitis may have elevations in
circulating eosinophils, they do not have elevated eosinophil counts in their BAL fluid.
(Choice E) Even though the quantity of mast cells in BAL fluid may be elevated in various
pulmonary diseases, there are few, if any, conditions reported to have a predominance of such
cells.
(Choice F) Neutrophils would predominate in patients with bacterial pneumonias.
CYSTIC FIBROSIS

Telegram Group Invite Link: https://t.me/joinchat/zyFOIg0j5goxY2E1


https://t.me/joinchat/zyFOIg0j5goxY2E1

Telegram Group Invite Link: https://t.me/USMLEWorldStep1

This patient's fatal, massive hemoptysis is suggestive of hemorrhage from the bronchial
arteries. A common underlying cause of massive hemoptysis is bronchiectasis, as indicated
by this patient's autopsy showing thick, widened airways and mucus plugging. Bronchiectasis
is often associated with cystic fibrosis and immunodeficiency disorders, which causes chronic
and recurrent airway inflammation and infection. This stimulates neovascularization
and bronchial artery hypertrophy. The dilated, fragile, and tortuous vessels are in close
proximity to the bronchioles and prone to rupture with increased airway pressure, such as
during coughing (eg, acute pulmonary exacerbation).
Normally, blood supply to the lungs consists of the pulmonary circulation and the bronchial
circulation. Pulmonary arteries deliver deoxygenated blood from the right ventricle to the lung
parenchyma (ie, respiratory bronchioles to alveoli) to be oxygenated and returned to the heart;
bronchial arteries arise from the aorta and supply the endobronchial tree (ie, bronchi to terminal
bronchioles) with oxygenated blood. Although bleeding from the low-pressure pulmonary
circulation is more common, hemorrhage from the high-pressure, systemic bronchial

Telegram Group Invite Link: https://t.me/joinchat/zyFOIg0j5goxY2E1


https://t.me/joinchat/zyFOIg0j5goxY2E1

USMLE e e r m h nne s
Me est i e s: https://t.me/USMLEMe est

K p n ssr m i e s: https://t.me/USMLEK p n

USMLE i e s Step : https://t.me/USMLE Step

e ker i e s: https://t.me/USMLE e ker

r s e n i e s Step : https://t.me/USMLE r sAn e n

r s e n i e s Step K: https://t.me/USMLE r sAn e n Step K

Ph se i e s: https://t.me/USMLEPh se

Pi ri e i e s: https://t.me/USMLEPi ri e

P th m i e s: https://t.me/USMLEP th m

nA i s i h it : https://t.me/USMLE nA i s

Am ss r p: https://t.me/USMLEAm ss

Le t ri nk: https://t.me/USMLELe t ri

En p int P s: https://t.me/USMLEEn p int

USMLE MEs: https://t.me/USMLE ME

L E Me E : https://t.me/USMLE n ineMe E

USMLE r Step : https://t.me/USMLE r Step

USMLE r Step K: https://t.me/USMLE r Step K

USMLE r Step nk: https://t.me/USMLE r Step

USMLE Anki: https://t.me/USMLEAnki

KissPrep: https://t.me/USMLEKissPrep
Telegram Group Invite Link: https://t.me/USMLEWorldStep1

circulation is usually responsible for massive, life-threatening hemoptysis.

(Choices B and C) Hemoptysis due to hypertrophied pulmonary arteries or diffuse alveolar


hemorrhage (ie, alveolar capillary vasculitis) involves bleeding from the pulmonary circulation,
which is a low-pressure system that is unlikely to result in life-threatening hemoptysis. In
addition, bronchiectasis is not associated with either condition.
(Choice D) A fistula between the tracheobronchial tree and aorta is usually fatal and can occur
with chronic vascular or airway inflammation, typically due to an implanted medical device or

Telegram Group Invite Link: https://t.me/joinchat/zyFOIg0j5goxY2E1


https://t.me/joinchat/zyFOIg0j5goxY2E1

Telegram Group Invite Link: https://t.me/USMLEWorldStep1

stent. A fistula is an uncommon cause of hemoptysis that would not cause dilated bronchial
walls on pathology.
(Choice E) Carcinoid tumors are highly vascular endobronchial tumors that commonly present
with hemoptysis. However, autopsy would show a discrete mass (not bronchiectasis).
Succinylcholine

Train-of-four (TOF) stimulation is used during anesthesia to assess the degree of paralysis
induced by neuromuscular junction (NMJ)-blocking agents. A peripheral nerve is stimulated 4
times in quick succession and the muscular response is recorded. The height of each bar
represents the strength of each twitch; higher bars indicate the activation of increasing numbers
of individual muscle fibers (myocytes).
When a nondepolarizing NMJ blocker (eg, vecuronium) is administered, competitive inhibition
of postsynaptic acetylcholine receptors at the motor endplate prevents some of these fibers
from activating, decreasing the strength of the twitch. TOF stimulation shows a progressive
reduction in each of the 4 responses (fading pattern) as a result of less acetylcholine being
released with each subsequent impulse (due to the additional effect of presynaptic acetylcholine
receptor blockade). In contrast, depolarizing blockers (eg, succinylcholine) initially function
by preventing repolarization of the motor endplate and show equal reduction of all 4 twitches
during TOF stimulation (phase I blockade). The responses remain equal because the
presynaptic acetylcholine receptor stimulation helps to mobilize presynaptic acetylcholine
vesicles for release. Persistent exposure to succinylcholine results in eventual transition
to phase II blockade as the acetylcholine receptors become desensitized and inactivated (ie,
functionally similar to nondepolarizing blockade).
Succinylcholine is commonly administered for rapid-sequence intubation due to its rapid onset
(<1 minute). The duration of action is determined by its metabolism by plasma cholinesterase
and is typically <10 minutes. However, some patients are homozygous for an atypical plasma
cholinesterase, which breaks down succinylcholine more slowly. In these patients, the paralysis
can last for hours and they must be maintained on mechanical ventilation until spontaneous
respirations resume.
(Choice A) Dantrolene relaxes skeletal muscle by reducing the release of Ca2+ from the
sarcoplasmic reticulum. Dantrolene is used to treat malignant hyperthermia and neuroleptic
malignant syndrome. It is not routinely used as a neuromuscular paralytic agent.

Telegram Group Invite Link: https://t.me/joinchat/zyFOIg0j5goxY2E1


Telegram Group Invite Link: https://t.me/USMLEWorldStep1

(Choice B) Benzodiazepines are effective for sedation, but they have no direct action at the
NMJ and do not provide sufficient muscle paralysis to facilitate intubation.
(Choices C and E) Pancuronium and tubocurarine are nondepolarizing NMJ blockers. Unlike
depolarizing NMJ drugs, these agents do not function in distinct phases and their TOF
responses always display a fading pattern. Neostigmine administration reverses
nondepolarizing NMJ blockade.

Telegram Group Invite Link: https://t.me/joinchat/zyFOIg0j5goxY2E1


https://t.me/joinchat/zyFOIg0j5goxY2E1

Telegram Group Invite Link: https://t.me/USMLEWorldStep1

asthma

Telegram Group Invite Link: https://t.me/joinchat/zyFOIg0j5goxY2E1


Telegram Group Invite Link: https://t.me/USMLEWorldStep1

This patient's frequent episodes of shortness of breath, chest tightness, and cough complicated
by respiratory failure requiring hospitalization is consistent with asthma. The mainstays of
medication therapy for asthma include an as-needed short-acting beta agonist (eg, albuterol),
an inhaled corticosteroid (eg, mometasone), and a long-acting beta agonist (eg,
salmeterol). For patients whose symptoms remain uncontrolled after such therapy is optimized,
the addition of an IgE-binding monoclonal antibody (eg, omalizumab) may be beneficial.
Many patients with asthma have high levels of circulating IgE antibodies generated against
environmental antigens (eg, dust mites, animal dander). Omalizumab binds to the IgE antibody
heavy chain and blocks the binding of IgE to the IgE receptors on mast cells. By preventing this
binding, omalizumab inhibits proinflammatory substance release (including histamine and
leukotrienes) by mast cells, markedly reducing airway inflammation.
(Choice A) Decreasing airway collagen deposition may help slow the progression of idiopathic
pulmonary fibrosis (IPF). Certain tyrosine kinase inhibitors (eg, nintedanib) decrease fibroblast
proliferation and have shown benefit in treating IPF.
(Choice C) Decreasing alveolar destruction would be helpful in treating chronic obstructive
pulmonary disease (COPD). Smoking cessation is the most effective way to prevent and/or
slow the rate of progression of COPD.
(Choice D) Beta-2 agonists (eg, albuterol, salmeterol) cause bronchodilation via cyclic AMP–
mediated smooth muscle relaxation and are effective symptomatic treatments for
asthma. These drugs do not significantly reduce airway inflammation.
(Choice E) Increasing pulmonary vasodilation helps improve pulmonary arterial hypertension
and can be achieved with several classes of drugs, including endothelin receptor antagonists
(eg, bosentan), phosphodiesterase inhibitors (eg, sildenafil), and prostacyclin analogs.

Telegram Group Invite Link: https://t.me/joinchat/zyFOIg0j5goxY2E1


https://t.me/joinchat/zyFOIg0j5goxY2E1

Telegram Group Invite Link: https://t.me/USMLEWorldStep1

Septic
emboli

Telegram Group Invite Link: https://t.me/joinchat/zyFOIg0j5goxY2E1


Telegram Group Invite Link: https://t.me/USMLEWorldStep1

Telegram Group Invite Link: https://t.me/joinchat/zyFOIg0j5goxY2E1


https://t.me/joinchat/zyFOIg0j5goxY2E1

Telegram Group Invite Link: https://t.me/USMLEWorldStep1

The lung specimen above shows multiple wedge-shaped hemorrhagic infarcts in the periphery
of the lung, which, given this patient's presentation, are most likely due to septic pulmonary
emboli.
Patients with intravenous drug use are at increased risk of developing tricuspid valve
endocarditis, most commonly due to Staphylococcus aureus. The clinical presentation is
typically acute with high-grade fever, and tachycardia and hypotension due to septic shock. The
majority of patients with tricuspid valve endocarditis experience embolization of tricuspid valve
vegetation fragments; these fragments lodge in distal portions of the pulmonary arterial tree and
cause septic pulmonary infarcts. The infarcts are typically wedge-shaped due to the triangular
perfusion field of small arteries at the lung periphery. Due to the relatively low density of lung
tissue (which allows blood seepage into tissue during infarction) and a dual blood supply (ie,
pulmonary and bronchial arteries), pulmonary infarcts are typically hemorrhagic (red) rather
than ischemic (white).
(Choice A) Miliary tuberculosis typically has a subacute or chronic presentation and is
accompanied by night sweats, weight loss, and anorexia. Innumerable, small, white nodules
resembling millet seeds are typically seen; peripherally located wedge-shaped hemorrhages in
the lung are not characteristic.
(Choice B) Mycotic aneurysms may occur as a complication of endocarditis; however, they
refer to the local destruction and dilation of an arterial wall due to infection. Despite their name,
mycotic aneurysms are very rarely associated with fungal infections.
(Choice C) Small airway obstruction is characteristic of asthma, which typically presents with
dyspnea, cough, wheezing, and chest tightness. Lung autopsy usually reveals the occlusion of
bronchi and bronchioles by thick mucous plugs containing shed epithelium.
(Choice E) Venous thromboembolism in the pulmonary vasculature (ie, thrombotic pulmonary
embolism) can also cause wedge-shaped hemorrhagic infarct in the periphery of the
lung. However, this patient's high-grade fever and recent intravenous drug use are more
suggestive of septic emboli from tricuspid valve endocarditis. In addition, multiple pulmonary
infarcts are more consistent with septic embolization than venous thromboembolism.

Asbestosis

Telegram Group Invite Link: https://t.me/joinchat/zyFOIg0j5goxY2E1


Telegram Group Invite Link: https://t.me/USMLEWorldStep1

This nonsmoking patient with dyspnea, cough, and weight loss has pleural thickening on
imaging; gross pathology of the lungs demonstrates an encasement of the lungs in a rind-like,
calcified mass. This presentation is consistent with mesothelioma, a rare, malignant neoplasm
arising from the mesothelial cells that line body cavities (eg, pleural, peritoneal, pericardial).
In early mesothelioma, multiple nodules form on the parietal pleura and gradually grow to
encase the lung parenchyma (which is typically uninvolved). Pleural effusions are common and
Telegram Group Invite Link: https://t.me/joinchat/zyFOIg0j5goxY2E1
https://t.me/joinchat/zyFOIg0j5goxY2E1

Telegram Group Invite Link: https://t.me/USMLEWorldStep1

frequently hemorrhagic. Mesotheliomas typically present with slowly progressive dyspnea,


cough, chest pain, and weight loss. Asbestos exposure is the primary risk factor; individuals
with occupational exposure (eg, mining, shipbuilding, insulation installation) are at increased
risk.
Histologically, mesothelioma can appear as cuboidal or flattened cells (epithelium-like)
or spindle cells (stromal-like). Immunohistochemistry is an important diagnostic marker; nearly
all cases of mesothelioma will stain positive for cytokeratins and many will also typically stain
positive for calretinin. Electron microscopy usually shows polygonal tumor cells with numerous
long, slender microvilli and abundant tonofilaments.
(Choice A) Tuberculosis is characterized by caseating granulomas with giant cell formation;
patients typically have hilar adenopathy and upper lobe infiltrates. Tuberculosis occurs most
commonly in immunosuppressed patients or those at high exposure risk (eg, health care
workers, incarcerated individuals).
(Choice B) Silicosis is associated with inhaled silica (eg, mining, sand blasting); histology
demonstrates nodules composed of whorled collagen fibers and dust-laden
macrophages. Imaging demonstrates innumerable upper lobe–predominant nodes within the
pulmonary parenchyma.
(Choice C) Small cell lung cancer is characterized by small round cells; however, this
malignancy typically arises in the central airways (not the pleura). Neuroendocrine markers,
including chromogranin and synaptophysin, are typically positive.
(Choice E) Empyema—composed of a thick, fibrinous exudate and pus—typically occurs when
bacteria infect the pleural space (typically from an associated pneumonia). Patients often
present with fever, chills, and (occasionally) sepsis, not months of weight loss and dyspnea.
Ipratropium
The figure on the left shows a decrease in bronchial diameter of normal lungs following
administration of an irritant but minimal change in bronchial diameter of lungs without an intact
vagus nerve. This implies that the bronchoconstriction is mediated by
the parasympathetic nervous system. When the vagus nerve is stimulated (eg, via an inhaled
irritant), acetylcholine is released and produces bronchoconstriction by acting on muscarinic
receptors. The figure on the right demonstrates minimal decrease in bronchial diameter in
normal lungs following administration of the irritant, suggesting that the study medication blocks
the action of acetylcholine. Ipratropium, an anticholinergic and derivative of atropine, has this
effect.
Ipratropium and other anticholinergic drugs used for obstructive lung disease (eg, tiotropium)
are less potent bronchodilators than beta-2 adrenergic agonists but can enhance the
bronchodilatory effect of these agents. In addition, ipratropium and similar drugs block the
parasympathetic stimulation of tracheobronchial submucosal glands, decreasing mucus
secretion and providing further benefit in treating asthma and chronic obstructive pulmonary
disease (COPD).
(Choice A) Albuterol is a short-acting selective beta-2 adrenergic agonist that is the first-line
treatment for asthma and COPD. The bronchodilatory effects of albuterol are independent of
parasympathetic innervation.
(Choice B) Inhaled glucocorticoids (eg, flunisolide) have potent local anti-inflammatory effects
in the airway and are used in the treatment of persistent asthma and COPD with frequent
exacerbations.

Telegram Group Invite Link: https://t.me/joinchat/zyFOIg0j5goxY2E1


Telegram Group Invite Link: https://t.me/USMLEWorldStep1

(Choice D) Nifedipine causes bronchodilation by blocking calcium influx into bronchial smooth
muscle cells; however, the drug is generally not used in treating obstructive lung disease.
(Choice E) Methylxanthines, such as theophylline and aminophylline, cause bronchodilation by
blocking phosphodiesterase activity, thereby increasing the intracellular concentration of cyclic
AMP.
(Choice F) Zileuton inhibits 5-lipoxygenase and thereby decreases the production of
leukotrienes to reduce bronchoconstriction and improve asthma symptoms.

AAT DEFICENCY
Neutrophil elastase is released by both neutrophils and macrophages and is the primary
protease responsible for extracellular elastin degradation. The major serum inhibitor of elastase
is alpha-1 antitrypsin (AAT). In this study, the patients whose serum is unable to inhibit elastin
degradation likely have AAT deficiency, a condition characterized by early-onset panacinar
emphysema due to the unopposed action of neutrophil elastase on alveolar walls.
Tobacco exposure dramatically accelerates the development of emphysema in patients with
AAT deficiency by inducing inflammation (increasing neutrophil and macrophage activation) and
permanently inactivating the already low quantities of AAT through oxidation of a crucial
methionine residue. Therefore, patients with AAT deficiency should be counseled to avoid
smoking and exposure to second-hand smoke.
(Choice A) Patients with hypertrophic cardiomyopathy are at risk for sudden cardiac arrest with
exertion and are typically counseled to avoid participation in most competitive sports.
(Choice B) Patients with chronic granulomatous disease (impaired phagocytic intracellular
killing due to a defect in nicotinamide adenine dinucleotide phosphate [NADPH] oxidase) should
avoid certain immunosuppressive agents (eg, tumor necrosis factor-alpha blockers) due to risk
of overwhelming infection.
(Choice D) Periodic aortic diameter monitoring is indicated in patients with Marfan syndrome
(impaired fibrillin synthesis), who are at risk for both aortic dissection and chronic aortic
regurgitation due to aortic root dilation.
(Choice E) Patients with osteogenesis imperfecta (decreased production of type 1 collagen)
are at high risk of developing osteoporosis and should undergo regular bone density monitoring.
(Choice F) Patients with Ehlers-Danlos syndrome (impaired collagen synthesis) are at risk for
scleral fragility, ocular globe rupture, and retinal detachment and should undergo routine
ophthalmologic exams.
Left sided heart failure
This patient's dyspnea, orthopnea, bibasilar crackles, and S3 suggest left-sided heart
failure due to recent myocardial infarction (MI). Atherosclerotic plaque rupture in the left
anterior descending or left circumflex artery can infarct large areas of the left ventricular
myocardium, leading to decreased contractility and reduced cardiac output. This can result
in elevation of end-diastolic pressure that impairs diastolic return from the pulmonary veins,
leading to increased hydrostatic pressure in the pulmonary capillaries with transudation of fluid
into the lung parenchyma (pulmonary edema).
Pulmonary edema in the alveoli impairs ventilation and causes intrapulmonary shunting. In
addition, edema in the lung interstitium makes the lungs heavy and stiff, restricting pulmonary

Telegram Group Invite Link: https://t.me/joinchat/zyFOIg0j5goxY2E1


https://t.me/joinchat/zyFOIg0j5goxY2E1

Telegram Group Invite Link: https://t.me/USMLEWorldStep1

expansion during inspiration. This decreased lung compliance can mimic that seen with other
causes of restrictive lung physiology (eg, interstitial lung disease).
(Choice A) Surfactant decreases the surface tension of the fluid layer that lines alveolar cells,
helping prevent alveolar collapse. Pulmonary edema dilutes pulmonary surfactant, increasing
both alveolar surface tension and the tendency for alveolar collapse.
(Choice C) Ventilation-perfusion (V/Q) mismatch exists on a spectrum with intrapulmonary
shunting (perfusion without ventilation) and dead space ventilation (ventilation without perfusion)
on either end. Pulmonary edema causes V/Q mismatch via increased intrapulmonary shunting
with a corresponding decrease in dead space ventilation. V/Q mismatch via increased dead
space ventilation is seen with pulmonary embolism.
(Choice D) Functional residual capacity is the volume of air in the lungs at the end of normal
expiration. It is increased in conditions with increased lung compliance (eg, chronic obstructive
pulmonary disease) and decreased in conditions such as pulmonary edema that reduce lung
compliance.
(Choice E) Interventricular septal rupture is a mechanical complication of MI that causes left-to-
right cardiac shunting. However, this complication usually occurs within 3-5 days following MI
and precipitates a dramatic and acute, rather than gradual, presentation of heart failure.
Right sided heart failure
This patient with chronic obstructive pulmonary disease (COPD) has evidence of pulmonary
hypertension complicated by right-sided heart failure. Hypoxic vasoconstriction is a
physiologic mechanism unique to lung tissue that helps minimize ventilation-perfusion mismatch
and increase overall gas exchange efficiency. However, chronic and diffuse pulmonary
vasoconstriction, which occurs with advanced hypoxic lung disease (eg, COPD, interstitial
lung disease, obesity hypoventilation syndrome), and can lead to pulmonary
hypertension (ie, increased right ventricular afterload).
Pulmonary hypertension typically presents with dyspnea and/or fatigue, and some patients may
develop exertional angina or syncope due to reduced cardiac output. Physical examination can
reveal left parasternal lift (due to right ventricular hypertrophy) and a loud pulmonic
component of S2 due to high pulmonary artery pressure. Over time, the right ventricle may be
unable to pump against the increased afterload and right-sided heart failure can develop,

Telegram Group Invite Link: https://t.me/joinchat/zyFOIg0j5goxY2E1


Telegram Group Invite Link: https://t.me/USMLEWorldStep1

evidenced by jugular venous distension and prominent lower extremity edema.

(Choices A, B, and D) The hemodynamic parameters in pulmonary hypertension secondary to


hypoxic lung disease are similar to those in pulmonary arterial hypertension. Left ventricular
preload and stroke volume (cardiac output) are decreased due to impaired pumping of blood
through the lungs to the left atrium. Pulmonary capillary wedge pressure, an estimate of left
atrial pressure and a representation of left-sided preload, is also decreased.

Telegram Group Invite Link: https://t.me/joinchat/zyFOIg0j5goxY2E1


https://t.me/joinchat/zyFOIg0j5goxY2E1

Telegram Group Invite Link: https://t.me/USMLEWorldStep1

(Choice C) Pulmonary arterial compliance decreases with pulmonary hypertension due to


vascular remodeling and stiffening of the pulmonary arterial wall.
Asthma

This patient's acute-onset shortness of breath and chest tightness are consistent with
the airway spasm and bronchoconstriction that occurs in asthma. She received
a bronchodilator to reduce airway resistance and increase airway flow. In asthma,
bronchodilation is typically accomplished with a beta-2 receptor agonist (eg, albuterol); these
drugs work by stimulating beta-2 adrenergic receptors on bronchial smooth muscle cells,
leading to an increase in cyclic AMP and consequent smooth muscle relaxation (decrease in
smooth muscle tone).

(Choices A and B) Corticosteroids, both inhaled (eg, budesonide, fluticasone) and systemic
(eg, prednisone), modify gene transcription to reduce the airway inflammatory response in
patients with asthma. Corticosteroids are also used to stimulate alveolar surfactant production
and reduce the risk of neonatal respiratory distress syndrome in premature infants.
(Choice D) Inhaled antibiotics (eg, tobramycin) are used to reduce lower-airway bacterial load
in patients with cystic fibrosis.
(Choice E) Several medications, including endothelin receptor antagonists (eg, bosentan),
phosphodiesterase inhibitors (eg, sildenafil), and prostacyclin analogs, are used to lower

Telegram Group Invite Link: https://t.me/joinchat/zyFOIg0j5goxY2E1


Telegram Group Invite Link: https://t.me/USMLEWorldStep1

pulmonary vascular resistance in patients with pulmonary arterial hypertension.

High altitude illness

Telegram Group Invite Link: https://t.me/joinchat/zyFOIg0j5goxY2E1


https://t.me/joinchat/zyFOIg0j5goxY2E1

Telegram Group Invite Link: https://t.me/USMLEWorldStep1

This patient with dyspnea, hypoxemia, and patchy alveolar infiltrates shortly after arriving at high
altitude likely has high-altitude pulmonary edema (HAPE). HAPE can occur within several
days of arrival at an altitude >2,500 m (~8,000 ft), with the risk increasing as altitude
increases. The pathophysiology is driven by the reduced partial pressure of oxygen (PiO2) at
high altitude, which leads to hypoxic pulmonary vasoconstriction and consequent increased
pulmonary arterial pressure. In some individuals with genetic predisposition, the hypoxic
vasoconstriction is unevenly distributed, exposing the capillary beds in relatively less
vasoconstricted areas to high perfusion pressure. This causes alveolar-capillary membrane
disruption and leads to patchy, bilateral pulmonary edema.
HAPE typically presents with dyspnea (progressive over 1-2 days) and cough (sometimes with
hemoptysis). Accompanying symptoms of acute mountain sickness (eg, headache, nausea)
are often present, and physical examination reveals bilateral lung
crackles. The hypoxemia characteristically improves with supplemental oxygen (likely due
to alleviation of pulmonary vasoconstriction), which differentiates HAPE from other causes of
diffuse pulmonary edema. Although potentially fatal, HAPE usually resolves rapidly upon

Telegram Group Invite Link: https://t.me/joinchat/zyFOIg0j5goxY2E1


Telegram Group Invite Link: https://t.me/USMLEWorldStep1

descent to lower altitudes.

(Choice B) Acute pulmonary embolism causes dyspnea and hypoxemia; however, jugular
venous distension and an absence of pulmonary edema (eg, no ausculatory crackles, clear
lungs on chest x-ray) are also expected.
(Choice C) Hyperventilation leading to reduced PaCO2 and respiratory alkalosis is expected at
high altitude. An extreme drop in PaCO2 can cause severe alkalosis with associated tingling,
numbness, tetany, and possibly seizures, but it does not cause pulmonary edema.
(Choice D) Left ventricular function is unaffected in HAPE (the pulmonary edema is
noncardiogenic). Myocardial infarction can lead to acute left ventricular dysfunction with
pulmonary edema, but it would be expected to present with severe chest pain and to cause
jugular venous distension.
(Choice E) In response to the reduced PiO2 at high altitude, the body increases erythrocyte
production to increase O2-carrying capacity. However, this secondary polycythemia does not
contribute to pulmonary edema.

Telegram Group Invite Link: https://t.me/joinchat/zyFOIg0j5goxY2E1


https://t.me/joinchat/zyFOIg0j5goxY2E1

Telegram Group Invite Link: https://t.me/USMLEWorldStep1

COPD

Smoking is the strongest risk factor for chronic obstructive pulmonary


disease (COPD). Cigarette smoke contains numerous toxins (eg, formaldehyde, cadmium)
that impair ciliary function (contributing to chronic bronchitis) and cause oxidative cellular
damage (contributing to centrilobular emphysema). These changes are responsible for the
airway obstruction and air-trapping that cause the reduction in forced expiratory volume in 1
second (FEV1) that characterizes COPD.
Beginning at approximately age 30, there is a gradual decline in FEV1 with age. Smoking-
induced lung damage in COPD rapidly accelerates this normal age-related decline in FEV1, and
once the decline occurs it is not recoverable. However, smoking cessation can markedly slow
the decline in FEV1 in patients with COPD, an improvement that is typically apparent within the
first year of smoking cessation.
Following smoking cessation, the trajectory of this patient's FEV1 decline will improve compared
with if he had continued to smoke, but his FEV1 will not return to the level it would have been
had he never smoked. This trajectory is best represented by curve C.

Telegram Group Invite Link: https://t.me/joinchat/zyFOIg0j5goxY2E1


https://t.me/joinchat/zyFOIg0j5goxY2E1

USMLE e e r m h nne s
Me est i e s: https://t.me/USMLEMe est

K p n ssr m i e s: https://t.me/USMLEK p n

USMLE i e s Step : https://t.me/USMLE Step

e ker i e s: https://t.me/USMLE e ker

r s e n i e s Step : https://t.me/USMLE r sAn e n

r s e n i e s Step K: https://t.me/USMLE r sAn e n Step K

Ph se i e s: https://t.me/USMLEPh se

Pi ri e i e s: https://t.me/USMLEPi ri e

P th m i e s: https://t.me/USMLEP th m

nA i s i h it : https://t.me/USMLE nA i s

Am ss r p: https://t.me/USMLEAm ss

Le t ri nk: https://t.me/USMLELe t ri

En p int P s: https://t.me/USMLEEn p int

USMLE MEs: https://t.me/USMLE ME

L E Me E : https://t.me/USMLE n ineMe E

USMLE r Step : https://t.me/USMLE r Step

USMLE r Step K: https://t.me/USMLE r Step K

USMLE r Step nk: https://t.me/USMLE r Step

USMLE Anki: https://t.me/USMLEAnki

KissPrep: https://t.me/USMLEKissPrep
Telegram Group Invite Link: https://t.me/USMLEWorldStep1

(Choice A) It is not physiologically possible for FEV1 to increase with age in either smokers or
nonsmokers.
(Choice B) The FEV1 does not remain constant after age 30, even in nonsmokers.
(Choice D) With smoking cessation, the trajectory of this patient's FEV1 decline will improve
rather than stay the same.
(Choice E) This patient would experience a more rapid decline in FEV1 if he were to increase
his rate of smoking.

Telegram Group Invite Link: https://t.me/joinchat/zyFOIg0j5goxY2E1


https://t.me/joinchat/zyFOIg0j5goxY2E1

Telegram Group Invite Link: https://t.me/USMLEWorldStep1

Pneumothorax

This patient's chest x-ray shows no peripheral lung markings on the right side and a tissue mass
representing the collapsed right lung, findings consistent with pneumothorax; the stab wound
likely breached the chest wall and pleura, allowing air to enter the intrapleural space.
The lungs exert collapsing force at all lung volumes, and the chest wall exerts expanding
force at all but very high lung volumes. In the normal respiratory system, the lung and chest
wall are in equilibrium (represented by the blue dot) at the functional residual capacity (FRC),
and they move in concert throughout the respiratory cycle with the opposing forces working
together to maintain negative intrapleural pressure.
When an abnormal communication pathway is created between the lung (or exterior chest) and
the intrapleural space, intrapleural pressure equalizes with alveolar (ie, atmospheric) pressure,
resulting in loss of intrapleural negative pressure. With the lung and chest wall no longer
pulling each other in opposite directions, the chest wall springs outward to its equilibrium
position (red dot), increasing hemithorax volume.

Telegram Group Invite Link: https://t.me/joinchat/zyFOIg0j5goxY2E1


Telegram Group Invite Link: https://t.me/USMLEWorldStep1

Loss of negative intrapleural pressure also causes the lung to collapse toward its equilibrium
position (black dot). Compliance is defined as change in volume per change in pressure and is
represented by the slope of the pressure-volume curve (eg, high compliance is illustrated by a
steep-sloping curve). Lung compliance demonstrates hysteresis (ie, compliance differs during
inspiration and expiration) due to the effects of alveolar surface tension. The flatness of
the inspiratory lung compliance curve at very low lung volumes reflects the difficulty in
inflating a collapsed lung and represents decreased compliance compared to that at the FRC.

Telegram Group Invite Link: https://t.me/joinchat/zyFOIg0j5goxY2E1


https://t.me/joinchat/zyFOIg0j5goxY2E1

Telegram Group Invite Link: https://t.me/USMLEWorldStep1

Telegram Group Invite Link: https://t.me/joinchat/zyFOIg0j5goxY2E1


Telegram Group Invite Link: https://t.me/USMLEWorldStep1

Telegram Group Invite Link: https://t.me/joinchat/zyFOIg0j5goxY2E1


https://t.me/joinchat/zyFOIg0j5goxY2E1

Telegram Group Invite Link: https://t.me/USMLEWorldStep1

Pleural effusion

This patient's clinical findings are consistent with reactivation tuberculosis, given his
nonresponse to routine pneumonia treatment, prior residence in an endemic region (ie,
Southeast Asia), and CT scan showing calcified hilar adenopathy (representing granulomatous
lymph node involvement). Reactivation tuberculosis typically affects the upper lobes of the
lungs and is often associated with pleural effusion.
Pulmonary infection causes pleural effusion primarily via an inflammatory increase in vascular
and pleural membrane permeability, leading to relatively high pleural fluid protein and lactate
dehydrogenase (LDH) concentration (Choice D). Such effusions are exudative by Light
criteria, with pleural fluid/serum protein ratio >0.5, pleural fluid/serum LDH ratio >0.6, and/or
pleural fluid LDH >⅔ the upper limit of normal serum LDH. Normal serum total protein ranges
from 6.0 g/dL to 7.8 g/dL; therefore, pleural fluid protein >4 g/dL (ratio >0.5) is most likely in

Telegram Group Invite Link: https://t.me/joinchat/zyFOIg0j5goxY2E1


Telegram Group Invite Link: https://t.me/USMLEWorldStep1

this patient.

(Choice A) Pleural fluid cytology showing atypical cells can be seen with a malignant pleural
effusion, but is not expected with tuberculous effusion. Malignant effusions are also typically
exudative by Light criteria.
(Choice B) An elevated pleural fluid triglyceride level is diagnostic of a chylothorax, which
results from disruption (eg, trauma, malignant invasion) of the thoracic duct leading to leakage
of chylomicron-rich lymphatic fluid into the pleural space.
(Choice E) Tuberculosis pleural effusions are typically lymphocyte predominant. Neutrophil
predominance is expected with a typical parapneumonic effusion (eg, due to community-
acquired bacterial pneumonia).
Pulmonary artery hypertension
Autopsy reveals significant thickening of the right ventricular free wall (almost 2 cm in
thickness; normal: 3-4 mm) compared to the left ventricle. This finding of severe right
ventricular hypertrophy in a young woman with progressive dyspnea and sudden death is
suggestive of pulmonary arterial hypertension (PAH).

Telegram Group Invite Link: https://t.me/joinchat/zyFOIg0j5goxY2E1


https://t.me/joinchat/zyFOIg0j5goxY2E1

Telegram Group Invite Link: https://t.me/USMLEWorldStep1

Pulmonary hypertension can be caused by a number of conditions, including chronic lung


disease and left heart failure. Pulmonary hypertension due to primary processes affecting the
lung vasculature is referred to as PAH, a condition that most often affects women ages 20-
40. PAH is characterized by intimal hyperplasia and fibrosis, medial hypertrophy, and formation
of capillary tufts.
Long-standing pulmonary hypertension eventually leads to hypertrophy and/or dilation of the
right ventricle (cor pulmonale). Patients typically present with progressive dyspnea, exertional
angina, or syncope. As PAH progresses, signs of right-sided heart failure (eg, hepatomegaly,
ascites, peripheral edema) can become prominent. The most common cause of death in
patients with PAH is right heart failure with circulatory collapse and respiratory failure.

Telegram Group Invite Link: https://t.me/joinchat/zyFOIg0j5goxY2E1


Telegram Group Invite Link: https://t.me/USMLEWorldStep1

Telegram Group Invite Link: https://t.me/joinchat/zyFOIg0j5goxY2E1


https://t.me/joinchat/zyFOIg0j5goxY2E1

Telegram Group Invite Link: https://t.me/USMLEWorldStep1

(Choices A and C) Aortic stenosis and essential hypertension cause left ventricular wall
thickening and hypertrophy, not right ventricular hypertrophy.
(Choice B) Dilated cardiomyopathy typically increases the size of one or both ventricular
chambers. This patient's ventricular cavities are small and normal in size, not
enlarged. Moreover, the left ventricle is relatively normal in shape (elliptical, with a long axis
approximately twice that of lateral short axis dimensions), which is not the case with left
ventricular dilation.
(Choice D) Myocardial infarctions occur more frequently in older patients (age
>65). Morphological changes in the myocardium include an area of reddish-blue discoloration
that evolves into a yellow-tan discrete infarct with eventual formation of a white scar.
(Choice F) Wolff-Parkinson-White (WPW) syndrome should be suspected in any case of
sudden cardiac death in an otherwise healthy young individual. WPW syndrome is an
electrophysiological abnormality of atrioventricular (AV) node conduction that is discernible on
histology as small accessory AV impulse conduction pathway(s) anatomically separate from the
AV node. There are no consistent gross morphological changes to the ventricles.
Pleural effusion

Under normal physiologic conditions, pleural fluid enters the pleural space from parietal pleural
microvessels and is removed by lymphatics at a constant rate. Pathologic states that disrupt
pleural capillary hydrostatic or oncotic pressure, decrease pleural space pressure, reduce
lymphatic drainage, or increase vascular membrane permeability can lead to pleural
effusion. Pleural effusions that develop due to pressure changes (eg, heart failure, cirrhosis,
nephrotic syndrome) are typically transudative, whereas those that develop due
to inflammation and consequent increased vascular membrane permeability (eg, infection,
malignancy, rheumatologic disease), are typically exudative.
The Light criteria are used to differentiate transudative and exudative pleural effusions and aid
in the differential diagnosis. Transudative effusions have a low fluid-to-serum ratio of total
Telegram Group Invite Link: https://t.me/joinchat/zyFOIg0j5goxY2E1
Telegram Group Invite Link: https://t.me/USMLEWorldStep1

protein and lactate dehydrogenase and low absolute levels of lactate dehydrogenase. In
contrast, exudative effusions have a high fluid-to-serum ratio of total protein
(>0.5) or lactate dehydrogenase (>0.6) or high absolute levels of lactate dehydrogenase (>2/3
the serum upper limit of normal).
This patient with a pleural fluid-to-serum total protein ratio >0.5 has an exudative effusion likely
due to pneumonia (ie, parapneumonic effusion).

(Choice A) Decreased plasma oncotic pressure occurs in conditions that cause


hypoalbuminemia (eg, cirrhosis, nephrotic syndrome) and can lead to transudative pleural
effusion.
(Choice B) Decreased thoracic lymphatic flow (eg, due to malignant involvement of lymphatic
ducts) can cause a chylous effusion (chylothorax). Although this type of effusion is usually
exudative by Light criteria, the fluid is typically milky white (rather than serosanguinous) in gross
appearance due to high triglyceride (ie, chylomicron) content.
(Choice C) Increased intraabdominal hydrostatic pressure occurs with abdominal ascites that
develops due to portal hypertension (eg, cirrhosis). The ascites can be forced into the right-
sided pleural cavity through fenestrations in the diaphragm, resulting in a transudative pleural
effusion known as hepatic hydrothorax.

Telegram Group Invite Link: https://t.me/joinchat/zyFOIg0j5goxY2E1


https://t.me/joinchat/zyFOIg0j5goxY2E1

Telegram Group Invite Link: https://t.me/USMLEWorldStep1

(Choice D) Increased intrapleural negative pressure occurs in large-volume atelectasis (lung


collapse) and can lead to a transudative pleural effusion.
(Choice F) Increased hydrostatic pressure in the pulmonary venous system occurs in heart
failure and leads to transudative pleural effusion.
Lung
abscess

This patient's excessive alcohol consumption, subacute pulmonary symptoms, and dense lung
opacity with an air-fluid level are consistent with lung abscess. Most cases are due
to aspiration of oropharyngeal flora into the alveoli during reduced levels of consciousness
from drug or alcohol use disorder (which impairs cough and gag reflexes) or episodes of
dysphagia (eg, previous stroke, esophageal stricture). Periodontal disease is frequently
present, which increases the concentration of anaerobic bacteria in the gingival
crevices. Facultative and strict anaerobic oral organisms such as Bacteroides, Prevotella,
Fusobacterium, and Peptostreptococcus are the common pathogens, but most lung abscesses
are polymicrobial.
Manifestations typically include subacute fever, cough with foul-smelling sputum, night
sweats, and weight loss; digital clubbing sometimes occurs. Diagnosis is usually made when
chest imaging reveals a dense fluid collection or cavitary infiltrate (often with an air-fluid level)
in dependent portions of the lung (eg, superior portion of lower lobe).

Telegram Group Invite Link: https://t.me/joinchat/zyFOIg0j5goxY2E1


Telegram Group Invite Link: https://t.me/USMLEWorldStep1

Treatment with ampicillin/sulbactam or a carbapenem is considered first-line therapy due to


excellent coverage of oral anaerobes and aerobic gram-positive organisms (eg, Streptococcus
pneumoniae). Clindamycin was used in the past but is now reserved for those with penicillin
allergies due to high risk of Clostridioides (formerly Clostridium) difficile infection.
(Choice A) Mycoplasma pneumoniae is a cell wall–deficient bacteria that causes atypical
pneumonia. Patients frequently have subacute cough and low-grade fever, but chest imaging
usually demonstrates bilateral reticulo-nodular opacities. The presence of foul-smelling sputum
and a cavitary infiltrate with an air-fluid level makes lung abscess more likely.
(Choice B) Although gram-negative intestinal flora (eg, Klebsiella pneumoniae) can cause lung
abscess, empiric treatment must target upper respiratory anaerobic organisms because they
are present in most lung abscesses.
(Choice C) Skin flora (eg, coagulase-negative staphylococci) are common causes of blood
culture contaminants, but they do not typically cause pulmonary infection.
(Choice D) Methicillin-resistant Staphylococcus aureus can cause hospital-acquired or post–
viral pneumonia complicated by lung abscess formation. However, purulent, foul-smelling
sputum is highly indicative of anaerobic infection rather than aerobic infection. In addition, S
aureus is not a common cause of lung abscess in patients at high risk for aspiration (eg,
alcohol/drug use).

Telegram Group Invite Link: https://t.me/joinchat/zyFOIg0j5goxY2E1


https://t.me/joinchat/zyFOIg0j5goxY2E1

Telegram Group Invite Link: https://t.me/USMLEWorldStep1

Flow volume
curve

Flow-volume loops plot air velocity in relation to lung volume during a cycle of maximal
inspiration and expiration. This patient's flow-volume loop exhibits a leftward shift
indicating increased total lung capacity and residual volume. In addition, there is a
characteristic "scooped-out" expiratory pattern consistent with the reduced expiratory flow
rates seen in obstructive lung disease. This hyperinflation and expiratory airflow obstruction
occur in chronic obstructive pulmonary disease (COPD). Airflow obstruction in COPD is
partially due to anatomic narrowing of the bronchi (chronic bronchitis). In addition, decreased

Telegram Group Invite Link: https://t.me/joinchat/zyFOIg0j5goxY2E1


Telegram Group Invite Link: https://t.me/USMLEWorldStep1

lung elasticity resulting from destruction of interalveolar walls (emphysema) promotes


dynamic compression of the airways during expiration (intrapleural pressure becomes greater
than airway pressure), further contributing to expiratory airflow obstruction.
Hyperinflation in COPD also reduces the inspiratory reserve volume, limiting the maximal tidal
volume. This is especially true during exercise, when expiration time is limited. Because
patients with COPD require extra time for exhalation, increasing amounts of air can become
trapped in the lungs during rapid breathing, leading to further reduction in tidal volume. This is
known as dynamic hyperinflation and is believed to be a major cause of dyspnea and
exercise limitation in COPD.

(Choice A) Alveolar hyaline membranes are seen in acute respiratory distress syndrome. The
pulmonary edema tends to reduce lung compliance, causing a restrictive flow-volume pattern
(purple curve).
(Choice B) Atelectasis due to extrinsic pulmonary compression may occur when the pleural
cavity fills with air (ie, pneumothorax) or fluid (eg, exudate, blood). Lung volumes and total lung
compliance are likely reduced under such conditions, producing a restrictive flow-volume
pattern.
(Choice D) Diffuse intraalveolar hemorrhage tends to dilute alveolar surfactant, increasing
alveolar surface tension and promoting alveolar atelectasis. These changes would reduce lung
volumes and lung compliance, resulting in a restrictive flow-volume pattern.
(Choice E) Pulmonary fibrosis tends to decrease lung volume and compliance, producing a
restrictive pattern. In restrictive diseases, total lung capacity and residual volume are
reduced. The expiratory flow rate is increased relative to the low lung volumes but is reduced
compared to normal peak expiratory flow rates.
(Choice F) In pulmonary thromboembolism there is decreased perfusion but minimal effect on
ventilation. Therefore, the flow-volume curve will remain mostly normal.

Telegram Group Invite Link: https://t.me/joinchat/zyFOIg0j5goxY2E1


https://t.me/joinchat/zyFOIg0j5goxY2E1

Telegram Group Invite Link: https://t.me/USMLEWorldStep1

sacoidosis

This young patient has arthralgias, dyspnea, cough, and erythema nodosum (tender,
subcutaneous, lower extremity nodules). This, in conjunction with lung nodules and hilar
fullness on x-ray, is highly suggestive of sarcoidosis, an inflammatory disease characterized by
granuloma formation in multiple tissues. Diagnosis is confirmed by biopsy
showing noncaseating granulomas composed of epithelioid cells (activated macrophages)
and giant multinucleated cells without central necrosis. Typical manifestations include hilar
adenopathy, pulmonary infiltrates (eg, nodules, interstitial lung disease), skin (eg, erythema
nodosum) and ocular (eg, anterior uveitis) involvement, polyarthritis, and constitutional
symptoms (eg, fatigue, weight loss).

Telegram Group Invite Link: https://t.me/joinchat/zyFOIg0j5goxY2E1


Telegram Group Invite Link: https://t.me/USMLEWorldStep1

Many patients with sarcoidosis do not require treatment; however, those with significant
symptoms (eg, dyspnea, chest pain) or progressive pulmonary disease (ie, worsening
opacities/fibrosis or pulmonary function tests) should be treated. Oral glucocorticoids (eg,
prednisone) are the initial treatment of choice.

Telegram Group Invite Link: https://t.me/joinchat/zyFOIg0j5goxY2E1


https://t.me/joinchat/zyFOIg0j5goxY2E1

Telegram Group Invite Link: https://t.me/USMLEWorldStep1

Telegram Group Invite Link: https://t.me/joinchat/zyFOIg0j5goxY2E1


Telegram Group Invite Link: https://t.me/USMLEWorldStep1

(Choice A) Ceftriaxone is appropriate treatment for disseminated gonococcal infection, which


causes arthralgias and tenosynovitis. However, patients typically have a pustular, painless
dermatitis; lung involvement would be unexpected.
(Choice B) Doxycycline is used to treat Lyme disease, which presents with an expansive
erythematous rash with central clearing (erythema migrans), fatigue, and arthralgias; pulmonary
involvement is unexpected. Histology demonstrates an inflammatory infiltrate with spirochetes.
(Choice C) Etanercept is a tumor necrosis factor-alpha inhibitor that is used in rheumatoid
arthritis, a condition that can cause arthralgias, skin nodules (rheumatoid nodules), and
interstitial lung disease; however, it typically involves the metacarpophalangeal and proximal
interphalangeal joints. Pathology shows a periarticular inflammatory infiltrate with joint
destruction; noncaseating granulomas would not be seen.
(Choice D) Penicillin is used to treat syphilis, but the rash associated with this disease is
typically maculopapular and involves the palms and soles. Histopathology may demonstrate
lymphoplasmacytic inflammation and granulomas, and Treponema pallidum is visible on
darkfield microscopy.
Delayed passage of meconium

This patient's abdominal distension, bilious emesis, and x-ray findings (air-fluid levels, small
bowel dilation) are consistent with bowel obstruction. The intraoperative finding of a
green inspissated mass (dehydrated meconium) in the distal ileum points to the diagnosis
of meconium ileus as the source of obstruction. Meconium ileus is a very specific finding
for cystic fibrosis (CF). Although only 10%-20% of infants with CF will have meconium ileus at
birth, almost all full-term infants with meconium ileus are eventually diagnosed with
CF. Abnormalities in chloride, sodium, and water transport by the ductal epithelium of intestinal
mucous glands cause isotonic dehydration of the lumen contents. This results in secretion
of abnormally viscous mucus into the small bowel. In the United States, pneumonia,
bronchiectasis, and cor pulmonale account for most deaths due to CF.
(Choice A) Patients with Hirschsprung disease (HD) typically present with signs of intestinal
obstruction in the newborn period that are similar to meconium ileus. However, HD typically

Telegram Group Invite Link: https://t.me/joinchat/zyFOIg0j5goxY2E1


https://t.me/joinchat/zyFOIg0j5goxY2E1

Telegram Group Invite Link: https://t.me/USMLEWorldStep1

presents with increased rectal tone, "squirt sign" (forceful expulsion of stool after rectal
examination), and obstruction at the rectosigmoid region with a transition zone between the
aganglionic rectum and proximal dilated colon. Enterocolitis (diarrhea, abdominal
pain/distension, and fever) is the most common cause of mortality in patients with HD.
(Choice B) Focal biliary or multilobular cirrhosis due to plugging of bile canaliculi by dehydrated
mucinous material develops in approximately 5% of CF patients. Death from cirrhotic liver
failure is much less likely than terminal pneumonia.
(Choice C) Most patients with CF have exocrine pancreatic insufficiency, which causes
malabsorption and steatorrhea. Infants often have large foul-smelling stools, abdominal
distention, and poor weight gain. Consequences include deficiencies of fat-soluble vitamins,
hypoproteinemia with generalized edema, and persistent diarrhea with rectal
prolapse. However, pancreatic involvement is rarely fatal with pancreatic enzyme
supplementation.
(Choice E) Although the CFTR protein is expressed in nephrons, patients with CF usually do
not manifest serious renal dysfunction or failure. Patients with CF, however, are at elevated risk
for nephrolithiasis and renal damage from nephrotoxic medications (eg, aminoglycosides).
Oxidative metabolism
Long-distance running and other forms of physical exercise cause increased oxidative
metabolism of glucose and fatty acids in the skeletal muscle. This markedly increases the
rates of O2 consumption and CO2 production, which must be balanced by increases in skeletal
muscle perfusion and alveolar ventilation.
The cardiovascular response to exercise involves vasoconstriction in the splanchnic circulation
and vasodilation in skeletal muscle, shunting blood toward exercising muscle. The
vasodilation is predominant, resulting in overall decreased systemic vascular
resistance. There is also an increase in heart rate and stroke volume that markedly increases
cardiac output and O2 delivery to the tissues. Increased O2 extraction by the tissues leads to a
concomitant increase in CO2 production, which is absorbed by the systemic capillaries and
transported in high quantities via venous blood to the lungs, where it is expired.
(Choices A and B) Mean values for arterial O2 and CO2 content remain essentially constant
during exercise, even during periods of intense exertion. This is likely accomplished via tight
regulation of increases in alveolar ventilation and gas exchange efficiency.
(Choices C and D) Tissue pH typically decreases during exercise due to production of
carbonic and lactic acid in active skeletal muscle. The acidic pH causes a decrease in
hemoglobin affinity for O2 (ie, right shift in the O2-hemoglobin dissociation curve) to facilitate
unloading of O2 in the tissues. Although venous pH may decrease significantly during moderate
exercise, there is typically little change in arterial pH due to respiratory compensation.
(Choice E) Physiologic dead space (ie, the air in the respiratory system that does not
participate in gas exchange) is decreased during exercise due to a decrease in pulmonary
vascular resistance that allows perfusion of additional pulmonary capillary beds.
(Choice G) The mean O2 content of venous blood remains the same or decreases with
exercise because the rate of O2 extraction by the tissues outpaces the rate of oxygen delivery
(cardiac output). In most healthy individuals, cardiac output is the major limiting factor to
O2 consumption during exercise.

Telegram Group Invite Link: https://t.me/joinchat/zyFOIg0j5goxY2E1


Telegram Group Invite Link: https://t.me/USMLEWorldStep1

atelactasis

This preterm infant with increased work of breathing and hypoxia has diffuse ground-glass
opacities and air bronchograms on imaging. These findings are consistent with neonatal
respiratory distress syndrome (RDS). RDS is caused by immaturity of type 2 pneumocytes,
which normally produce alveolar surfactant. Lack of surfactant causes decreased compliance
and increased surface tension of alveoli, leading to alveolar collapse at the end of
expiration. This diffuse atelectasis results in the characteristic reticular or ground-glass
opacities on chest x-ray. Unlike alveoli, larger airways remain patent and filled with air due to
their cartilaginous walls, making them visible (air bronchograms) against the reticular
background.
Management of RDS is respiratory support (to maintain alveolar pressure and prevent collapse)
and surfactant (to reduce surface tension). During the first week of life, type 2 pneumocytes

Telegram Group Invite Link: https://t.me/joinchat/zyFOIg0j5goxY2E1


https://t.me/joinchat/zyFOIg0j5goxY2E1

Telegram Group Invite Link: https://t.me/USMLEWorldStep1

begin to release endogenous surfactant, and respiratory distress typically begins to improve.

(Choice A) Pneumothorax is characterized by air leakage into the pleural space with
subsequent respiratory distress. However, x-ray would show a visceral pleural line with absent

Telegram Group Invite Link: https://t.me/joinchat/zyFOIg0j5goxY2E1


https://t.me/joinchat/zyFOIg0j5goxY2E1

USMLE e e r m h nne s
Me est i e s: https://t.me/USMLEMe est

K p n ssr m i e s: https://t.me/USMLEK p n

USMLE i e s Step : https://t.me/USMLE Step

e ker i e s: https://t.me/USMLE e ker

r s e n i e s Step : https://t.me/USMLE r sAn e n

r s e n i e s Step K: https://t.me/USMLE r sAn e n Step K

Ph se i e s: https://t.me/USMLEPh se

Pi ri e i e s: https://t.me/USMLEPi ri e

P th m i e s: https://t.me/USMLEP th m

nA i s i h it : https://t.me/USMLE nA i s

Am ss r p: https://t.me/USMLEAm ss

Le t ri nk: https://t.me/USMLELe t ri

En p int P s: https://t.me/USMLEEn p int

USMLE MEs: https://t.me/USMLE ME

L E Me E : https://t.me/USMLE n ineMe E

USMLE r Step : https://t.me/USMLE r Step

USMLE r Step K: https://t.me/USMLE r Step K

USMLE r Step nk: https://t.me/USMLE r Step

USMLE Anki: https://t.me/USMLEAnki

KissPrep: https://t.me/USMLEKissPrep
Telegram Group Invite Link: https://t.me/USMLEWorldStep1

distal lung markings.

(Choice B) Bronchopulmonary dysplasia (BPD), which causes tachypnea, retractions, and


hypoxia, is classically caused by prolonged (>1 month) mechanical ventilation, leading to lung
injury and diffuse fibrosis. Another form of BPD is caused by an arrest of pulmonary
development, leading to a decreased number and function of alveoli and pulmonary
capillaries. Although this infant is at risk for developing both forms of BPD due to prematurity,
this condition is diagnosed at age >1 month, not immediately after birth.
(Choice D) Occlusion of the bronchioles by meconium causes meconium aspiration
syndrome. In contrast to this patient, affected infants have patchy bilateral infiltrates and lung
hyperexpansion due to air trapping. In addition, this patient's amniotic fluid is clear, making this
diagnosis unlikely.
(Choice E) Retained extracellular fluid is seen in transient tachypnea of the newborn. X-ray
findings include streaky interstitial fluid collections and fluid in the minor fissures, neither of

Telegram Group Invite Link: https://t.me/joinchat/zyFOIg0j5goxY2E1


https://t.me/joinchat/zyFOIg0j5goxY2E1

Telegram Group Invite Link: https://t.me/USMLEWorldStep1

which are seen in this patient.

baroreceptor

Telegram Group Invite Link: https://t.me/joinchat/zyFOIg0j5goxY2E1


Telegram Group Invite Link: https://t.me/USMLEWorldStep1

Telegram Group Invite Link: https://t.me/joinchat/zyFOIg0j5goxY2E1


https://t.me/joinchat/zyFOIg0j5goxY2E1

Telegram Group Invite Link: https://t.me/USMLEWorldStep1

This patient with chronic obstructive pulmonary disease (COPD) developed a decrease in
respiratory rate as a result of oxygen supplementation. Oxygen has a minimal effect on
respiratory drive unless the arterial partial pressure of oxygen (PaO2) drops below 60-70 mm
Hg; therefore, the arterial partial pressure of carbon dioxide (PaCO2) is the major stimulator of
respiration in healthy individuals. However, patients with long-standing COPD have decreased
sensitivity to PaCO2 (due to chronic CO2 retention) and may also have profound hypoxemia
(PaO2 <60 mm Hg); therefore, PaO2 levels can become a significant contributor to respiratory
drive.
The depth and rate of respirations are controlled by the medullary respiratory center based on
input from central and peripheral chemoreceptors and airway mechanoreceptors. Peripheral
chemoreceptors found in the carotid and aortic bodies are the primary sites for sensing
PaO2 and are stimulated by hypoxemia. When supplemental oxygen is administered, the
rapid increase in PaO2 can reduce peripheral chemoreceptor stimulation and decrease the
respiratory rate.
Reduced respiratory rate is a minor contributor to the oxygen-induced hypercapnia that can
occur in patients with COPD; increased ventilation-perfusion mismatch triggered by alleviation
of pulmonary vasoconstriction in poorly ventilated areas is the major mechanism.

(Choice B) Central chemoreceptors, located in the medulla, are more involved in the
respiratory response to hypercapnia than to hypoxemia. CO2 readily diffuses through the blood-
brain barrier and forms hydrogen ions in the cerebrospinal fluid; the resulting decrease in pH is
detected by medullary neurons, triggering an increase in respiration. Because the blood-brain

Telegram Group Invite Link: https://t.me/joinchat/zyFOIg0j5goxY2E1


Telegram Group Invite Link: https://t.me/USMLEWorldStep1

barrier is relatively impermeable to hydrogen ions, blood pH itself has little effect on central
chemoreceptors.
(Choice C) The juxtaglomerular apparatus is located in the kidneys and consists of
juxtaglomerular cells and the macula densa. It helps regulate renal blood flow and the
glomerular filtration rate via renin secretion.
(Choices D and E) Pulmonary stretch receptors include myelinated and unmyelinated C fibers
in the lungs and airways. These receptors regulate the duration of inspiration depending on the
degree of lung distension (Hering-Breuer reflex).

Telegram Group Invite Link: https://t.me/joinchat/zyFOIg0j5goxY2E1


https://t.me/joinchat/zyFOIg0j5goxY2E1

Telegram Group Invite Link: https://t.me/USMLEWorldStep1

Coal worker
pneumoconiosis

Telegram Group Invite Link: https://t.me/joinchat/zyFOIg0j5goxY2E1


Telegram Group Invite Link: https://t.me/USMLEWorldStep1

This patient has coal worker pneumoconiosis, a form of interstitial lung fibrosis secondary to
long-term inhalation of coal particles.
Inorganic dust is constantly being inhaled and must be cleared by the respiratory tract to
prevent disease. The clearance mechanisms used by the lungs vary depending on the size of
the particles. Larger particles become trapped by mucus secretions in the trachea, bronchi, and
proximal bronchioles; these trapped particles are swept upward toward the pharynx by the
collective beating of ciliated cells. The finest particles (<2 µm) can travel past the highly
ciliated airways into the respiratory bronchioles and alveoli, where they are phagocytized
by alveolar macrophages.
Engulfment of inorganic dust causes macrophage activation and the release of a number of
cytokines that induce pulmonary inflammation. Growth factors, including platelet-derived growth
factor and insulin-like growth factor, are also released and stimulate fibroblasts to proliferate and
produce collagen. This production results in progressive interstitial lung fibrosis that
characterizes the pneumoconioses.
(Choices A and E) The largest dust particles become trapped in the upper airways (eg, nasal
passages, pharynx) and are cleared through coughing and sneezing.
(Choice B) Immunologic memory can help with the clearance of previously encountered
microorganisms but would not contribute to the removal of inorganic dust particles.
(Choice C) Mucociliary transport is the primary means of eliminating particles that lodge in the
bronchi and bronchioles (typically, medium-sized particles).

Telegram Group Invite Link: https://t.me/joinchat/zyFOIg0j5goxY2E1


https://t.me/joinchat/zyFOIg0j5goxY2E1

Telegram Group Invite Link: https://t.me/USMLEWorldStep1

aging

The human respiratory system undergoes significant changes during normal aging. The
mechanics of breathing are compromised due to alteration of lung, chest wall, and total
respiratory system compliance:

• Lung compliance increases ("loose, baggy lungs") due to loss of elastin content. This
process resembles mild emphysema and leads to dynamic expiratory airflow obstruction,
premature airway closure, and progressive hyperinflation.

• Chest wall compliance decreases ("stiff rib cage") due to degenerative changes
(ossification, arthritis) of the sternocostal and costovertebral joints and kyphosis of the
thoracic spine.

• Total respiratory system compliance, the combination of lung parenchymal and chest
wall compliance, is decreased because the stiffness of the chest wall dominates over
the increased laxity of the lung.

As these mechanical changes occur, there is also a marked increase in dead space
ventilation. Although anatomic dead space (nasopharynx to terminal bronchioles) remains
constant after early adulthood when the bronchial tree reaches full maturity, alveolar dead
space increases with age due to progressive elastin degeneration, alveolar simplification, and
capillary dropout.

Telegram Group Invite Link: https://t.me/joinchat/zyFOIg0j5goxY2E1


Telegram Group Invite Link: https://t.me/USMLEWorldStep1

The net result is a mechanically disadvantaged respiratory system (lower compliance) that must
contend with increased dead space. A greater work of breathing is required to maintain the
same alveolar minute ventilation and the efficiency of breathing (ventilation per unit of work)
steadily declines.

Telegram Group Invite Link: https://t.me/joinchat/zyFOIg0j5goxY2E1


https://t.me/joinchat/zyFOIg0j5goxY2E1

Telegram Group Invite Link: https://t.me/USMLEWorldStep1

Telegram Group Invite Link: https://t.me/joinchat/zyFOIg0j5goxY2E1


Telegram Group Invite Link: https://t.me/USMLEWorldStep1

Anatomic
shunting

The graph illustrates the pO2 of blood as it moves through the lungs and into the systemic
circulation. Blood in the pulmonary arteries comes from the systemic venous system and is
normally deoxygenated. As blood moves through the pulmonary capillaries, it becomes
progressively more oxygenated until it equilibrates with the alveolar pO2 (~104 mm Hg when
breathing room air). This is represented by the curve's first upward deflection and subsequent
plateau. However, blood in the left atrium and ventricle has a slightly lower pO2 (~100 mm Hg)
than blood in the pulmonary capillaries. This is due to the mixing of deoxygenated blood with
oxygenated blood from the pulmonary veins and is represented by the downward deflection
identified by the arrow. This deoxygenated blood comes from the following sources:
1. The bronchial arteries carry blood to the bronchi and bronchioles and, together with the
pulmonary artery, form the dual blood supply to the lungs. The bronchial veins return
only a portion of this blood to the right heart via the azygos and hemi-azygos veins; most
of the blood supplied by the bronchial arteries returns to the left heart via the pulmonary
veins.
2. Small cardiac (thebesian) veins that drain into the left atrium and ventricle also
contribute to normal anatomic shunting.

Telegram Group Invite Link: https://t.me/joinchat/zyFOIg0j5goxY2E1


https://t.me/joinchat/zyFOIg0j5goxY2E1

Telegram Group Invite Link: https://t.me/USMLEWorldStep1

(Choice A) Hypoxia due to a diffusion limitation frequently results from pulmonary fibrosis,
acute respiratory distress syndrome, emphysema, or hyaline membrane disease of the
infant. In diffusion-limited gas exchange, the blood pO2 does not equilibrate with the alveolar
pO2 by the end of its traversal through the alveolar capillaries.
(Choice B) Dead space ventilation (wasted ventilation) refers to the volume of inspired air that
does not participate in gas exchange. Anatomic dead space refers to the volume of air that
remains in the conducting parts of the airways, such as the bronchi and bronchioles. Alveolar
dead space refers to the volume of air in the alveoli that cannot participate in gas exchange due
to poor perfusion.
(Choice D) Perfusion-limited gas exchange occurs in the normal physiologic resting state, as
equilibration of the blood with the alveolar gas is complete by the time the blood traverses one-
third of the total pulmonary capillary length.
(Choice E) The hemoglobin dissociation curve shifts to the right in the systemic capillaries to
increase oxygen delivery to the body tissues. Increases in pCO2, temperature, and 2,3-
diphosphoglycerate concentration cause right-shifting of the curve; the opposite conditions

Telegram Group Invite Link: https://t.me/joinchat/zyFOIg0j5goxY2E1


Telegram Group Invite Link: https://t.me/USMLEWorldStep1

cause a left shift and favor oxygen loading in the lungs.

Telegram Group Invite Link: https://t.me/joinchat/zyFOIg0j5goxY2E1


https://t.me/joinchat/zyFOIg0j5goxY2E1

Telegram Group Invite Link: https://t.me/USMLEWorldStep1

COPD

This patient with dyspnea, productive cough, and an obstructive pattern on pulmonary function
testing—in the setting of a prolonged smoking history—likely has chronic obstructive
pulmonary disease (COPD). COPD is characterized by chronic airway inflammation, which
results in both parenchymal destruction (emphysema) and remodeling of the airways (chronic
bronchitis). The primary cell lines that are increased in COPD are neutrophils, macrophages,
and CD8+ T lymphocytes. These cells release enzymes and proteases, such as
neutrophil elastase, that cause alveolar damage, reduced ciliary motion, and increased mucus
secretion by goblet cells. In addition, the inflammatory cells show impaired ability to
phagocytize bacterial pathogens, possibly contributing to increased risk of respiratory infections
such as community-acquired pneumonia.

Telegram Group Invite Link: https://t.me/joinchat/zyFOIg0j5goxY2E1


Telegram Group Invite Link: https://t.me/USMLEWorldStep1

(Choice A) The involvement of B lymphocytes in COPD pathogenesis is controversial;


however, they do not appear to have a major role, and their increased numbers may actually be
protective against bacterial colonization.
(Choice B) Although CD4+ T lymphocytes are slightly increased in COPD, their numbers and
pathologic role are much less significant than those of CD8+ T lymphocytes. However, CD4+ T
lymphocytes have been implicated in asthma pathogenesis.
(Choice C) Eosinophils play an important role in asthma but do not appear to have a major
impact in COPD. Their increased presence in patients with COPD may indicate coexisting
asthma.
(Choice D) Patients with mast cell disorders can have flushing, abdominal discomfort, and
respiratory symptoms such as dyspnea and wheezing due to bronchoconstriction. However,
mast cells do not play an important role in the pathogenesis of COPD.

TB

Telegram Group Invite Link: https://t.me/joinchat/zyFOIg0j5goxY2E1


https://t.me/joinchat/zyFOIg0j5goxY2E1

Telegram Group Invite Link: https://t.me/USMLEWorldStep1

Telegram Group Invite Link: https://t.me/joinchat/zyFOIg0j5goxY2E1


Telegram Group Invite Link: https://t.me/USMLEWorldStep1

Primary infection with Mycobacterium tuberculosis begins with the inhalation of


contaminated respiratory droplets into the lower lobe of the lung. Initially, the organism
replicates in an unchecked fashion within the alveoli due to virulence factors (eg, cord factor)
that prevent bacterial destruction by alveolar macrophages. The pathogen is also carried by
infected macrophages through the draining lymphatic system to ipsilateral hilar lymph
nodes. These initial areas of infection (lower lobe, ipsilateral hilar lymph node) are collectively
referred to as a Ghon complex.
After a few weeks, antigen presenting cells display mycobacterial antigens leading to the
stimulation of CD4 cells and the subsequent cytokine-mediated activation of
macrophages. Activated macrophages control the infection (in >95% of patients) by forming
mature phagolysosomes to kill intracellular mycobacteria and by differentiating into cells (eg,
epithelioid cells, Langhans giant cells) that wall off extracellular mycobacteria within
granulomas. Over time, granulomas become fibrosed and calcified, which reinforces their
structure. It also allows areas of primary infection to be visualized on x-ray or gross
pathology. A fibrosed and calcified Ghon complex is referred to as a Ranke complex.
(Choices A and C) Most patients with primary tuberculosis contain the infection, but those with
ineffective immunity (eg, extremes of age, immunosuppression) can develop progressive illness
with hematogenous spread to distant areas (eg, Potts disease, meningitis). Widespread,
massive hematogenous dissemination can also occur, resulting in innumerable, small, millet

Telegram Group Invite Link: https://t.me/joinchat/zyFOIg0j5goxY2E1


https://t.me/joinchat/zyFOIg0j5goxY2E1

Telegram Group Invite Link: https://t.me/USMLEWorldStep1

seed-like lesions in multiple organs (miliary tuberculosis).

(Choices B and E) Primary tuberculosis infections are usually contained within granulomas but
not fully eliminated. If the walls of the granuloma weaken, M tuberculosis can escape and
cause reactivation (secondary) disease. This is most common in the setting of acquired
immunosuppression (eg, HIV, tumor necrosis factor-alpha inhibitors) and usually presents with

Telegram Group Invite Link: https://t.me/joinchat/zyFOIg0j5goxY2E1


Telegram Group Invite Link: https://t.me/USMLEWorldStep1

symptoms (eg, weight loss, cough, fever) and apical cavitary lesions.

Seizure
Peptostreptococcus and Fusobacterium are anaerobic bacteria that are part of normal mouth
flora. The presence of these organisms in this patient's lung lesion is highly suggestive of a
developing lung abscess. As an abscess evolves, it typically forms a cavitary lesion with an
identifiable air-fluid level on imaging. Symptoms are often indolent and include fever, night
sweats, weight loss, and a cough producing foul-smelling sputum (indicating anaerobes).
Lung abscesses are usually caused by one of the following:
• Aspiration of oral bacteria into the lower airways (most common) – These abscesses
are usually composed of a combination of anaerobic oral
flora (eg, Peptostreptococcus, Prevotella, Bacteroides, Fusobacterium) and aerobic
organisms (eg, Streptococcus). Risk is greatest in those who have conditions
associated with loss of consciousness or impaired swallowing, such as alcoholism,
drug abuse, neurologic disease (eg, seizures, stroke), or anatomic abnormalities (eg,
esophageal strictures or diverticula).
• Bacterial pneumonia – Lung abscess can occur in the setting of certain bacterial
pneumonias such as those due to Staphylococcus aureus, Escherichia coli, Klebsiella

Telegram Group Invite Link: https://t.me/joinchat/zyFOIg0j5goxY2E1


https://t.me/joinchat/zyFOIg0j5goxY2E1

Telegram Group Invite Link: https://t.me/USMLEWorldStep1

pneumoniae, or Pseudomonas aeruginosa. Most cases arise in the hospital setting and
occur in patients with immunosuppression, older age, or underlying lung disease.
• Bacteremia and/or infectious endocarditis – Hematogenous spread of an infection to the
lung usually causes multiple, monomicrobial lung abscesses. The most common
causative agents are Staphylococcus and Streptococcus species.

Telegram Group Invite Link: https://t.me/joinchat/zyFOIg0j5goxY2E1


Telegram Group Invite Link: https://t.me/USMLEWorldStep1

(Choice A) Mitral valve prolapse is a risk factor for development of subacute bacterial
endocarditis (with Streptococcus species). Left-sided endocarditis could cause septic
embolization to downstream organs (eg, spleen, brain) but would not send septic emboli to the
lungs.
(Choice B) Bronchogenic carcinoma can cause bronchial obstruction and postobstructive
pneumonia, which may lead to secondary abscess formation. However, these lung abscesses
would be less likely to contain oral anaerobic flora.
(Choice C) Lung abscess may be associated with a prior penetrating
trauma. Staphylococcus and Streptococcus species (introduced from the skin) are usually
responsible.
(Choice E) Tobacco smoking compromises pulmonary defenses and predisposes patients to
developing community-acquired pneumonia due to common pathogens such as Streptococcus
pneumoniae and Mycoplasma pneumoniae. These organisms are less likely to cause lung
abscess.
(Choice F) Urinary infections can be associated with bacteremia, particularly in those with
diabetes mellitus, advanced age, or structural urinary abnormalities. However, most urinary
infections are caused by coliform bacteria (eg, E coli) from the lower gastrointestinal tract. Lung
abscess is not a common complication of urinary infection,
and Peptostreptococcus and Fusobacterium would be unusual urinary organisms.

Telegram Group Invite Link: https://t.me/joinchat/zyFOIg0j5goxY2E1


https://t.me/joinchat/zyFOIg0j5goxY2E1

Telegram Group Invite Link: https://t.me/USMLEWorldStep1

ARDS

This patient with pancreatitis (abdominal pain, elevated lipase, recent heavy alcohol intake)
has developed hypoxic respiratory failure with bilateral pulmonary infiltrates. In a young patient
without a history of cardiac disease, this is highly suggestive of acute respiratory distress
syndrome (ARDS). Direct (eg, irritant chemicals, trauma, gastric aspiration) or indirect (eg,
pancreatitis, sepsis) pulmonary insults cause excessive cytokine release, leading to endothelial
activation, neutrophilic migration to the lungs, and degranulation with release of toxic
mediators. This process worsens the pulmonary alveolar and endothelial injury, leading
to increased pulmonary capillary permeability, leakage of fluid into the alveoli, and

Telegram Group Invite Link: https://t.me/joinchat/zyFOIg0j5goxY2E1


Telegram Group Invite Link: https://t.me/USMLEWorldStep1

pulmonary edema (Choice A).

ARDS-associated interstitial edema and hyaline membrane formation along with fluid-filled
alveoli collapse and atelectasis result in decreased lung compliance and increased work of
breathing (Choices B and E). The fluid-filled, collapsed alveoli are unable to deliver oxygen
despite continuing to receive adequate blood flow, resulting in ventilation/perfusion
mismatch (ie, physiologic shunting) and hypoxia (Choice D).
Noncardiogenic pulmonary edema (eg, ARDS, high-altitude pulmonary edema) can be
distinguished from cardiogenic pulmonary edema (eg, decompensated left ventricular failure,
volume overload) by a normal pulmonary capillary wedge pressure (6-12 mm Hg). Other
distinguishing features include the clinical history (pneumonia/sepsis or pancreatitis suggests
ARDS) and physical exam (peripheral edema and jugular venous distension suggest a
cardiogenic source).

Telegram Group Invite Link: https://t.me/joinchat/zyFOIg0j5goxY2E1


https://t.me/joinchat/zyFOIg0j5goxY2E1

Telegram Group Invite Link: https://t.me/USMLEWorldStep1

asthma

This patient's intermittent nocturnal dyspnea and episodic cough that began after a respiratory
infection suggest a diagnosis of asthma, a disease characterized by airway inflammation and
bronchial hyperreactivity to various stimuli, including physical, chemical, and allergenic
irritants. Patients often have a family history of other diseases in the "allergic triad" (ie, allergic
rhinitis, atopic dermatitis, asthma). Asthma is an obstructive airway disease (FEV1/FVC ≤70%);
however, airflow obstruction is variable, and lung function studies may be normal between
exacerbations.

Bronchoprovocation testing can be useful in diagnosis when a patient presents with a history
concerning for asthma but has normal spirometry values. Methacholine is an inhaled
muscarinic cholinergic agonist that induces bronchoconstriction; patients are given increasing
doses followed by serial spirometry. Patients with asthma demonstrate hyperresponsivity to

Telegram Group Invite Link: https://t.me/joinchat/zyFOIg0j5goxY2E1


https://t.me/joinchat/zyFOIg0j5goxY2E1

USMLE e e r m h nne s
Me est i e s: https://t.me/USMLEMe est

K p n ssr m i e s: https://t.me/USMLEK p n

USMLE i e s Step : https://t.me/USMLE Step

e ker i e s: https://t.me/USMLE e ker

r s e n i e s Step : https://t.me/USMLE r sAn e n

r s e n i e s Step K: https://t.me/USMLE r sAn e n Step K

Ph se i e s: https://t.me/USMLEPh se

Pi ri e i e s: https://t.me/USMLEPi ri e

P th m i e s: https://t.me/USMLEP th m

nA i s i h it : https://t.me/USMLE nA i s

Am ss r p: https://t.me/USMLEAm ss

Le t ri nk: https://t.me/USMLELe t ri

En p int P s: https://t.me/USMLEEn p int

USMLE MEs: https://t.me/USMLE ME

L E Me E : https://t.me/USMLE n ineMe E

USMLE r Step : https://t.me/USMLE r Step

USMLE r Step K: https://t.me/USMLE r Step K

USMLE r Step nk: https://t.me/USMLE r Step

USMLE Anki: https://t.me/USMLEAnki

KissPrep: https://t.me/USMLEKissPrep
Telegram Group Invite Link: https://t.me/USMLEWorldStep1

bronchoprovocation with a reduction in FEV1 at lower doses than in those without asthma.

Conversely, bronchodilator administration can help diagnose patients who have obstructive
values on spirometry at baseline. Spirometry is performed before and after administration of a
bronchodilator (eg, levalbuterol); reversal of the airflow obstruction after therapy suggests the
diagnosis of asthma, while irreversible obstruction suggests an alternative diagnosis (eg,
bronchiolitis, COPD). However, in a patient who is currently asymptomatic with normal lung
function studies, this is unlikely to aid in diagnosis (Choice D).
(Choice A) Epinephrine, an adrenergic agonist, is used for treatment of anaphylaxis. Patients
can develop respiratory compromise due to swelling of the airways; however, this is typically
accompanied by hypotension and skin involvement (eg, hives, lip swelling). Epinephrine is not
indicated in the workup of asthma.
(Choices B and F) Gadolinium is a contrast material used for MRI. Regadenoson is a
coronary vasodilator used in cardiac stress testing to evaluate for coronary artery
disease. Neither is indicated in the workup of asthma.
(Choices C and G) Ipratropium bromide is a muscarinic cholinergic antagonist used for asthma
and chronic obstructive pulmonary disease, and theophylline is a phosphodiesterase-3 inhibitor
sometimes used for refractory asthma. These medications can treat asthma but are not used
for diagnostic purposes.

Telegram Group Invite Link: https://t.me/joinchat/zyFOIg0j5goxY2E1


https://t.me/joinchat/zyFOIg0j5goxY2E1

Telegram Group Invite Link: https://t.me/USMLEWorldStep1

Lung cells

This patient's presentation is consistent with pulmonary alveolar proteinosis (PAP), a rare
condition characterized by progressive respiratory dysfunction due to the accumulation of
surfactant debris within alveolar spaces. Surfactant is a lipoproteinaceous material that
appears pink with periodic acid–Schiff staining; it forms concentrically laminated structures
(ie, lamellar bodies) that can be seen on electron microscopy.

Telegram Group Invite Link: https://t.me/joinchat/zyFOIg0j5goxY2E1


Telegram Group Invite Link: https://t.me/USMLEWorldStep1

PAP is caused by an imbalance between surfactant production and clearance. In healthy lungs,
surfactant is secreted by type 2 pneumocytes and eventually cleared by alveolar
macrophages. In PAP, surfactant removal is impaired due to compromised alveolar
macrophage function (eg, usually because of defects in granulocyte-monocyte colony-
stimulating factor [GM-CSF] signaling).
Treatment involves therapeutic whole-lung lavage to wash away surfactant and inhaled GM-
CSF replacement therapy.
(Choice B) Ciliated epithelial cells line the conducting airways and are responsible for
mucociliary escalator function, a key process in airway clearance. Primary ciliary dyskinesia
(eg, Kartagener syndrome) is characterized by impaired ciliary function and leads to
mucostasis, recurrent pneumonia, and bronchiectasis.
(Choice C) Club cells (bronchiolar cells) are located at the bronchoalveolar alveolar junction
and protect and repair the distal airway (terminal bronchioles). Club cell dysfunction has been
implicated in the pathogenesis of COPD.
(Choice D) Fibroblasts produce and maintain the connective tissue matrix of the lung, lending
structural support and elasticity. Loss of fibroblast function contributes to degenerative airspace
dilation in emphysema.
(Choice E) Type 1 pneumocytes form the alveolar gas-exchanging surface. Rapid destruction
and loss of type 1 pneumocytes occurs in early acute lung injury. It does not result in the
abnormal accumulation of surfactant products.
auscultation

This patient's chest x-ray shows blunting of the right costophrenic angle and opacification of
much of the right lung consistent with a large, right-sided pleural effusion. His unintentional
weight loss and smoking history suggest a malignant etiology.
Pleural effusions are collections of fluid that form between the visceral pleura that lines the lung
and the parietal pleura that lines the thoracic cavity. On physical examination, the fluid acts to
insulate vibrations and sounds originating within the airways of the lung. Therefore, both tactile
fremitus, the vibration created by vocalized sound (eg, saying "ninety-nine"), and breath

Telegram Group Invite Link: https://t.me/joinchat/zyFOIg0j5goxY2E1


https://t.me/joinchat/zyFOIg0j5goxY2E1

Telegram Group Invite Link: https://t.me/USMLEWorldStep1

sounds are decreased over a pleural effusion. In addition, the relatively high density of pleural
fluid compared with air creates dullness to percussion.
(Choice A) Bronchial breath sounds describe the loud and relatively high-pitched breath
sounds that are normally heard over the trachea and main bronchi; breath sounds at the
periphery of the lung are typically vesicular (soft and relatively low-pitched) in nature. Bronchial
breath sounds in the periphery of the lung suggest alveolar consolidation (eg, pneumonia) or
fibrosis, but they should not be present with pleural effusion.
(Choice C) Inspiratory crackles are formed by alveoli and small airways popping open and may
be heard with pneumonia and atelectasis. The descriptor "fine" is often used to describe soft,
high-pitch inspiratory crackles associated with pulmonary fibrosis.
(Choice D) A focal, monophasic wheeze may suggest partial obstruction of a large airway (eg,
due to a foreign body or tumor).
(Choice E) Percussion notes exist on a spectrum. Normal lung, which represents a composite
of parenchymal tissue and air, generates sound that is considered resonant to percussion. The
relatively lower density of air alone (ie, pneumothorax) causes hyperresonance to percussion.
(Choice F) Rhonchi are coarse rattling or wheezing sounds that are caused by mucus
secretions in the airways and may be heard in patients with chronic obstructive pulmonary
disease or bronchiectasis. Rhonchi will often clear with coughing. They are not an expected
finding with pleural effusion.
pneumothorax

This patient with sudden-onset shortness of breath and a physical examination showing
subcutaneous crepitus and unilaterally decreased breath sounds most likely has
a pneumothorax. In cystic fibrosis, chronic lung damage, combined with mucus plugging and
large alveolar pressure surges (eg, coughing), predisposes to spontaneous alveolar rupture.

Telegram Group Invite Link: https://t.me/joinchat/zyFOIg0j5goxY2E1


Telegram Group Invite Link: https://t.me/USMLEWorldStep1

Pressure in the lungs is normally equivalent to atmospheric pressure (ie, 0 cm H2O) at end
expiration, and the pressure in the intrapleural space is negative (eg, −5 cm H2O) due to the
expanding tendency of the chest wall and collapsing tendency of the lungs. Alveolar rupture
creates continuity between the lungs and the pleural space, with pressure equalization and loss
of intrapleural negative pressure. Shortness of breath results from inability to expand the
ruptured lung, and during rupture, air may be forced into the subcutaneous tissues of the chest
wall to cause crepitus.
Simple pneumothorax is most common, but tension pneumothorax can develop if the rupture
creates a one-way tissue valve that opens during inspiration to allow air into the pleural space
and then closes during expiration to trap that air. As a result, intrapleural pressure increases
with each breath and leads to contralateral mediastinal shift with vena cava
collapse and decreased venous return to the heart. The reduced cardiac output
causes hypotension and tachycardia (ie, obstructive shock) and can rapidly lead to cardiac

Telegram Group Invite Link: https://t.me/joinchat/zyFOIg0j5goxY2E1


https://t.me/joinchat/zyFOIg0j5goxY2E1

Telegram Group Invite Link: https://t.me/USMLEWorldStep1

arrest; therefore, emergent decompression of the pleural space is needed.

Telegram Group Invite Link: https://t.me/joinchat/zyFOIg0j5goxY2E1


Telegram Group Invite Link: https://t.me/USMLEWorldStep1

Telegram Group Invite Link: https://t.me/joinchat/zyFOIg0j5goxY2E1


https://t.me/joinchat/zyFOIg0j5goxY2E1

Telegram Group Invite Link: https://t.me/USMLEWorldStep1

Telegram Group Invite Link: https://t.me/joinchat/zyFOIg0j5goxY2E1


Telegram Group Invite Link: https://t.me/USMLEWorldStep1

(Choice A) Alveolar consolidation with inflammatory exudate occurs with pneumonia, which
can cause hypotension (ie, septic shock). However, increased breath sounds are expected with
pneumonia, and it would not explain subcutaneous crepitus.
(Choice B) Bronchial obstruction with alveolar air reabsorption describes atelectasis (eg, due
to a large mucus plug). Shortness of breath and decreased breath sounds are typical, but
hypotension and subcutaneous crepitus are not expected.
(Choice C) Diffuse bronchiolar constriction occurs with asthma exacerbation and is not typical
of cystic fibrosis. Hypotension and unilaterally decreased breath sounds are not expected with
asthma exacerbation.
(Choice D) Increased dead-space ventilation occurs with pulmonary embolism. Although
hypotension (ie, obstructive shock) can occur, decreased breath sounds and subcutaneous
crepitus are not typical.

Telegram Group Invite Link: https://t.me/joinchat/zyFOIg0j5goxY2E1


https://t.me/joinchat/zyFOIg0j5goxY2E1

Telegram Group Invite Link: https://t.me/USMLEWorldStep1

Pulmonary
embolism

Telegram Group Invite Link: https://t.me/joinchat/zyFOIg0j5goxY2E1


Telegram Group Invite Link: https://t.me/USMLEWorldStep1

Telegram Group Invite Link: https://t.me/joinchat/zyFOIg0j5goxY2E1


https://t.me/joinchat/zyFOIg0j5goxY2E1

Telegram Group Invite Link: https://t.me/USMLEWorldStep1

This patient's autopsy shows a large thromboembolism lodged in the right pulmonary artery;
he most likely died from a massive pulmonary embolism (PE) that originated as a lower
extremity deep vein thrombosis. Massive PE can lead to sudden occlusion of >50% of the
pulmonary arterial circulation, causing a rapid increase in pulmonary vascular resistance and
right ventricular (RV) pressure load. This can result in acute RV dysfunction and inability to
pump blood through the pulmonary circulation to the left side of the heart. With a marked
decrease in left ventricular preload (left ventricular end-diastolic volume) there is a sudden loss
of cardiac output.
Depending on the degree of compensation for the loss of cardiac output, patients can
experience lightheadedness, syncope, or sudden cardiac death (SCD). Less commonly,
acute PE may also cause SCD due to cardiac arrhythmia induced by RV strain and ischemia.
(Choice A) Coronary atheromatous plaque rupture leads to acute myocardial infarction; SCD
can result due to ventricular arrhythmia (ie, ventricular tachycardia, ventricular fibrillation)
triggered by myocardial ischemia/infarction. There would not be evidence of thromboembolism
in the pulmonary arteries.

(Choices C and D) Malignant tumor may erode into a bronchial artery, resulting in acute
hemorrhage, or it may erode into a bronchus resulting in airway obstruction. Neither of these

Telegram Group Invite Link: https://t.me/joinchat/zyFOIg0j5goxY2E1


Telegram Group Invite Link: https://t.me/USMLEWorldStep1

scenarios is likely to cause SCD. Bronchial artery hemorrhage would likely present with
hemoptysis and, if fatal, would be evidenced by blood or thrombus in the airways on gross
pathology. Bronchial airway obstruction is likely to present with respiratory difficulty or distress.
(Choice E) Sleep-related upper airway obstruction occurs in obstructive sleep apnea, which
may cause SCD due to hypoxemia-related cardiac arrhythmia. However, there would not be
evidence of thromboembolic disease on autopsy.
aging

The human respiratory system undergoes significant changes during normal aging. Pulmonary
gas exchange becomes subtly impaired due to a sequence of gradual changes:

• Lung elastin degrades over time, leading to alveolar simplification (loss of surface area
due to airspace enlargement). Loss of elastic fiber support causes dynamic airflow
obstruction resembling emphysema, leading to hyperinflation. Therefore, inspired tidal
volume has less opportunity to participate in gas exchange. In other words,
physiologic dead space is increased.

• The muscles of respiration (chiefly the diaphragm) undergo remodeling over time, with
atrophy of fast-twitch muscle fibers. This leads to lower peak force production for
maneuvers such as deep inspiration or coughing. In combination with chest wall
stiffening, this allows a small degree of microatelectasis to develop at the lung
bases. This results in intrapulmonary shunting (perfusion of blood through atelectatic
[nonventilated] alveoli).
Telegram Group Invite Link: https://t.me/joinchat/zyFOIg0j5goxY2E1
https://t.me/joinchat/zyFOIg0j5goxY2E1

Telegram Group Invite Link: https://t.me/USMLEWorldStep1

• This combination of increased dead space and increased shunt effect leads to
greater ventilation-perfusion mismatch. This ultimately causes the alveolar-arterial
(A-a) oxygen gradient to progressively widen over decades. When evaluating
an elevated A-a gradient, patient age must be accounted for to determine whether it is
within the expected limits of normal aging (expected A-a gradient = age/3 mm Hg).

• Because of the higher A-a gradient, arterial partial pressure of oxygen


(PaO2) falls during aging (from 90 to 70 mm Hg by age 80). Since PaO2 still remains on
the plateau portion of the oxyhemoglobin desaturation curve, no change in SaO2 would
be detected by simple pulse oximetry.

In contrast to decreased PaO2, the partial pressure of carbon dioxide in arterial blood (PaCO2)
(reflecting total alveolar ventilation) is not significantly changed. Although dead space
increases and peak diaphragm strength is reduced, these changes are not enough to cause
alveolar hypoventilation by themselves; resting hypoventilation (elevated PaCO2) at any age is
always pathologic. Instead, elderly persons are less able to compensate for high minute
ventilation loads (eg, sepsis, thyrotoxicosis), making them prone to hypercapnia when acutely ill.

Telegram Group Invite Link: https://t.me/joinchat/zyFOIg0j5goxY2E1


Telegram Group Invite Link: https://t.me/USMLEWorldStep1

Telegram Group Invite Link: https://t.me/joinchat/zyFOIg0j5goxY2E1


https://t.me/joinchat/zyFOIg0j5goxY2E1

Telegram Group Invite Link: https://t.me/USMLEWorldStep1

TB

Telegram Group Invite Link: https://t.me/joinchat/zyFOIg0j5goxY2E1


Telegram Group Invite Link: https://t.me/USMLEWorldStep1

Telegram Group Invite Link: https://t.me/joinchat/zyFOIg0j5goxY2E1


https://t.me/joinchat/zyFOIg0j5goxY2E1

Telegram Group Invite Link: https://t.me/USMLEWorldStep1

This patient's symptoms (cough, hemoptysis, weight loss), acid-fast bacilli on sputum culture,
and upper lobe cavitary lesion are suggestive of secondary (reactivation) tuberculosis. His
advanced age, multiple comorbidities, and partial immune suppression secondary to chronic
oral corticosteroid use also place him at risk for reactivation disease.
Primary tuberculosis infection occurs following inhalation of aerosolized Mycobacterium
tuberculosis. The organisms are deposited in the lower lungs and phagocytosed by alveolar
macrophages, where they proliferate until the macrophages are activated by TH1
lymphocytes. The infection can be subsequently eliminated if the area of involvement is small
enough. However, larger regions of caseating necrosis become walled off, allowing M
tuberculosis to survive in a dormant state without causing disease or symptoms. Later in life
(usually following immunosuppression by drugs or HIV) the bacteria can reactivate and
establish infection in the upper lungs (particularly the apex). The predilection for upper lung
regions may be related to decreased lymphatic flow or increased oxygen tension. The
organisms multiply in the apices, causing caseous and liquefactive necrosis and
extensive cavitary disease. Erosion into the pulmonary vessels can result in severe
hemoptysis. Hematogenous dissemination may also occur, causing miliary or extrapulmonary
(eg, Potts disease, tuberculous meningitis) tuberculosis.
(Choice A) Primary tuberculosis infection often begins as a focal lesion in the mid-to-lower
lungs (Ghon focus). M tuberculosis then spreads lymphatically to the hilar lymph nodes,
forming a Ghon complex. The organisms can remain dormant in a walled-off Ghon complex for
many years before reactivating. Alternatively, the lesion may heal, forming a benign, calcified
Ranke complex that is not associated with reactivation tuberculosis.
(Choice B) A Ghon complex forms during primary tuberculosis infection and consists of a Ghon
focus and hilar lymphadenopathy. This patient's upper lung involvement and cavitary lesion are
more characteristic of secondary (reactivation) tuberculosis. Healing of this lesion would result
in a persistent cavity that may become secondarily infected with Aspergillus flavus and form
fungus balls.
(Choice D) M tuberculosis is a facultative intracellular bacterium that can survive and multiply
within macrophages; as a result, circulating antibodies cannot bind it to promote phagocytosis or
complement-mediated killing. Therefore, humoral immunity plays no role in the control of M
tuberculosis.
(Choice E) A negative skin tuberculin test after M tuberculosis exposure suggests anergy
against tuberculosis antigens and a weak cell-mediated immune response. This can occur in
the setting of HIV, sarcoidosis, and other illnesses.

Telegram Group Invite Link: https://t.me/joinchat/zyFOIg0j5goxY2E1


Telegram Group Invite Link: https://t.me/USMLEWorldStep1

smoke

This patient has fever, cough, and focal crackles, which are findings consistent
with pneumonia. Transmission typically occurs via droplet spread of a bacterium
(eg, Streptococcus pneumoniae) with subsequent colonization of the nasopharynx and
microaspiration into the lungs.
Respiratory defense mechanisms often prevent infection of the lower respiratory tract, and the
first line of defense is the ciliated epithelium and mucus-producing cells that line the respiratory
tract. Pathogens become entrapped in mucus and are transported upward and out of the
nasopharynx by the sweeping cilia. Other defense mechanisms include binding by
immunoglobulin (eg, secretory IgA, opsonizing IgG and IgM), phagocytosis by alveolar and
interstitial macrophages, and cell-mediated cytotoxic activity.
Secondhand smoke exposure disrupts natural host defenses and increases the risk of
pneumonia, particularly in infants who have a parent that smokes. Cigarette smoke induces
mucus overproduction and impairs ciliary function, leading to delayed clearance of pathogens
and respiratory secretions. In addition to respiratory infections, children with secondhand
smoke exposure are also at increased risk for asthma and wheezing and are more likely to
have recurrent otitis media.
(Choice A) A decreased number of monocytes may occur with bone marrow dysfunction,
which would typically cause severe, disseminated infections (not seen in this patient). In
addition, although the phagocytic function of alveolar macrophages (differentiated type of
monocyte) is impaired in patients with cigarette smoke exposure, these cells are typically
present in increased (not decreased) numbers.
(Choice B) Although cigarette smoke exposure is associated with increased risk of asthma,
airway hyperreactivity causes bronchoconstriction and wheezing, not pneumonia as seen in this
patient.
(Choice D) Inhibited B cell maturation, as seen with X-linked agammaglobulinemia, typically
presents with recurrent sinopulmonary infections in young boys, not girls. In addition, cigarette
smoke can increase (not decrease) IgE production due to allergic sensitization.

Telegram Group Invite Link: https://t.me/joinchat/zyFOIg0j5goxY2E1


https://t.me/joinchat/zyFOIg0j5goxY2E1

Telegram Group Invite Link: https://t.me/USMLEWorldStep1

(Choice E) Reduced alveolar surface area occurs with emphysema, which is a late lung finding
in patients who smoke. Emphysema would not be present in infancy and does not cause
pneumonia.

Alveolar
hypereventilation

This patient with acute-onset dyspnea and chest discomfort likely has an acute pulmonary
embolism (PE). PE leads to increased dead-space ventilation with a consequent
ventilation/perfusion (V/Q) mismatch that causes hypoxemia. The acute hypoxemia along with
pulmonary parenchymal inflammation triggers an increase in respiratory drive

Telegram Group Invite Link: https://t.me/joinchat/zyFOIg0j5goxY2E1


Telegram Group Invite Link: https://t.me/USMLEWorldStep1

and hyperventilation.

Because the rate of CO2 removal is closely tied to ventilation rate, alveolar hyperventilation leads
to increased expiration of CO2 with resulting hypocapnia (low arterial partial pressure of
carbon dioxide [PaCO2]) and respiratory alkalosis. In contrast, the rate of O2 absorption
becomes capped once hemoglobin is saturated, which occurs at relatively low arterial partial
pressure of oxygen (PaO2) levels (eg, hemoglobin is 85% saturated at PaO2 50 mm

Telegram Group Invite Link: https://t.me/joinchat/zyFOIg0j5goxY2E1


https://t.me/joinchat/zyFOIg0j5goxY2E1

Telegram Group Invite Link: https://t.me/USMLEWorldStep1

Hg). Therefore, the blood in highly ventilated lung regions cannot absorb extra O2 to
compensate for poorly ventilated regions, and hyperventilation does not significantly increase
PaO2.
It follows that patients with an acute V/Q mismatch (eg, due to PE or pneumonia) typically have
hypocapnia with respiratory alkalosis and persistent hypoxemia. Without treatment, prolonged
hyperventilation can lead to respiratory muscle fatigue (Choice E) with consequent respiratory
failure; these patients will have hypoventilation with hypercapnia and respiratory acidosis.
(Choices B and D) Both decreased chest wall compliance, as occurs in obesity hypoventilation
syndrome, and poor respiratory drive, as occurs in opioid overdose and some types of stroke,
lead to hypoventilation with hypoxemia and increased PaCO2 levels.
(Choice C) Expiratory air trapping occurs in chronic obstructive lung disease. These patients
often have chronic respiratory acidosis (high PaCO2) that may worsen (acute on chronic
respiratory acidosis) during exacerbation.

Sepsis
This patient with pyelonephritis (eg, flank pain, pyuria) has likely developed sepsis (eg, fever,
peripheral blood leukocytosis). Sepsis is a florid host inflammatory response to an infectious
pathogen that can lead to widespread dysfunction of multiple organ systems. Cardiovascular
manifestations include profound vasodilation resulting in low systemic vascular resistance and
hypotension. Refractory hypotension (ie, vasoplegia) despite adequate intravascular volume
(ie, normal jugular venous pressure), as in this patient, indicates progression to septic shock.
Septic shock is an extreme manifestation of sepsis, in which vasodilatory mediators overwhelm
endogenous vasoconstrictor mechanisms. Key vasodilators include nitric oxide and
prostaglandins; their production is upregulated by acute-phase cytokines such as tumor
necrosis factor-α, IL-1, and IL-6 released from innate immune cells and endothelium exposed to
bacterial cell wall glycoproteins and endotoxins (Choices C and D).
The normal physiologic response to hypotension is compensatory secretion of vasopressin
(antidiuretic hormone) by the posterior pituitary. In addition to promoting water retention (via
renal collecting duct V2 receptors), vasopressin is one of the most potent vasoconstrictors (via
arteriolar smooth muscle V1 receptors). In septic shock, however, vasopressin is suppressed,
possibly due to impaired baroreceptor reflexes and/or depletion of endogenous stores.
For this reason, infusion of vasopressin can be helpful during treatment of septic shock, often
decreasing the need for other pharmacologic vasoconstrictor agents (eg, norepinephrine).
(Choice A) C-reactive protein is an acute-phase protein produced in the liver in response to
inflammation; it activates the classical complement pathway and promotes phagocytosis,
thereby facilitating clearance of bacteria and cellular debris. Levels of C-reactive protein are
elevated in sepsis.

Telegram Group Invite Link: https://t.me/joinchat/zyFOIg0j5goxY2E1


Telegram Group Invite Link: https://t.me/USMLEWorldStep1

Telegram Group Invite Link: https://t.me/joinchat/zyFOIg0j5goxY2E1


https://t.me/joinchat/zyFOIg0j5goxY2E1

Telegram Group Invite Link: https://t.me/USMLEWorldStep1

(Choice B) Endotoxin (lipopolysaccharide) a cell wall component of gram-negative bacteria


that triggers an acute inflammatory response by binding to toll-like receptor 4 on immune
cells. The vast majority of urinary tract infections are caused by gram-negative bacteria, and
nitrite production is generally exclusive to gram-negative bacteria; therefore, endotoxin
concentrations are likely markedly elevated in this patient.

Telegram Group Invite Link: https://t.me/joinchat/zyFOIg0j5goxY2E1


Telegram Group Invite Link: https://t.me/USMLEWorldStep1

CYP450

This patient was most likely treated with ciprofloxacin (a fluoroquinolone) for her urinary tract
infection and experienced subsequent theophylline toxicity. Theophylline is an adenosine
receptor antagonist and phosphodiesterase inhibitor that is sometimes used as an alternate
therapy for asthma and chronic obstructive pulmonary disease. It causes bronchodilation
primarily by increasing intracellular cyclic AMP levels (similar to beta-adrenergic agonists) and
also has mild anti-inflammatory effects.
Theophylline is metabolized predominantly by hepatic cytochrome oxidases. Inhibition of
these enzymes by concurrent illness (eg, infection with fever) or ingestion of certain drugs or
substances (eg, ciprofloxacin) can raise serum concentrations and cause theophylline toxicity
due to the drug's narrow therapeutic index. Toxicity presents with excessive CNS stimulation
(eg, tremor, insomnia, seizures), gastrointestinal disturbances, and cardiovascular
abnormalities (eg, hypotension, tachycardia, cardiac arrhythmias).
(Choices A, B, and E) Amoxicillin and cephalexin are beta-lactams that inhibit bacterial cell
wall synthesis. Nitrofurantoin inactivates or alters bacterial ribosomal proteins to inhibit protein

Telegram Group Invite Link: https://t.me/joinchat/zyFOIg0j5goxY2E1


https://t.me/joinchat/zyFOIg0j5goxY2E1

Telegram Group Invite Link: https://t.me/USMLEWorldStep1

and cell wall synthesis. These antibiotics do not interact significantly with the hepatic
cytochrome oxidase system.
(Choice D) Haloperidol blocks postsynaptic dopamine (D2) receptors in the brain but does not
interact significantly with theophylline to cause toxicity. Chronic haloperidol use can cause
neuroleptic malignant syndrome presenting with fever, mental status changes, muscle rigidity,
and autonomic instability.
Sarcoidosis

This patient with cough, night sweats, and bilateral hilar adenopathy likely has sarcoidosis, a
systemic inflammatory disease characterized by noncaseating granulomas (non-necrotic
aggregates of epithelioid macrophages frequently with multinucleated giant cells). It typically
presents in young adults (women > men) and occurs more commonly in African Americans.
Any organ can be affected by sarcoidosis; however, the lungs (eg, reticular/nodular infiltrates),
lymph nodes (eg, hilar adenopathy), skin (erythematous rash), and eyes (eg, anterior uveitis)
are often involved. In addition to pulmonary symptoms (eg, cough, chest pain,
dyspnea), constitutional symptoms (including fever, weight loss, fatigue, night sweats, and

Telegram Group Invite Link: https://t.me/joinchat/zyFOIg0j5goxY2E1


Telegram Group Invite Link: https://t.me/USMLEWorldStep1

arthralgias) are common.

(Choice A) Acute HIV infection can present with constitutional symptoms and weight loss;
however, noncaseating granulomas are not consistent with isolated HIV infection and suggest
sarcoidosis.
(Choice B) Hodgkin lymphoma can cause supraclavicular adenopathy and B symptoms (eg,
weight loss, fever, night sweats); however, biopsy would demonstrate Reed-Sternberg

Telegram Group Invite Link: https://t.me/joinchat/zyFOIg0j5goxY2E1


https://t.me/joinchat/zyFOIg0j5goxY2E1

Telegram Group Invite Link: https://t.me/USMLEWorldStep1

cells surrounded by an inflammatory infiltrate.

(Choices C and D) Adenocarcinoma and squamous cell carcinoma of the lung can cause
constitutional symptoms and cough; however, a mass would be expected on imaging. In
addition, biopsy of squamous cell carcinoma would show sheets or nests of malignant cells with
squamous differentiation (eg, keratinization), whereas adenocarcinoma would show glandular

Telegram Group Invite Link: https://t.me/joinchat/zyFOIg0j5goxY2E1


https://t.me/joinchat/zyFOIg0j5goxY2E1

USMLE e e r m h nne s
Me est i e s: https://t.me/USMLEMe est

K p n ssr m i e s: https://t.me/USMLEK p n

USMLE i e s Step : https://t.me/USMLE Step

e ker i e s: https://t.me/USMLE e ker

r s e n i e s Step : https://t.me/USMLE r sAn e n

r s e n i e s Step K: https://t.me/USMLE r sAn e n Step K

Ph se i e s: https://t.me/USMLEPh se

Pi ri e i e s: https://t.me/USMLEPi ri e

P th m i e s: https://t.me/USMLEP th m

nA i s i h it : https://t.me/USMLE nA i s

Am ss r p: https://t.me/USMLEAm ss

Le t ri nk: https://t.me/USMLELe t ri

En p int P s: https://t.me/USMLEEn p int

USMLE MEs: https://t.me/USMLE ME

L E Me E : https://t.me/USMLE n ineMe E

USMLE r Step : https://t.me/USMLE r Step

USMLE r Step K: https://t.me/USMLE r Step K

USMLE r Step nk: https://t.me/USMLE r Step

USMLE Anki: https://t.me/USMLEAnki

KissPrep: https://t.me/USMLEKissPrep
Telegram Group Invite Link: https://t.me/USMLEWorldStep1

differentiation with atypia.

Telegram Group Invite Link: https://t.me/joinchat/zyFOIg0j5goxY2E1


https://t.me/joinchat/zyFOIg0j5goxY2E1

Telegram Group Invite Link: https://t.me/USMLEWorldStep1

Telegram Group Invite Link: https://t.me/joinchat/zyFOIg0j5goxY2E1


Telegram Group Invite Link: https://t.me/USMLEWorldStep1

(Choice E) Mycobacterium avium complex can cause similar symptoms with noncaseating
(and caseating) granulomas but typically occurs in severely immunocompromised patients. In
addition, Mycobacterium avium pulmonary disease typically causes cavitations and pulmonary

Telegram Group Invite Link: https://t.me/joinchat/zyFOIg0j5goxY2E1


https://t.me/joinchat/zyFOIg0j5goxY2E1

Telegram Group Invite Link: https://t.me/USMLEWorldStep1

infiltrates on imaging.

(Choice F) Tuberculosis can cause weight loss, cough, and night sweats, particularly in those
with risk factors (eg, immunosuppression, imprisonment, immigrants, HIV positive). However,

Telegram Group Invite Link: https://t.me/joinchat/zyFOIg0j5goxY2E1


Telegram Group Invite Link: https://t.me/USMLEWorldStep1

biopsy would typically show caseating granulomas and acid-fast bacilli.

Telegram Group Invite Link: https://t.me/joinchat/zyFOIg0j5goxY2E1


https://t.me/joinchat/zyFOIg0j5goxY2E1

Telegram Group Invite Link: https://t.me/USMLEWorldStep1

sarcoidosis

This patient likely has sarcoidosis, a chronic multisystem disorder characterized by the
formation of noncaseating granulomas (eg, epithelioid macrophages, multinucleated giant
cells). It typically affects young adults and is more prevalent in African American individuals
and in women.
Any organ can be affected, but the lungs are initially involved in most cases. Patients often
have the insidious onset of respiratory symptoms (eg, cough, dyspnea, chest pain)
accompanied by fatigue, fever, and weight loss. The disease may also be incidentally detected
in asymptomatic patients with typical chest x-ray findings (eg, bilateral hilar
lymphadenopathy, reticular opacities).
(Choice A) Allergic bronchopulmonary aspergillosis commonly occurs in patients with asthma
or cystic fibrosis and presents with recurrent disease exacerbations. Microscopic examination
would likely show mucin within the bronchial lumens with numerous eosinophils and
rare Aspergillus hyphae.
(Choice B) Churg-Strauss syndrome (eosinophilic granulomatosis with polyangiitis) usually
presents with chronic rhinosinusitis, asthma, and eosinophilia. The granulomas seen with
Churg-Strauss syndrome typically have eosinophilic infiltration and extensive necrosis.

Telegram Group Invite Link: https://t.me/joinchat/zyFOIg0j5goxY2E1


Telegram Group Invite Link: https://t.me/USMLEWorldStep1

(Choices C, E, and G) Desquamative interstitial pneumonia (DIP), idiopathic pulmonary


fibrosis (IPF), and pulmonary alveolar proteinosis (PAP) are associated with cigarette smoking
and typically present with the insidious onset of progressive dyspnea and chronic
cough. However, DIP is characterized by numerous intraalveolar macrophages, IPF is
associated with patchy fibrosis and inflammation, and PAP demonstrates lipoproteinaceous
material within the alveoli.

(Choice D) Hodgkin lymphoma typically presents with lymphadenopathy and B symptoms (eg,
fever, night sweats, weight loss), and histopathology demonstrates Reed-Sternberg cells

Telegram Group Invite Link: https://t.me/joinchat/zyFOIg0j5goxY2E1


https://t.me/joinchat/zyFOIg0j5goxY2E1

Telegram Group Invite Link: https://t.me/USMLEWorldStep1

surrounded by inflammatory cells.

(Choice F) Systemic sclerosis results in progressive collagen deposition and fibrosis of the skin
and visceral organs (eg, gastrointestinal tract, lungs, kidneys), which can lead to dyspnea and
cough. However, biopsy would demonstrate excessive collagen deposition, not granulomas.
(Choice I) Small cell carcinoma is a highly malignant lung cancer that is strongly associated
with smoking. Histopathologic features include small cells with scant cytoplasm, granular

Telegram Group Invite Link: https://t.me/joinchat/zyFOIg0j5goxY2E1


Telegram Group Invite Link: https://t.me/USMLEWorldStep1

nuclear chromatin (salt and pepper pattern), and indistinct nucleoli.

Telegram Group Invite Link: https://t.me/joinchat/zyFOIg0j5goxY2E1


https://t.me/joinchat/zyFOIg0j5goxY2E1

Telegram Group Invite Link: https://t.me/USMLEWorldStep1

Chest
xray

In patients with neck injuries, it is important to remember that the lung apices and cervical
pleura extend above the clavicle and first rib through the superior thoracic aperture. Stab
wounds immediately above the clavicle and lateral to the manubrium can puncture the pleura
and cause pneumothorax, tension pneumothorax, or hemothorax.
This patient most likely has a tension pneumothorax as a result of his pleural injury. As an
increasing volume of air accumulates within the pleural space, the lungs and mediastinum
deviate to the opposite side of the chest. Increased pressure within the chest cavity decreases
systemic venous return to the heart, lowering cardiac output. Signs and symptoms of tension
pneumothorax include tachycardia, hypotension, tachypnea, hypoxemia, and hyperresonance
to percussion and absence of breath sounds on the affected side. Treatment is with emergency

Telegram Group Invite Link: https://t.me/joinchat/zyFOIg0j5goxY2E1


Telegram Group Invite Link: https://t.me/USMLEWorldStep1

needle thoracostomy or chest tube.

(Choice A) The accessory nerve (cranial nerve XI) innervates the sternocleidomastoid and
trapezius muscles. This nerve may be injured during surgery involving the posterior triangle of
the neck (a region bounded by the sternocleidomastoid muscle, trapezius muscle, and clavicle).
(Choice B) The ansa cervicalis arises from the C1, C2, and C3 nerve roots and innervates the
sternohyoid, sternothyroid, and omohyoid muscles of the anterior neck. Penetrating trauma to
the neck above the cricoid cartilage can injure this nerve.
(Choice C) The carotid body, which contains O2, CO2, and H+ chemoreceptors, lies at the
bifurcation of the common carotid artery (just inferior to the hyoid bone).
(Choice D) The inferior thyroid artery arises from the thyrocervical trunk and courses posterior
to the carotid artery and jugular vein to supply the inferior pole of the thyroid gland. Injury to the
inferior thyroid artery is commonly associated with hoarseness because it runs adjacent to the
recurrent laryngeal nerve.

Telegram Group Invite Link: https://t.me/joinchat/zyFOIg0j5goxY2E1


https://t.me/joinchat/zyFOIg0j5goxY2E1

Telegram Group Invite Link: https://t.me/USMLEWorldStep1

Asthma
Asthma is a disease of airway inflammation and bronchial hyperreactivity, with variable airflow
obstruction that improves with bronchodilator medications. Classic symptoms include cough,
shortness of breath, and wheeze, although these are nonspecific. Most patients with asthma
have a chronic reduction in the FEV1/FVC ratio (≤70%; normal ~80%) due to increased
expiratory airflow resistance. However, asymptomatic patients may have normal lung function
tests, with alterations in lung function occurring only during exacerbations. This patient with
intermittent shortness of breath has a normal FEV1/FVC ratio, therefore, further studies are
required to exclude the diagnosis of asthma.
Bronchial challenge testing is a highly sensitive test with high negative predictive
value used to assess bronchial hyperreactivity and exclude asthma. A provocative stimulus
(typically aerosolized methacholine, a cholinergic muscarinic agonist) is administered at
increasing concentrations to induce bronchoconstriction. Patients with asthma are
hyperresponsive to this stimulus and experience a decline in FEV1 at lower doses than
nonasthmatics. However, the test is nonspecific and positive results may also be seen with
chronic obstructive pulmonary disease, cystic fibrosis, and allergic rhinitis.
(Choice A) Although peripheral eosinophil counts may be elevated in asthmatics, the presence
of eosinophilia is not a sensitive or specific indicator of asthma and may be seen in other
allergic diseases (eg, allergic rhinitis, drug reactions), parasitic infections (eg, strongyloides),
and certain myeloid neoplasms.
(Choice B) Most patients with asthma are sensitive to a variety of inhaled allergens, and in
such patients, skin tests to various allergens will provoke a localized, immediate, type I
hypersensitivity reaction. However, patients may have allergic disease without asthma and
patients with nonallergic (intrinsic) asthma may not have allergies; therefore this test is not
useful for diagnosing asthma.
(Choice D) During an asthma attack, the lungs may appear hyperinflated on chest x-
ray. However, chest x-rays are often normal between attacks; therefore, a chest x-ray cannot
exclude asthma.
(Choice E) Although serum IgE levels are generally elevated in patients with allergic asthma,
patients with nonimmune-mediated asthma have normal levels of serum IgE. Intrinsic asthma
can be precipitated by pulmonary infections (especially viral), aspirin ingestion, cold air, inhaled
irritants (eg, perfumes), stress, and exercise.

Telegram Group Invite Link: https://t.me/joinchat/zyFOIg0j5goxY2E1


Telegram Group Invite Link: https://t.me/USMLEWorldStep1

ARDS

This patient who developed hypoxia, pulmonary edema (bilateral pulmonary infiltrates), and
protein-rich fluid in the alveoli had acute respiratory distress syndrome (ARDS). ARDS is a
severe inflammatory reaction that occurs in the lungs and results in respiratory failure with
hypoxemia and noncardiogenic pulmonary edema. Risk factors include sepsis,
severe trauma (eg, long-bone fractures with fat embolism, extensive tissue injury),
and transfusion of blood products, all of which can initiate pulmonary injury and trigger a
cascade of inflammation:
• In response to injury, alveolar macrophages release multiple proinflammatory cytokines
that recruit neutrophils to the lungs (Choice A).
• Upon arrival to pulmonary tissue, the neutrophils release inflammatory mediators (eg,
proteases, free radicals). This increases inflammation and leads to damage to the
pulmonary endothelium and alveolar pneumocytes.

Telegram Group Invite Link: https://t.me/joinchat/zyFOIg0j5goxY2E1


https://t.me/joinchat/zyFOIg0j5goxY2E1

Telegram Group Invite Link: https://t.me/USMLEWorldStep1

• The alveolar-capillary barrier, which is formed by the endothelial cells and type I
pneumocytes, is then destroyed, allowing fluid and red blood cells to escape the
vascular and interstitial spaces and pour into the alveoli. There, they combine with
material from necrotic cells and form a thick proteinaceous fluid and a hyaline
membrane.
• Destruction of type II pneumocytes, which normally produce surfactant and proliferate in
response to injury, leads to alveolar collapse (but does not cause destruction of the
alveolar capillary barrier) (Choice F).
The net effect is impaired alveolar gas exchange and respiratory failure, with affected patients
having a high mortality rate.
(Choices B and D) Ciliated epithelium and goblet cells line the larger airways. Goblet cells
secrete mucus that traps particulate matter (eg, dust, bacteria), while ciliated epithelial cells
assist in its elimination by sweeping the mucus up the bronchi and trachea to the pharynx,
where it is swallowed. These cells are not prominently involved in ARDS.
(Choice C) Club cells (formerly called Clara cells) are nonciliated, secretory constituents of the
terminal respiratory epithelium. They secrete protein and surfactant components and help
detoxify inhaled substances (eg, tobacco smoke). They are not prominently involved in ARDS.

Pneumonia

This middle-aged man presenting with new-onset fevers, productive cough, and a dense lobar
infiltrate likely has community-acquired pneumonia (CAP). CAP in otherwise healthy
individuals is most commonly caused by Streptococcus pneumoniae, the most common
bacterial etiology worldwide. Tobacco use further increases its risk. In nonelderly patients,
pneumococcal pneumonia presents with abrupt-onset fevers, rigors, tachypnea, and productive
cough with consolidation in one lobe of the lungs.
Neutrophil myeloperoxidase is responsible for the green color of pus and sputum in bacterial
infections. It is a blue-green heme-based pigmented molecule contained within the azurophilic
granules of neutrophils and catalyzes the production of hypochlorous acid from chloride and

Telegram Group Invite Link: https://t.me/joinchat/zyFOIg0j5goxY2E1


Telegram Group Invite Link: https://t.me/USMLEWorldStep1

hydrogen peroxide during the phagocytic respiratory burst.

(Choices A and B) Both the "currant jelly" sputum seen in pneumonia caused by Klebsiella
pneumoniae and the "rusty" color occasionally seen in pneumococcal pneumonia are due to
extravasation of red blood cells and hemoglobin into the sputum caused by extensive
inflammation and necrosis.
(Choice C) Pseudomonas pneumonia can cause a blue-green pigment due to the production of
pyocyanin, but Pseudomonas is not a common cause of CAP in otherwise healthy adults.
(Choice D) Respiratory epithelium sloughing and mucopolysaccharide production can
contribute to the formation of sputum in patients with respiratory infection but does not
contribute to sputum's green color.

Telegram Group Invite Link: https://t.me/joinchat/zyFOIg0j5goxY2E1


https://t.me/joinchat/zyFOIg0j5goxY2E1

Telegram Group Invite Link: https://t.me/USMLEWorldStep1

Pulmonary
embolism

Sudden-onset shortness of breath and chest pain in a hospitalized patient should raise
suspicion for pulmonary embolism (PE). This patient has 2 important risk factors for
PE: Immobilization (causes venous stasis) and recent surgery (inflammation induces a
hypercoagulable state). The risk of PE is high after large orthopedic procedures; prophylactic
anticoagulation reduces but does not eliminate this risk. Thrombi most commonly originate in

Telegram Group Invite Link: https://t.me/joinchat/zyFOIg0j5goxY2E1


Telegram Group Invite Link: https://t.me/USMLEWorldStep1

the deep veins of the pelvis and lower extremities before embolizing to the lungs. Although the
risk of fat embolism is also increased following a long-bone fracture, this patient lacks the typical
skin rash and neurologic findings (eg, confusion).
Thrombotic occlusion of the pulmonary circulation leads to redistribution of blood flow in the
lungs, resulting in a ventilation/perfusion (V/Q) mismatch. With many well-ventilated alveoli
now inaccessible to blood flow (dead-space ventilation), the remaining accessible alveoli are
unable to fully oxygenate the volume of blood that continues to flow through the pulmonary
circulation, and hypoxemia results. As with any V/Q mismatch that affects only part of the
lungs, supplemental oxygen can help correct the hypoxemia by increasing the alveolar partial
pressure of oxygen, allowing accessible alveoli to transfer additional oxygen to the blood.
(Choice A) Alveolar hypoventilation refers to a global decrease in the alveolar partial pressure
of oxygen caused by a decrease in tidal volume or respiratory rate. Patients also develop
respiratory acidosis due to decreased CO2 excretion. This patient's respiratory alkalosis is more
typical of V/Q mismatch with subsequent hyperventilation, as V/Q mismatch does not hinder

Telegram Group Invite Link: https://t.me/joinchat/zyFOIg0j5goxY2E1


https://t.me/joinchat/zyFOIg0j5goxY2E1

Telegram Group Invite Link: https://t.me/USMLEWorldStep1

CO2 removal as much as blood oxygenation.

Telegram Group Invite Link: https://t.me/joinchat/zyFOIg0j5goxY2E1


Telegram Group Invite Link: https://t.me/USMLEWorldStep1

(Choice B) Decreased blood oxygen-carrying capacity (ie, decreased hemoglobin


concentration) can occur due to postoperative bleeding. Although a decrease in hemoglobin
can lead to tissue hypoxia, the measured PaO2 (indicative of dissolved oxygen content rather
than hemoglobin-bound oxygen content) will not be affected.
(Choice C) Increased tissue oxygen extraction would cause venous oxygen levels to decrease
but would not typically affect the arterial oxygen content.
(Choice D) Hypoxemia due to a primary defect that impairs gas diffusion between the alveoli
and pulmonary capillaries can occur in chronic lung diseases, including pulmonary fibrosis and

Telegram Group Invite Link: https://t.me/joinchat/zyFOIg0j5goxY2E1


https://t.me/joinchat/zyFOIg0j5goxY2E1

Telegram Group Invite Link: https://t.me/USMLEWorldStep1

emphysema.

Telegram Group Invite Link: https://t.me/joinchat/zyFOIg0j5goxY2E1


Telegram Group Invite Link: https://t.me/USMLEWorldStep1

Obstructive sleep apnea

This patient has several features that suggest obstructive sleep apnea (OSA), including
daytime somnolence, nonrestorative sleep, and elevated blood pressure in the setting of
underlying obesity. OSA is caused by closure of the upper airway due to relaxation of
pharyngeal muscle tone during sleep. Additional contributing factors include obesity, tonsillar
hypertrophy, and hypothyroidism. When the airway is occluded, PO2 declines and PCO2 rises
until chemoreceptors in the carotid body and brainstem trigger arousal and pharyngeal tone
returns. Sleep is repeatedly disrupted throughout the night, even in the absence of cortical
awareness.
Typical symptoms include excessive daytime sleepiness, morning headaches, cognitive
impairment, and depression. Most patients also experience loud snoring due to partial closure
of the airway. Chronic OSA can lead to systemic and pulmonary hypertension with right heart
failure and an increased risk for cardiac arrhythmias.

(Choice A) Central sleep apnea is due to diminished respiratory drive from a neurologic
disorder. Symptoms may superficially resemble OSA, but it is usually associated with
significant underlying chronic illness (eg, congestive heart failure, cerebrovascular disease,
renal insufficiency) and is not more common in obesity.

Telegram Group Invite Link: https://t.me/joinchat/zyFOIg0j5goxY2E1


https://t.me/joinchat/zyFOIg0j5goxY2E1

Telegram Group Invite Link: https://t.me/USMLEWorldStep1

(Choice B) Narcolepsy is characterized by poorly regulated rapid eye movement (REM)


sleep. It frequently causes excessive daytime sleepiness, but patients also suffer from
cataplexy, sleep attacks, hypnagogic/hypnopompic hallucinations, and sleep paralysis.
(Choice C) Obesity hypoventilation syndrome (Pickwickian syndrome) is caused by restricted
expansion of the chest wall due to severe obesity. This leads to hypoventilation with a
chronically elevated PCO2 and reduced PO2. This patient's normal blood gases and mild obesity
are not consistent with obesity hypoventilation.
(Choice E) Patients with simple insomnia may have daytime fatigue, but nonrestorative sleep,
morning headaches, and elevated blood pressure suggest OSA.
(Choice F) Restless leg syndrome is characterized by vague discomfort in the limbs that is
brought on when trying to sleep and relieved with movement. Symptoms may recur through the
night and lead to nonrestorative sleep, but patients are usually aware of the symptoms.

Pulmonary
hypertension

This young woman with progressive dyspnea and fatigue and a loud pulmonic component of S2
on physical examination likely has pulmonary arterial hypertension (PAH). PAH results from
progressive remodeling of the small- and medium-sized pulmonary

Telegram Group Invite Link: https://t.me/joinchat/zyFOIg0j5goxY2E1


Telegram Group Invite Link: https://t.me/USMLEWorldStep1

arteries/arterioles. Endothelial dysfunction leads to an increase in


vasoconstrictive, proproliferative mediators (eg, endothelin, thromboxane A2) and
a decrease in vasodilative, antiproliferative mediators (eg, nitric oxide, prostacyclin). The
relative imbalance of these mediators leads to vasoconstriction and smooth muscle
proliferation with intimal thickening of the vascular walls; consequently, there is increased
pulmonary vascular resistance and elevated pulmonary arterial pressure. Over time, the right
ventricle is unable to pump against the increased afterload, and right-sided heart failure
develops.
Medical therapy for PAH targets the mediator imbalance created by endothelial
dysfunction. Endothelin receptor antagonists (eg, bosentan) are used to reduce
vasoconstriction, while prostacyclin analogues (eg, epoprostenol) and nitric oxide–enhancing
agents (eg, sildenafil) are used to promote vasodilation. All of these agents help reduce tissue
proliferation.
(Choices B, C, D, and E) These changes in the levels of prostacyclin, thromboxane, and nitric
oxide are not consistent with the changes that occur in PAH.
Pulmonary function test

Cystic fibrosis (CF) is an autosomal recessive disorder caused by a mutation (eg, ∆F508) in
the gene encoding the CF transmembrane conductance regulator. A defect in this chloride
channel results in the build-up of thick, dehydrated mucus, which causes progressive damage
to various organ systems, such as the respiratory and gastrointestinal tracts.
The lungs are the predominant organ affected in CF due to the accumulation of viscous
secretions in the airways. Bacteria colonize the airways and cause recurrent infection and
chronic inflammation that eventually leads to scarring of the bronchial walls (ie,
bronchiectasis). Progressive bronchiectasis (ie, weakened, dilated bronchioles that collapse
easily) and mucus-plugging result in obstructive lung disease.
An obstructive pattern on spirometry is characterized by decreased forced expiratory volume in
1 second (FEV1) and forced vital capacity (FVC). Because the decrease in FEV1 is more
profound in obstructive disease, FEV1/FVC ratio is also reduced. The airway obstruction
prevents full expiration (ie, causes air trapping), leading to increased residual volume (RV)

Telegram Group Invite Link: https://t.me/joinchat/zyFOIg0j5goxY2E1


https://t.me/joinchat/zyFOIg0j5goxY2E1

Telegram Group Invite Link: https://t.me/USMLEWorldStep1

and hyperinflated lungs with increased total lung capacity (TLC). This progressive increase is
clinically apparent by a barrel-shaped chest on examination and lung hyperexpansion on x-ray.

Telegram Group Invite Link: https://t.me/joinchat/zyFOIg0j5goxY2E1


Telegram Group Invite Link: https://t.me/USMLEWorldStep1

Telegram Group Invite Link: https://t.me/joinchat/zyFOIg0j5goxY2E1


https://t.me/joinchat/zyFOIg0j5goxY2E1

Telegram Group Invite Link: https://t.me/USMLEWorldStep1

(Choices A and C) As RV increases or decreases, TLC follows concordantly; therefore, these


results are unlikely to represent a true clinical situation.
(Choice B) Restrictive lung disease (eg, pulmonary fibrosis) is characterized by a normal-to-
increased FEV1/FVC ratio because FVC decreases to a greater extent than FEV1. RV and
TLC are both decreased.
(Choice E) Normal RV with increased TLC can be seen in highly trained athletes due to
increased FVC.
Alpha 1 antitrypsin

Telegram Group Invite Link: https://t.me/joinchat/zyFOIg0j5goxY2E1


Telegram Group Invite Link: https://t.me/USMLEWorldStep1

This patient has alpha-1 antitrypsin (AAT) deficiency, an autosomal codominant disorder that
can affect the lungs and liver. Produced primarily in the liver, AAT is a serum protein that
inhibits several different proteolytic enzymes (in particular, neutrophil elastase), thereby
reducing tissue damage caused by inflammation. Most individuals with AAT deficiency are
homozygous for the Z allele, which causes decreased secretion of AAT secondary to abnormal
protein folding. The diagnosis of AAT deficiency is established by measurement of the serum
AAT level, followed by confirmatory genetic testing.
Most individuals with AAT deficiency eventually develop severe panacinar emphysema due to
unchecked destruction of the interalveolar septa (which contain large amounts of
elastin). Smoking plays a synergistic role in the disease process by inducing lung inflammation
and permanently inactivating AAT through oxidation of a crucial methionine residue. Therefore,
smokers with AAT deficiency tend to develop dyspnea at a median age of 36 versus a median
age of 51 in nonsmokers.
Some patients with AAT deficiency develop liver disease due to intrahepatocyte
accumulation of polymerized AAT molecules; the liver disease can progress to cirrhosis (the
second most common cause of death behind emphysema in this population) and hepatocellular
carcinoma. Histologically, intracellular granules of unsecreted AAT are seen within the
periportal hepatocytes. These globules resist digestion by diastase (an enzyme that breaks

Telegram Group Invite Link: https://t.me/joinchat/zyFOIg0j5goxY2E1


https://t.me/joinchat/zyFOIg0j5goxY2E1

USMLE e e r m h nne s
Me est i e s: https://t.me/USMLEMe est

K p n ssr m i e s: https://t.me/USMLEK p n

USMLE i e s Step : https://t.me/USMLE Step

e ker i e s: https://t.me/USMLE e ker

r s e n i e s Step : https://t.me/USMLE r sAn e n

r s e n i e s Step K: https://t.me/USMLE r sAn e n Step K

Ph se i e s: https://t.me/USMLEPh se

Pi ri e i e s: https://t.me/USMLEPi ri e

P th m i e s: https://t.me/USMLEP th m

nA i s i h it : https://t.me/USMLE nA i s

Am ss r p: https://t.me/USMLEAm ss

Le t ri nk: https://t.me/USMLELe t ri

En p int P s: https://t.me/USMLEEn p int

USMLE MEs: https://t.me/USMLE ME

L E Me E : https://t.me/USMLE n ineMe E

USMLE r Step : https://t.me/USMLE r Step

USMLE r Step K: https://t.me/USMLE r Step K

USMLE r Step nk: https://t.me/USMLE r Step

USMLE Anki: https://t.me/USMLEAnki

KissPrep: https://t.me/USMLEKissPrep
https://t.me/joinchat/zyFOIg0j5goxY2E1

Telegram Group Invite Link: https://t.me/USMLEWorldStep1

down glycogen) and appear reddish-pink with the periodic acid–Schiff stain.

(Choice A) Bronchial hyperreactivity is a hallmark of chronic asthma.


(Choice C) Interstitial infiltration is a finding in interstitial pneumonitis.
(Choice D) Intraalveolar substance accumulation is a finding in alveolar proteinosis.
(Choice E) Pulmonary congestion is a finding in congestive heart failure.
(Choice F) Pulmonary vascular obstruction occurs in pulmonary embolism or vasculopathy.

Telegram Group Invite Link: https://t.me/joinchat/zyFOIg0j5goxY2E1


Telegram Group Invite Link: https://t.me/USMLEWorldStep1

Heart
failure

This patient with dyspnea, orthopnea, and pulmonary crackles has left-sided heart
failure (LHF) most likely due to long-standing, poorly controlled hypertension. Hypertensive
heart disease typically manifests as heart failure due to concentric left ventricular (LV)
hypertrophy and consequent LV diastolic dysfunction. LHF of any cause (eg, LV systolic
dysfunction, valvular dysfunction) will result in higher diastolic filling pressures. This
increase in pressure is transmitted backward to the left atrium and pulmonary veins, resulting
in pulmonary venous congestion and consequent elevations in pulmonary capillary and
pulmonary arterial pressure. The resulting pulmonary hypertension (PH) can lead to right-
sided heart failure with jugular venous distension and peripheral edema.
Over time, remodeling of the pulmonary vasculature occurs with increased smooth muscle
cell proliferation (medial hypertrophy) and collagen deposition (intimal thickening and
fibrosis). The remodeling is less extensive than in (primary) pulmonary arterial hypertension;

Telegram Group Invite Link: https://t.me/joinchat/zyFOIg0j5goxY2E1


https://t.me/joinchat/zyFOIg0j5goxY2E1

Telegram Group Invite Link: https://t.me/USMLEWorldStep1

therefore, the PH is at least partially reversible with treatment of the LHF.

(Choices A and E) Hypoxia-induced vasoconstriction and emphysematous obliteration of the


vasculature underlie PH that occurs secondary to chronic obstructive pulmonary
disease. Although there is a small component of hypoxia-induced pulmonary vasoconstriction
in LHF, pulmonary venous congestion is the major cause of PH.
(Choice B) Congenital heart disease that causes left-to-right shunting (eg, ventricular septal
defect, atrial septal defect) can lead to PH via an increase in pulmonary arterial blood flow. In
LHF, PH occurs due to backwards transmission of pressure from increased pulmonary venous
blood volume; pulmonary arterial blood flow remains the same or decreases as LHF worsens.
(Choice D) Inflammatory large-vessel vasculitis (ie, Takayasu arteritis, giant cell arteritis) can
sometimes involve the pulmonary arteries and cause PH. However, pulmonary vascular
inflammation does not play a role in PH occurring secondary to LHF.
(Choice F) Thrombotic obstruction of the pulmonary arterial tree occurs in acute pulmonary
embolism, and some patients can develop chronic thromboembolic PH despite receiving
appropriate anticoagulation therapy. However, this patient has no history of thromboembolic
disease, and his orthopnea and crackles are more suggestive of PH due to LHF.

Telegram Group Invite Link: https://t.me/joinchat/zyFOIg0j5goxY2E1


Telegram Group Invite Link: https://t.me/USMLEWorldStep1

Sweat
duct

The ΔF508 mutation in the cystic fibrosis transmembrane conductance regulator (CFTR)
protein is the most common mutation in patients with cystic fibrosis (CF). In unaffected
individuals, the CFTR protein serves as a chloride channel that regulates the flow of sodium,
chloride, and water across the epithelial membranes of the airways, biliary tree, intestines,
sweat ducts, and pancreatic ducts.
Newborn screening for CF is performed in the United States, but positive results should be
confirmed by sweat testing. When eccrine sweat is first produced, it is isotonic with
extracellular fluid. As sweat travels through the eccrine duct to the skin's surface, chloride is
normally resorbed via CFTR, and sodium and water follow. CFTR facilitates production of
hypotonic sweat in patients without CF. However, patients with CF are unable to reabsorb
chloride and sodium in the eccrine ducts and therefore secrete sweat with high sodium and
chloride levels.
The most likely cause of this patient's symptoms (lethargy, vomiting) is hyponatremia due to
excessive salt wasting from his sweat. Risk factors for hyponatremia in patients with CF
include exclusive breast or formula feeding prior to the introduction of sodium-rich, solid foods;
exposure to high-temperature environments; and exercise. Therefore, salt supplementation is
recommended for patients with CF.
(Choice A) Chloride ions are an important contributor to extracellular fluid volume, and
hypochloremia is also very common in patients with CF due to sweat losses. In contrast,
hyperchloremia is more likely to develop in normal patients with excessive sweating due to
increased free water loss from hypotonic sweat.
(Choice B) Hyperkalemia typically occurs with cell lysis (eg, rhabdomyolysis, tumor lysis
syndrome), metabolic acidosis (due to intracellular to extracellular K+ shift), or abnormalities with
renal excretion.

Telegram Group Invite Link: https://t.me/joinchat/zyFOIg0j5goxY2E1


https://t.me/joinchat/zyFOIg0j5goxY2E1

Telegram Group Invite Link: https://t.me/USMLEWorldStep1

(Choice C) Hypernatremia classically occurs with excessive salt intake or free water loss (eg,
diarrhea, diabetes insipidus, improperly prepared infant formula). Patients with CF typically do
not experience hypernatremia as they lose excess sodium through their sweat glands.
(Choice D) Hypomagnesemia most commonly arises from urinary loss due to medications (eg,
loop diuretics), alcohol use, or uncontrolled diabetes mellitus with polyuria.
Asthma
Asthma is a complex disease involving both chronic inflammation and airway
hyperreactivity. Inhaled corticosteroids (eg, fluticasone, budesonide) are a mainstay of
asthma therapy. Corticosteroids reduce inflammation by inhibiting nuclear transcription of
inflammatory genes (in part leading to downregulation of the enzyme phospholipase A2) to
decrease the production of leukotrienes and other inflammatory mediators.
Although corticosteroids have no direct effect on bronchodilation, these drugs provide an
additional benefit in asthma by potentiating the bronchodilatory effect of beta-2 agonists (eg,
albuterol). This is accomplished by stimulating the upregulation of beta-2
receptors in bronchial smooth muscle, increasing cellular responsiveness to adrenergic
stimulus. This medication effect is similar to the permissive effect of endogenous cortisol on
blood pressure, which involves upregulation of alpha-1 receptors and increased responsiveness
of vascular smooth muscle cells to epinephrine.

Telegram Group Invite Link: https://t.me/joinchat/zyFOIg0j5goxY2E1


Telegram Group Invite Link: https://t.me/USMLEWorldStep1

(Choice A) Corticosteroids increase, rather than decrease, the production of lung surfactants
and may be used to help prevent infant respiratory distress syndrome in premature infants.
(Choice B) Guaifenesin reduces the viscosity (ie, thickness) of respiratory tract mucus and is
often used in the symptomatic management of viral upper respiratory tract
infections. Decreased mucus viscosity is not a direct effect of corticosteroids.
(Choice C) Theophylline provides benefit in asthma by stimulating bronchodilation via inhibition
of phosphodiesterase-3 and may also create an anti-inflammatory effect via inhibition of
phosphodiesterase-4. Corticosteroids may somewhat decrease phosphodiesterase activity but
likely do not have a significant effect.
(Choice D) Omalizumab is a monoclonal antibody that treats asthma by binding to free IgE to
decrease IgE binding to surface receptors on mast cells and basophils. This inhibits the
degranulation of inflammatory mediators. The anti-inflammatory effects of corticosteroids may
also contribute to a decrease, rather than an increase, in surface-bound IgE.

Telegram Group Invite Link: https://t.me/joinchat/zyFOIg0j5goxY2E1


https://t.me/joinchat/zyFOIg0j5goxY2E1

Telegram Group Invite Link: https://t.me/USMLEWorldStep1

Thorasic
duct

This patient with a pleural effusion 2 days after cervical lymph node removal most likely has
a chylothorax due to intraoperative injury of the thoracic duct.
The thoracic duct is the largest lymphatic vessel. It originates at about the level of T12 in the
abdomen where the lumbar and intestinal lymph trunks converge. It travels superiorly through
the mediastinum (posterior to the esophagus) and drains into the junction between the left
subclavian and jugular veins in the lower neck. It is most commonly injured during thoracic

Telegram Group Invite Link: https://t.me/joinchat/zyFOIg0j5goxY2E1


Telegram Group Invite Link: https://t.me/USMLEWorldStep1

procedures (eg, esophagectomy) but can be injured in neck procedures as well (eg, cervical
lymph node removal). Injury can result in leakage of lymph into either the neck or the thorax
because a transected duct may retract into the chest.
The thoracic duct collects lymph from most of the body, including the abdominal viscera, the
viscera of the left hemithorax, all tissue inferior to the umbilicus, and all left-sided tissues
superior to the umbilicus (the right head, arm, and thorax drain into the much smaller right
lymphatic duct).
(Choices A and F) The trachea and esophagus are deeper structures in the central neck that
are covered by strap muscles; they are unlikely to be injured during removal of cervical lymph
nodes in the lateral neck. In addition, although injury to the esophagus or trachea can cause
pleural effusion, such injury is typically accompanied by air accumulation (eg, crepitus in the
neck and/or pneumomediastinum).
(Choice B) The innominate artery is the first branch of the aortic arch, and it crosses the
midtrachea from left to right (in the thorax). It does not travel through the left neck. In addition,
injury to the innominate artery would cause massive blood loss, resulting in immediate
hemodynamic instability.
(Choice C) The apex of the lung can project into the neck and be injured in some neck surgical
procedures. However, injury would cause an acute pneumothorax rather than a slowly
accumulating pleural effusion.
(Choice D) The phrenic nerve, which originates from C3-C5 and travels with the internal jugular
vein, can be injured during neck procedures. However, such an injury would present with
elevation of the hemidiaphragm.

Telegram Group Invite Link: https://t.me/joinchat/zyFOIg0j5goxY2E1


https://t.me/joinchat/zyFOIg0j5goxY2E1

Telegram Group Invite Link: https://t.me/USMLEWorldStep1

Exercise induced
bronchospasn

Exercise-induced bronchoconstriction is common in patients with underlying asthma. Asthma


occurs due to a complex inflammatory response to the environment that involves Th2
lymphocytes, eosinophils, and mast cells. Eosinophils and mast cells synthesize and
release leukotrienes, which are inflammatory mediators that induce bronchial smooth muscle
contraction (bronchoconstriction), bronchial mucus secretion, and bronchial edema. The
leukotrienes mainly responsible for these responses are the cysteinyl-containing leukotrienes
such as leukotriene C4, D4, and E4.
Leukotriene receptor antagonists (eg, montelukast, zafirlukast) effectively treat asthma by
binding to leukotriene receptors on bronchial smooth muscle cells and blocking the effects of
cysteinyl-containing leukotrienes.
(Choice A) Albuterol causes short-acting bronchodilation via selective stimulation of beta 2
receptors. It is typically used as an as-needed rescue inhaler in patients with asthma.
(Choice B) Dextromethorphan suppresses cough via stimulation of sigma opioid receptors in
the CNS (it does not stimulate mu or delta opioid receptors and therefore does not cause other
typical opioid effects). It is commonly used in cold medication but is not effective for treatment
of asthma.

Telegram Group Invite Link: https://t.me/joinchat/zyFOIg0j5goxY2E1


Telegram Group Invite Link: https://t.me/USMLEWorldStep1

(Choice C) Inhaled corticosteroids (eg, fluticasone, budesonide) help treat asthma by inhibiting
nuclear transcription of inflammatory genes leading to widespread anti-inflammatory
effects. One effect is reduced activity of the enzyme phospholipase A2 to decrease the
production of leukotrienes. Corticosteroids do not block cell surface receptors.
(Choice D) Ipratropium stimulates bronchodilation via blockade of the binding of acetylcholine
(a neurotransmitter) to muscarinic receptors. Inhaled ipratropium is sometimes used in the
treatment of asthma but is more commonly used for chronic obstructive pulmonary disease.
(Choice F) Zileuton is a 5-lipoxygenase enzyme inhibitor that reduces the production of
leukotrienes (it does not block leukotriene cell surface receptors). It can be used for treatment
of asthma, but it is often avoided due to concerns about toxicity.
Squamous
metaplasia

Normal bronchi are lined mainly by pseudostratified ciliated columnar cells that propel inhaled
particles toward the oropharynx for removal by swallowing or expectoration. Interspersed goblet
cells produce mucus that traps inhaled debris. The ciliated and goblet cells of respiratory
epithelium are together responsible for mucociliary clearance.
In response to chronic irritation, such as smoking, the respiratory epithelium may be replaced
with squamous epithelium. This change of cell types is called squamous metaplasia and is
seen histologically as a transition to stratified polygonal epithelial cells. This transition is initially
adaptive, as squamous cells are more resistant to irritation than columnar cells. However, the
lack of cilia and goblet cells prevents effective mucociliary clearance and increases the risk of
respiratory infections. Squamous bronchial metaplasia is reversible and may resolve on
discontinuation of smoking, but persistent irritant exposure can cause progression to dysplasia
or squamous cell carcinoma.
Other examples of metaplasia include Barrett esophagus, in which esophageal squamous
epithelium is replaced by intestinal columnar epithelium in response to chronic acid
exposure. The metaplasia seen in Barrett esophagus is also associated with an increased risk
of malignancy.

Telegram Group Invite Link: https://t.me/joinchat/zyFOIg0j5goxY2E1


https://t.me/joinchat/zyFOIg0j5goxY2E1

Telegram Group Invite Link: https://t.me/USMLEWorldStep1

(Choice B) Squamous cell carcinoma of the cervix is caused by oncogenic strains of human
papillomavirus (HPV) that integrate into the host cell genome, leading to expression of viral
genes that inactivate host tumor-suppressor proteins. Although HPV infects the metaplastic
epithelium at the cervical transformation zone, squamous metaplasia of the cervix is a normal
response to vaginal acidity and is not associated with malignancy.
(Choice C) Hypertrophic cardiomyopathy is an autosomal dominant defect of the β-myosin
heavy chains. This abnormality leads to disordered myocardial fibers, left ventricular outflow
obstruction, arrhythmia, and increased risk of sudden death.
(Choice D) Interstitial cystitis is characterized by urinary frequency, urgency, and pelvic
pain. Gross findings include erythema and ulceration of the bladder mucosa. Histopathology
shows inflammatory infiltrates and fibrosis.
(Choice E) Minimal change disease is associated with effacement of the processes of
glomerular podocytes and is visible only on electron microscopy. The underlying cell type is not
altered.
Oxygen
transfer

Oxygen transfer across the alveolar membrane depends on the (1) gas diffusion rate and (2)
capillary blood perfusion rate. In healthy lungs, diffusion of oxygen occurs very rapidly: red
blood cells become fully saturated with oxygen at only one-third of the total alveolar capillary
length. For this reason, oxygen saturation does not fall even with large increases in cardiac
output during exercise. In other words, normal oxygen transfer is perfusion limited (ie,
diffusion is so fast that oxygen transfer depends on the perfusion rate [cardiac output] only).
Diffusion limitation is a mechanism of hypoxia that occurs in diseases that disrupt the
alveolar-capillary membrane (eg, emphysema, pulmonary fibrosis). This patient's progressive
dyspnea and fine inspiratory crackles are consistent with interstitial lung disease. Fibrotic
thickening of the interstitial space (between the air and blood) increases the distance that

Telegram Group Invite Link: https://t.me/joinchat/zyFOIg0j5goxY2E1


Telegram Group Invite Link: https://t.me/USMLEWorldStep1

oxygen must cross, limiting the degree of oxygen diffusion. During exercise, the increase
in pulmonary blood flow accelerates transit through the pulmonary capillaries, reducing the
time for oxygen extraction. In patients with diffusion limitation, the increased blood flow during
exercise can result in exertional hypoxemia, even if oxygenation is normal at rest.

(Choices A, B, and D) In healthy individuals, exercise results in increased minute ventilation


(ie, product of increased respiratory rate × larger tidal volumes). At the same time, recruitment
of apical pulmonary capillaries (increased perfusion) promotes continued ventilation/perfusion
(V/Q) matching as ventilation increases. This remarkably stable V/Q matching during exercise
enhances oxygen exchange, preventing hypoxemia.
Asthma
This patient's autopsy findings of lung hyperinflation and bronchial inflammation are suggestive
of uncontrolled asthma, a disease characterized by chronic airway inflammation, airway
hyperresponsiveness, and intermittent bronchoconstriction. Chronic inflammation, composed
mainly of eosinophils, helper T cells, and mast cells, causes airway remodeling (ie, bronchial
wall thickening, increased smooth muscle), which further worsens airway obstruction and
asthma symptoms.
Corticosteroids inhibit the production of inflammatory mediators (eg, cytokines, prostaglandins,
leukotrienes), reduce leukocyte extravasation into the respiratory epithelium, and induce
apoptosis of inflammatory cells. In addition, corticosteroids decrease smooth muscle
proliferation and mucus production by goblet cells.
Systemic corticosteroids (eg, oral prednisone) are used in short courses to treat acute asthma
exacerbations, whereas inhaled corticosteroids (eg, fluticasone) reduce the frequency and
severity of exacerbations and are used for long-term asthma control in patients with persistent
symptoms. Suppression of airway inflammation is evident within hours of administration but
reaches maximal effect after several months of inhaled therapy. Nonadherence to long-term
therapy can increase the risk of life-threatening asthma exacerbation.

Telegram Group Invite Link: https://t.me/joinchat/zyFOIg0j5goxY2E1


https://t.me/joinchat/zyFOIg0j5goxY2E1

USMLE e e r m h nne s
Me est i e s: https://t.me/USMLEMe est

K p n ssr m i e s: https://t.me/USMLEK p n

USMLE i e s Step : https://t.me/USMLE Step

e ker i e s: https://t.me/USMLE e ker

r s e n i e s Step : https://t.me/USMLE r sAn e n

r s e n i e s Step K: https://t.me/USMLE r sAn e n Step K

Ph se i e s: https://t.me/USMLEPh se

Pi ri e i e s: https://t.me/USMLEPi ri e

P th m i e s: https://t.me/USMLEP th m

nA i s i h it : https://t.me/USMLE nA i s

Am ss r p: https://t.me/USMLEAm ss

Le t ri nk: https://t.me/USMLELe t ri

En p int P s: https://t.me/USMLEEn p int

USMLE MEs: https://t.me/USMLE ME

L E Me E : https://t.me/USMLE n ineMe E

USMLE r Step : https://t.me/USMLE r Step

USMLE r Step K: https://t.me/USMLE r Step K

USMLE r Step nk: https://t.me/USMLE r Step

USMLE Anki: https://t.me/USMLEAnki

KissPrep: https://t.me/USMLEKissPrep
https://t.me/joinchat/zyFOIg0j5goxY2E1

Telegram Group Invite Link: https://t.me/USMLEWorldStep1

(Choices A and C) Albuterol is a selective beta-2 adrenergic agonist that induces


bronchodilation. Acetylcholine released through vagal stimulation mediates bronchoconstriction
via muscarinic receptors on bronchial smooth muscle cells; ipratropium is a muscarinic
antagonist that promotes bronchodilation and reduced mucus production. Neither of these
agents has anti-inflammatory properties, and although short-acting beta agonists are widely
used to provide rapid relief of symptoms during an exacerbation, they do not prevent
inflammation-induced airway remodeling or alter the chronic disease course.
(Choice D) Magnesium sulfate inhibits calcium influx into airway smooth muscle and promotes
bronchodilation when given intravenously during an acute asthma flare-up. Magnesium also
reduces inflammation by stabilizing T cells and inhibiting mast cell degranulation; however, it is
less effective than corticosteroids.
(Choice E) Leukotrienes are potent inducers of airway bronchoconstriction and
inflammation. Leukotriene inhibitors (eg, montelukast) are useful for long-term control in some
patients with asthma. However, they possess less efficacy and have fewer anti-inflammatory
properties than inhaled glucocorticoids.
(Choice F) Theophylline, a phosphodiesterase inhibitor, results in increased intracellular cyclic
AMP, which contributes to its role as a bronchodilator. Theophylline also has anti-inflammatory
effects, although they are far less potent than glucocorticoids.

Telegram Group Invite Link: https://t.me/joinchat/zyFOIg0j5goxY2E1


Telegram Group Invite Link: https://t.me/USMLEWorldStep1

Pressure volume
curve

On the above pressure-volume curve, collapsing pressure is positive and expanding pressure is
negative. The lungs have a tendency to collapse at all lung volumes; therefore, the entire
lung curve is located in the positive region. In contrast, the resting chest wall has a tendency
to expand at all but very high lung volumes; therefore, most of the chest wall curve is located in
the negative region. The point where the collapsing force of the lungs is equivalent to the
expanding force of the chest wall is the resting equilibrium of the respiratory system (ie, end-
tidal expiration), represented by the black dot); the alveolar pressure at this point is equivalent to
atmospheric pressure (ie, 0 cm H2O) and the lung volume is the functional residual

Telegram Group Invite Link: https://t.me/joinchat/zyFOIg0j5goxY2E1


https://t.me/joinchat/zyFOIg0j5goxY2E1

Telegram Group Invite Link: https://t.me/USMLEWorldStep1

capacity (FRC).

Compliance is defined as change in volume per change in pressure and is represented by the
slope of the pressure-volume curve. A highly compliant container is able to stretch to
accommodate large increases in volume with little change in pressure, demonstrating a steep
compliance curve. Generally, lung compliance is greatest around the FRC and decreases at
very high and very low lung volumes (note that surface tension causes compliance
to differ during inspiration and expiration). The chest wall has low compliance at low lung
volumes and becomes more compliant as lung volume increases with inspiration. The
compliance of the respiratory system as a whole is the combined compliance of the lungs and

Telegram Group Invite Link: https://t.me/joinchat/zyFOIg0j5goxY2E1


Telegram Group Invite Link: https://t.me/USMLEWorldStep1

chest wall (represented by the blue curve).

Intrapleural pressures are not represented on the lung pressure-volume curve. The
expanding force of the chest wall working in opposition to the collapsing force of the lungs
creates negative intrapleural pressure throughout the respiratory cycle. Inspiration is driven
by active expansion of the chest wall, which generates even greater intrapleural negative
pressure and pulls the lungs outward. Intrapleural negative pressure peaks at maximal
inspiration at a value of approximately -8 cm H2O. During passive expiration the chest wall
relaxes and the respiratory system returns to its equilibrium position, in which intrapleural
pressure is approximately -5 cm H2O.

Telegram Group Invite Link: https://t.me/joinchat/zyFOIg0j5goxY2E1


https://t.me/joinchat/zyFOIg0j5goxY2E1

Telegram Group Invite Link: https://t.me/USMLEWorldStep1

Telegram Group Invite Link: https://t.me/joinchat/zyFOIg0j5goxY2E1


https://t.me/joinchat/zyFOIg0j5goxY2E1

USMLE e e r m h nne s
Me est i e s: https://t.me/USMLEMe est

K p n ssr m i e s: https://t.me/USMLEK p n

USMLE i e s Step : https://t.me/USMLE Step

e ker i e s: https://t.me/USMLE e ker

r s e n i e s Step : https://t.me/USMLE r sAn e n

r s e n i e s Step K: https://t.me/USMLE r sAn e n Step K

Ph se i e s: https://t.me/USMLEPh se

Pi ri e i e s: https://t.me/USMLEPi ri e

P th m i e s: https://t.me/USMLEP th m

nA i s i h it : https://t.me/USMLE nA i s

Am ss r p: https://t.me/USMLEAm ss

Le t ri nk: https://t.me/USMLELe t ri

En p int P s: https://t.me/USMLEEn p int

USMLE MEs: https://t.me/USMLE ME

L E Me E : https://t.me/USMLE n ineMe E

USMLE r Step : https://t.me/USMLE r Step

USMLE r Step K: https://t.me/USMLE r Step K

USMLE r Step nk: https://t.me/USMLE r Step

USMLE Anki: https://t.me/USMLEAnki

KissPrep: https://t.me/USMLEKissPrep

You might also like